Vous êtes sur la page 1sur 112

Nina Cooper

EMQ Lecture Notes

Respiratory
SHORTNESS OF BREATH

Case 1:
68, ex-smoker
SOB on exertion, over years
Wheezy for a few years
Regular productive cough

COPD
Investigations: postbronchodilator Spirometry
o FEV1 % lower
o FEV1:FVC
Classic = gradual onset, smoker,
productive cough
Destruction of alveolar space !
decreased gas exchange !
chronic bronchitis
COPD
Smoker/ex smoker
Wheeze
Chronic cough
Regular sputum
production
Exertional
breathlessness
Frequent winter
bronchitis

ASTHMA
May/may not smoke
Wheeze
Chronic nonproductive cough
Night time waking
with breathlessness or
wheeze
Significant diurnal
variation
FHx asthma
FHx atopy

Case 2:
Fever
Cough
Weight loss
Red Ziehl-Nielson Stain
TB

Long-standing cough (>2 weeks)


Racial profiling in EMQs normally
Asian
Systemic symptoms include
weight loss, anorexia, fever and
night sweats
Investigations:
o CXR: cavitatory nodules
o Sputum samples
o Microscopy
Culture in Lowenstein Jensen
medium for 4-6 weeks

Mycobacteria, bacilli, acid-fast =


appear red with ZN stain
Treatment RIPE
o Rifampicin flu-like
symptoms, hepatitis,
stains contacts orange
o Isoniazid hepatitis,
neuropathy
o Pyrazinamide hepatitis,
arthralgia, gout
o Ethambutol ocular
toxicity + colour vision
loss

Case 3:
Steatorrhoea, diabetes
Recurrent infections
Dilated airways
Clubbing
Cystic Fibrosis
Autosomal recessive
Carrier freq. 1 in 22 for
Caucasians
Defective Cl- excretion leads to
viscous mucous
Increased infections
Also assoc. with diabetes and
infertility
Bronchiectasis = permanent
dilation of bronchioles and elastic
damage
Signet rings seen on CT
Past history of infections e.g.
pertussis = bronchiectasis

Case 4:
Shipyard worker
End-inspiratory crackles
Progressive dyspnoea
Non-productive cough
Clubbing
Mesothelioma
Secondary to asbestosis (shipbuilding/power stations) or
pneumocoliosis (coal worker)
Leads to impaction ! fibrosis !
end inspiratory crackles
Also seen in idiopathic pulmonary
fibrosis (cryptogenic fibrosing

Nina Cooper

alveolitis) = progressive dyspnoea


and non-productive cough

Case 5:
Farmer
Many flu-like illnesses over
winter
Hypersensitivy Pneumonitis (EAA)
Farmer's lung - one of the most
common forms. Due to exposure
to mouldy hay. The major antigen
is Saccharopolyspora rectivirgula
Bird-fancier's lung - one of the
most common forms. Due to
exposure to avian proteins, eg
pigeons, parakeets
Cheese-worker's lung - exposure
to cheese mould, Penicillium casei.
Malt worker's lung - exposure to
Aspergillus clavatus in mouldy
malt
Hot tub lung - exposure to
Mycobacterium avium in poorlymaintained hot tubs
Chemical worker's lung - trimellitic
anhydride, diisocyanate and
methylene diisocyanate act as the
antigens during the manufacture
of plastics, polyurethane foam and
rubber.
Mushroom worker's lung exposure to thermophilic
actinomycetes in mushroom
compost

EMQ Lecture Notes

Pancoasts Tumour
Impinges on brachial plexus and
sympathetic chain
Leads to Horners syndrome
o PTOSIS
o ANHYDROSIS
o MYOSIS

Case 3:
Weight loss
Cough
Headaches
Worse bending over
Metastases
Liver: anorexia, nausea, weight
loss, RUQ pain
Bone: pain, fractures
Adrenals: usually asymptomatic
Brain: SOL

Case 4:
Lower back pain
Tired
Constipated
Hypercalcaemia
Squamous Cell Carcinoma
35% of all lung cancers
Arises from epithelial cells
Releases PTHrP
Same actions as PTH: bone
reabsorption, tubular calcium
reabsorption
Hypercalcaemia: stones, moans,
groans, bones

LUNG CANCER
Case 1:
SOB
Weight loss
Oedamatous
Flushed
SVC Syndrome
Blockage causes oedema of face
and arms e.g. due to tumour
Facial flushing
Dilated veins over arms, neck and
chest
Morning headaches

Case 2:
Ptosis
R arm pain

Case 5:
Recent haemoptysis
Dyspnoea
Truncal obesity
Moon face

Nina Cooper

EMQ Lecture Notes

Small Cell Carcinoma


20% of all lung cancers
2 year survival: 20-40% limited
disease, <5% extensive
Neuroendocrine origin often
secrete polypeptide hormones
Early metastases
Paraneoplastic syndromes
Cushings

siADH

Ectopic
ACTH
production
Moon face
Truncal
adiposity
Muscle
wasting
Thin skin and
striae

Cancer
produces
ADH
Hyponatraemia
Nausea
Malaise
Confusion

LambertEaton
Antibodies
against
tumour
interfere
with presynaptic Ca
Channels
Hyporeflexia
Weakness
Improves
after exercise
MG worsens
with exercise

Adenocarcinoma
Goblet cell origin
30% LC
Most common type in nonsmokers
Mets common

HAEMOPTYSIS
Systematic Approach
How much?
Mixed in?
Risk of asphyxiation?
From where?
o Upper airway
o Lower airway
liquid/clots, pink/red?
o Stomach haematemesis
black/coffee grounds
Age?
o Children = pneumonia,
foreign body
o Young adults = infection,
AI
o Old = cancer, chronic
disease, CCF
Risk factors
o Smoking
o Travel
o Anticoagulant drugs/
bleeding problems
o HIV

Happen before?

Case 1:
SOB
Coughing blood mixed in with
sputum
Fever
Pneumonia
Radiological pulmonary shadowing
LRTI = no radiological changes
CAPs are spread by droplet
inhalation
Present with cough, fever and
pleuritic chest pain
Cough is initially dry but later
becomes productive and may be
blood-stained
Examination reveals bronchial
breath sounds and coarse
creptitations in affected area
Severity is assessed via CURB-65
o CRP
o Urea
o RR
o BP
o Age >65
Can be lobar or
bronchopneumonia (patchy
alveolar consolidation in both
lobes)
Investigations = CXR and sputum
MCS
Treatment = relevant abx: oral
amoxicillin or IV clarithromycin +
oral co-amoxiclav
Pathogens
S. pneumonia
o Young and middle aged
o 30% all CAPs
o Rapid-onset
o High fevers, rigors and
pleuritic chest pain
o Rusty-coloured sputum
o CXR shows lobar
consolidation
Chlamydophila pneumonia
o Mild pneumonia in
younger people
o Assoc. with sinusitis,
pharyngitis and laryngitis
o CXR shows small
segmental infiltrates
Mycoplasma pneumonia
o Adolescents and you ng
adults

Nina Cooper

EMQ Lecture Notes

Cause epidemics in 3 year


cycles
o Few chest signs
o Present erythema
nodosum, pericarditis and
haemolytic anaemia
o CXR shows patchy/lobar
consolidation and hilar
lymphadenopathy
Staph. aureus
o Complicates an underlying
pneumonia
o Can arise from other foci
e.g. osteomyelitis
o Abscesses on XR - Cavities
with air-fluid levels
Pneumocytisis jiroveci
o Pneumocystisi pneumonia
= PCP
o Most common AIDSdefining illness
o Dry cough, fever, SOB,
weight loss and night
sweats
o Low O2 sats
o Widespread infiltrates on
CXR
Legionella pneumophila
o Legionnaires disease
o Localised epidemics
o Dry cough, myalgia,
malaise and GI symptoms
o Low sodium and low
albumin
o Air conditioning units
Chlamydophila psittaci
o Contracted from birds
o Bacteria shed through
faeces and nasal
discharges
o Fever, arthralgia,
diarrhoea, conjunctivitis
and headache
o Hepatosplenomegaly
recognised
o CXR = patchy lower lobe
consolidation
Klebsiella pneumoniae
o Purulent dark sputum
o CXR = upper lobe
consolidation
o

Case 2:
70M
40 pack year history
Blood in sputum

Lung Cancer
NSCC = 80%
SCC = 20%
See above notes.

Case 3:
3 year old child
Coughing up blood
Happened when came back
from nursery
Inhaled Foreign Body
Most common cause in young
children

Case 4:
2-3 cups of coffee-like
appearance
Been drinking
Oesophagial Varices
Dilated veins at distal end of
oesophagus
Caused by portal HTN secondary
to liver disease
o Poor blood flow and high
resistance !
backpressure ! tributary
formation
o Also see caput meducae,
varices on rectum and
spider naevi
o Weak and prone to
rupture
Bleeding worsened by fact that
can no longer metabolise vit K due
to LF!

Case 5:
20 year old male from
Bangladesh
Coughing up blood for few
weeks
Bilateral hilar lymphadenopathy
TB

Risk factors
o Hx
o HIV
o Travel
o Homeless
Investigations: CXR, ZN staining,
HIV serology
BHL can be caused by TB or
sarcoid

Nina Cooper

EMQ Lecture Notes

ASTHMA
Case 1:
8M
6 month history of night cough
and intermittent wheeze
Small for age
Mild eczema
Stepwise Management of Asthma
New diagnosis
Started on step 1 = short acting
B2 agonist e.g. salbutamol,
terbutaline
Step 2: step 1 + low dose inhaled
steroid e.g. budesonide,
fluticasone
Step 3: step 2 + high dose inhaled
steroid or long acting B2 agonist
e.g. salmeterol
Step 4: high dose inhaled steroids
+ long acting B2 agonist + other
drugs e.g. theophyllines,
anticholinergics (ipratropium),
leukotriene antagonists
(montelukast), mast cell stabilisers
(sodium cromoglicate)
Step 5: step 4 + oral steroids
(prednisolone)

Case 2:
16M
Poorly controlled chronic
asthma
Managed with long acting B2
agonist and high-dose inhaled
steroids
Parents want to know if
anything else can be given
Stepwise Management of Asthma
See above

Case 3:
23 year old man with known
asthma
A&E with SOB
Widespread wheeze
Breathless to talk
HR 115BPM
RR 36
Severe Exacerbation of Asthma
Features of severe asthma

Pt is too breathless to
finish sentences
o RR>25
o Pulse >100bpm
o Peak flow <50% predicted
Should treat immediately with
60% oxygen via facemask,
back-to-back nebulisers and
systemic steroids (either oral
prednisolone or intravenous
hydrocortisone)
Patients should have their sats
measured continuously and have
regular PEFR readings + serial
ABGs
o

Case 4:
21F, known asthma
SOB
Barely talk, silent chest, RR 36
per min
Appears to be some
improvement with O2,
salbutamol nebs and steroids
Peak flows still at 30% of best
Life-threating Exacerbation of
Asthma
Features
Fatigue or exhaustion
Central cyanosis
Decreasing consciousness
Bradycardia
Silent chest or poor respiratory
effort
Peak flow <33% predicted/best
Additional treatment required IV
MAGNESIUM SULPHATE
If deteriorates may require
intubation/ventilation

Nina Cooper

EMQ Lecture Notes

Cardiology
MURMURS
Case 1:
Anacrotic pulse (slow-rising)
Breathlessness
Syncope
Pain
Aortic Stenosis
Ejection systolic murmur
Characteristic breathlessness,
syncope and angina on exertion

Case 2:
26F
Pink frothy sputum
Atypical pain
MI
Mitral Regurgitation
Can present acutely or chronically
Acute = backflow, leading to pink
sputum
Mitral valve prolapse causes
atypical pain seen in young
Graham Steell murmur heard if the
regurgitation backflows through
the pulmonary valve too, due to LA
problem
Inferior MI can cause rupture of
the posteromedial papillary muscle
while anterolateral infarctions can
cause rupture of the anterolateral
papillary muscle
This leads to acute mitral
regurgitation (RV papillary rupture
is rare but can cause regurgitation
of the tricuspid valve)
Complete rupture of the papillary
muscle is fatal and causes wideopen MR
Those with incomplete rupture
need emergency cardiac surgery
with inotropic support considered
for transient stabilisation prior to
this

Case 3:
IV drug user
Low pitch murmer
Loudest in apex

Louder on inspiration
Pulsating uvula

Aortic Regurgitation
Multiple signs due to collapsing
pulse
o De Mussets (rhythmic
bobbing/nodding of head
in synchrony with beating
of the heart)
o Duroziezs sign (systolic
and diastolic murmurs
heard over the femoral
artery when it is gradually
compressed with the
stethoscope)
o Watson Hammer Pulse
(Large volume collapsing
pulse)
o Corrigans pulse (rapid
upstroke and collapse of
the carotid pulse)
o Quinkys sign (pulsation
of capillaries in nail bed)
o Traubes sign (a 'pistol
shot' systolic sound heard
over the femoral artery)
Austin flint murmur = loudest in
apex, mid-diastolic or pre-systolic,
heard in severe aortic regurgitation
Decrescendo diastolic murmur

Case 4:
30F
SLE
Muffled heart sounds
Pericardial Effusion
Lupus -> pericarditis
Adversely affects heart function =
cardiac tamponade
Transudative (congestive heart
failure, myxoedema, nephrotic
syndrome),
Exudative (tuberculosis, spread
from empyema)
Haemorrhagic (trauma, rupture
of aneurysms, malignant effusion).
Malignant (due to fluid
accumulation caused by
metastasis)

Nina Cooper

Case 5:
22M
Positive vasalva manoeuvre
Harsh ejection systolic murmur
HOCUM
Hypertrophic Obstructive
Cardiomyopathy
Disease of myocardium without
known cause
Cause of sudden cardiac death
Valsalva maneuver increases the
intensity of HOCUM murmurs,
namely those of dynamic
subvalvular left ventricular outflow
obstruction (decreases AS, PS,
TR)

EMQ Lecture Notes

HEART FAILURE
EMQ:

80F
IHD
Dyspnoea
Orthopnoea
PND

Upon examination, you hear fine


inspiratory creps + normal ECG
Blood test = BRAIN
NATRIURETIC PEPTIDE (check
for LV HF)
Fine insp creps = pulmonary
oedema
Her BNP is positive. What should the next
test be?
GOLD STANDARD =
TRANSOESOPHAGEAL ECHO
FIRST LINE = TRANSTHORACIC
DOPPLER ECHO
She is admitted. What should the first line
treatment be?
ACE INHIBITOR

Patent Ductus Arteriosus


Tachycardia
Respiratory problems
dyspnoea - shortness of breath
Continuous machine-like heart
murmur
Cardiomegaly
Left subclavicular thrill
Bounding pulse
Widened pulse pressure

She is discharged, but later presents with a


chronic dry cough. Secondary treatment:
ANG-II RECEPTOR
ANTAGONIST
6 months later, she is put on enalapril (ACE
inhibitor), atenolol (B-blocker) and
furosemide (diuretic)
Next = SPIRONOLACTONE
Can cause gynaecomastia,
menstrual disturbances etc.

Nina Cooper

INFECTIVE ENDOCARDITIS
EMQ:

22F
IV Drug User
Tricuspid murmur
Petechiae
Microvascular haematuria
Janeway lesions
Clubbing
Splinter haemorrhages

Modified Duke criteria for Infective


Endocarditis.
MAJOR:
Positive blood culture
o Typical organism in 2
separate cultures or
o Persistently positive +ve
cultures)
Endocardium involved
o Positive echo
o Or new valvular
regurgitation
Minor
Predisposition (cardiac lesion, IVD
user)
Fever >38c
Vascular/immunological signs
Positive blood culture that does not
meet criteria
Positive echo that doesnt meet
major criteria
Diagnosis: 2 major / 1 major + 3 minor /
5 minor

EMQ Lecture Notes

Fast AF
No discernable P waves
Chaotic excitation
Irregularly irregular
Typical = old, palpitations, SOB,
fatigue, syncope
Paroxysmal = <24h, stops itself
Persistent = >7 days, requires
cardiaversion
Permanent = doctor and patient
have lost hope of sinus rhythm
Causes can be cardiac or non-cardiac
(infection, thyroxtoxicosis)
Investigations
ECG
Ambulatory monitoring

Case 2:
36M
Palpitations
150bpm
Atrial

Flutter
Narrow complex tachycardia
Atrial Rate = 250-350bpm
Ventricular Beat = 150bpm with a
2:1 block
Saw tooth appearance on ECG
Acute = shock them

May show signs of septic shock. MANAGE:


ABC, fluid resuscitate
FBC
ECG
Echo
Manage w MC&S
Has MRSA = give IV VANCOMYCIN

ARRHYTHMIAS
Case 1:
85F
Acute SOB + pain
Irregular rhythm
HR 190

Recurrent = catheter ablation

Case 3:
56M
Previous MI
Broad complex tachycardia
Ventricular Tachycardia
Broad complex = ventricular
problem (i.e. not from SAN)

Nina Cooper

Risk = previous MI or inherited


syndromes
Emergency as can turn into VF ->
ischaemia/infarction

EMQ Lecture Notes

Back pain since prosthetic valve


put in 4/52

Suspected Infective Endocarditis

Case 4:
24M
Palpitations
Preexcitation
Wolff-Parkinson White
Often benign, 1-3/1000 people
Non-conductive AV septum is
bypassed by an accessory pathway
(bundle of Kent)
Pre-excitation, slurred upstroke =
delta wave

Case 5:
84M
HR 32
Broad QRS
Complete Heart Block
Broad QRS = conduction outside of
normal pathways
SAN spontaneous
AVN slower spontaneous
(~30bpm)
See P waves in unison + QRS
randomly appearing

Case 3:
72M
Shoes too small
Exertional dyspnoea
No PMHx
Suspected Heart Failure
First line BNP for HF if no
previous MI
If previous MI = stress echo

Case 4:
66F
Pansystolic murmer
Post-MI

INVESTIGATIONS
Case 1:
54
Diabetic
SOB
Central crushing chest pain
Suspected ACS
Do ECG first

Case 2:
65M
Fever

Suspected Mitral Regurgitation


TTE

Case 5:
55M
Hyperlipidaemia
Central chest pain on exertion
Relieved by rest

Nina Cooper

EMQ Lecture Notes

Diagnosing Stable Angina


Coronary angiography

Then exercise ECG/CT calcium


scoring

CHEST PAIN
Case 1:
70M
BMI 35
Collapse running
Tearing pain to back
BP 190/120 ! 160/90
Dissected Thoracic Aortic Aneurysm
Higher risk in connective tissue
disorders
Can cause hemiplegia, acute limb
ischaemia, MI
if CA
involvement

Case 2:
32M
Stabbing pain
Worse lying down/coughing
Improved by sitting upright
Pericardial rub audible
Pericarditis
Positional pain
Can be caused by infection, SLE,
rheumatic rever ! pericardial
effusion
The classical finding on
examination is a friction rub which
is said to sound like walking on
snow
There may be diffuse ST elevations
on ECG, an effusion on
echocardiography and blood
results suggesting inflammation
Complications include tamponade
and constrictive pericarditis
Prior viral infection is a risk factor
with the most common pericardial
infection being viral
Bacterial purulent pericarditis also
occurs
Inflammation is due either to direct
viral attack or immune mediated
damage
Other risk factors include male
gender, post-MI (both early and
Dresslers), post-pericardiotomy

syndrome, neoplasm from local


tumour invasion, uraemia and
autoimmune conditions such as RA
and SLE.

Case 3:
Central crushing chest pain
Bradycardia
MI
CT
Calcium
Scoring
RULE
OUT
Low risk
factors
Atypical
pain

Functional
Imaging
INTERMEDIATE
Low risk factors
Atypical pain
Women
Includes stress
echo, MPS with
SPECT and cardiac
MRI

Angiography

RULE IN
High risk
Typical pain
Can carry out
procedure?

Bradycardia -> inferior MI


Management = MONA LISA
MORPHINE
OXYGEN
NITRATES
ASPIRIN
LOOP DIURETIC
IV ACCESS
STREPTOKINASE
ANTIPLATELETS

Case 4:
Sharp chest pain, worse on
inspiration (pleuritic)
Haemoptysis
Sudden onset
Pulmonary Embolism
Antiphospholipid syndrome can
cause DVTs, leading to PE
First-line = CTPA
D-Dimers = non-specific
Wells Scoring

Nina Cooper

Case 5:
70F
Similar pain to her angina, but
no relief with GTN
Woke up from sleep
Previous episode 1/12 ago
Unstable Angina
Episodic
Has PMHx
Occurs at rest <20min
Crescendo effect gets worse, last
longer, more frequent
Prinzmetals Angina
ST Elevation
Wake from sleep
Female, age 50
Vasospasmic (functional, not
stenosis)
Ca channel blocker (NOT B
blocker)
See same ECG signs as pericarditis

HYPERTENSION
Case 1:
Tachycardia
HTN
Anxiety
Weight loss
Phaeochromocytoma
10% rule: malignant, bilateral,
extra adrenal, hereditary
Diagnose with 24h catecholamines
Require a/B blockade, then
surgical removal

Case 2:
18
Radial-femoral delay
Ejection systolic
murmur
Aortic

Coarctation
Interscapular murmur
Decreased femoral pulses
Seen in those with Turners
Distal to L subclavian = no radialradial delay

EMQ Lecture Notes

Case 3:
Myalgia
Polyuria
Polydipsia
Conns Syndrome
Aldosterone-secreting tumour
Raised Na, Low K, normal renal
function
Diagnosis: raised aldosterone,
suppressed renin
Surgical excision + spironolactone

Case 4:
Middle aged
Hep B +ve
Polyarteritis Nodosa
Necrotising vasculitis
Associated with microaneurysms,
MI
Higher risk in hep B endemic areas
ANCA +ve in <10%
Gold standard diagnosis =
histology

Case 5:
Arteriopath
61M
T2DM
Kidneys fail 2/52 after ACE
started
Renal Artery Stenosis
Normally due to atherosclerosis
(rarer = Takayasus arteritis,
thromboembolism)
Suspect if htn resistant to optimal
treatment
Diagnosis: USS, gold standard
= renal angiography
ACE inhibitors drastically
reduce GFR in those with RAD
Causes acute renal failure
U&Es should be checked before
and after starting ACEi

Nina Cooper

EMQ Lecture Notes

Statin therapy, blood sugar


control in diabetics and BP
control with antihypertensives
may also be necessary
Those with LMS disease, 3 vessel
disease or a reduced EF may
benefit from CABG
Single vessel disease may benefit
from PCI

CASE 2/3

A 55 year old man is admitted to A&E with


chest pain which is central in origin and
came on while he was waiting for his bus.
Troponin and CK-MB are not elevated.
An overweight 63 year old male with a
history of hypertension presents with
cardiac

ANGINA
CASE 1

A 73 year old banker complains of chest


pressure which comes on predictably on
exertion. It is relieved when he sits down
and rests.
= Stable Angina
Resting ECG is often normal
Patient is asymptomatic.
During exercise stress ECG (most
often the Bruce Protocol) there will
be ST segment depression
during exercise indicative of
ischaemia and the patient will
complain of chest pain
Those unable to exercise to an
adequate level may need stress
myocardial perfusion imaging
or stress echocardiography
1st line treatment involves
lifestyle changes and
antiplatelet therapy with
aspirin.
Anti-anginal therapy will also be
given, first line being betablockade

sounding chest pain while watching TV.


However, his cardiac biomarkers are not
elevated. An ECG is ordered which shows
ST depression and T wave inversion
= Unstable Angina
Characterised by chest pain at rest
ECG will typically show ST
depression and T wave
inversion
Acute management includes
antiplatelets and
antithrombotics to reduce
damage and complications
Long term management aims at
reducing risk factors
Key risk factors include obesity,
hypertension, smoking,
hyperlipidaemia, FH, DM and
positive FH
People with diabetes may again
present with atypical symptoms
Cardiac biomarkers will not be
elevated although in a patient
who has had an acute MI days
earlier, troponin may remain
elevated (remains elevated up to
10-14 days after release)
All patients with presumed cardiac
chest pain should in the first
instance get oxygen, morphine

Nina Cooper

and GTN with antiplatelet therapy


in the absence of contraindications
CASE 4

A 59 year old woman complains of chest


pain. ECG shows ST segment depression.
However, a subsequent coronary
angiogram is normal
= Syndrome X
Chest pain with usual ST segment
changes associated with coronary
artery disease but with normal
coronary arteries
It is treated with calcium channel
blockers such as nifedipine
CASE 5

A 57 year old female complains of chest


pain which occurs at rest. ECG performed
on A&E admission shows ST elevation but
a subsequent angiogram with a
provocative agent shows an exaggerated
spasm of the coronary arteries
= Variant Angina
Caused by coronary artery
vasospasm rather than
atherosclerosis
It occurs at rest and in cycles
Many patients will also have some
degree of atherosclerosis
although not in proportion to the
severity of the chest pain
experienced
ECG changes are of ST elevation
(rather than depression) when the
patient is experiencing an attack
and a stress ECG will be
negative
Patients with Prinzmetal angina
are often treated for ACS and
cardiac biomarkers may be
raised as vasospasm can cause
damage to the myocardium
The gold standard investigation
is with coronary angiography and
the injection of agents to try to
provoke a spasm

EMQ Lecture Notes

CASE 6

A 44 year old female complains of a two


week history of tight chest pain which
occurs when she is lying down.
= Decubitus Angina
This patient has chest pain which
occurs on lying down, which is
decubitus angina by definition

CARDIOLOGY DRUGS
Alpha 1 agonist
Alpha 1 agonists such as
phenylephrine are
vasoconstrictors
Also have a use as a mydriatic
They are used as nasal
decongestants as a result of
their vasoconstrictor effect
Alpha 2 agonist
Used for treatment of glaucoma
Decrease production of aqueous
fluid by the ciliary bodies of the
eye and also by increasing
uveoscleral outflow
Beta 1 agonist
Used to treat hypotension and
bradycardias
E.g. dobutamine
Beta 2 agonist
E.g. salbutamol
Used for bronchial smooth muscle
relaxation
Adrenaline
Non-selective adrenergic
stimulation
Increases HR and BP
Beta blocker
Lower heart rate, CO and
MABP during exercise
Also act to reduce renin release
as well as the release of NA
Use ranges from migraine
prophylaxis, anxiety and

Nina Cooper

hypertension to thyrotoxicosis,
post-MI and chronic heart failure
Also useful in arrhythmias where
they act to increase the
refractory period of the AVN.
Calcium channel blockers can
increase A-V block, slowing down
heart rate and worsening severe
heart failure, bradycardia and sick
sinus syndrome.

Calcium channel blocker


Non rate slowing e.g amlodipine
- leads to arterial vasodilation
by action on vascular smooth
muscle cells
It can lead to unwanted ankle
oedema, headache,
hypotension and palpitations
Used for hypertension and
angina
The palpitations a patient may
experience are due to reflex
tachycardia from arterial
vasodilation
Rate-slowing calcium channel
blockers also exist such as
verapamil and diltiazem and uses
for these also include
arrhythmias such as paroxysmal
SVT and AF
ACE inhibitor
Prevent the conversion of
angiotensin I to angiotensin II by
ACE
Uses
o Hypertension
o Heart failure
o Post-myocardial infarction
o Diabetic nephropathy
o Progressive renal
insufficiency
o Patients at high risk of
cardiovascular disease
E.g. Enalapril
Angiotensin Receptor Blocker
E.g. losartan
Antagonists of type 1 (AT1)
receptors for Ang II, preventing

EMQ Lecture Notes

the renal and vascular actions of


Ang II
Uses: hypertension, heart failure

Organic Nitrates
Uses
o Angina
o Acute and chronic heart
failure
o BP control during
anaesthesia
Mechanism of action: Release nitric
oxide (NO) in smooth muscle cells
(nitrates) or stimulate guanylate
cyclase (nicorandil) to cause
vasodilation
Reduce preload (venous return)
Venodilation
Reduce afterload (peripheral
resistance) Vasodilation
Example: Glyceryl trinitrate,
nicorandil
Chronic use can lead to tolerance
It is also weakly antiplatelet and
has a weak direct action to
vasodilate the coronary arteries
Spironolactone
Aldosterone antagonist
(potassium-sparing diuretic)
Used to treat oedema
Used to potentiate loop/thiazide
diuretics
Loop diuretic
Used in pulmonary oedema due to
LVF
Reduces breathlessness and
reduces preload
Used for patients with chronic HF
E.g. furosemide and bumetanide
Thiazides
Inhibit sodium reabsorption at the
beginning of the distal convoluted
tubule
Act within 1-2 hours
Management of HTN low dose
produces maximal effects with little
biochemical disturbances
E.g. bendroflumethiazide,
indapamide

Nina Cooper

MISC. CAUSES OF CHEST PAIN


CASE 1

A 32 year old woman has a 2 day history


of intermittent attacks of a sharp pain over
the lower left side of her chest. The pain is
exacerbated by movements of the rib cage
and the patient tells you it becomes
difficult to breathe. She has also felt
feverish.
= Borneholme Disease
Bornholm disease is caused by
Coxsackie B virus
Symptoms include the fever seen
as well as the characteristic attacks
of severe pain in the lower chest
Exacerbated by small movements
of the rib cage, which make it
difficult for the patient to breathe
CASE 2

A 22 year old woman complains of pain in


her breast. Upon deep palpation, you
notice tenderness over the right lower
inner quadrant.
= Costochondritis
Inflammation of the costal
cartilage
CASE 3

A 40 year old man complains of pain and


swelling over the ribs
= Tietzes Syndrome
Inflammation of the costal
cartilage
Similar to constochondritis but has
swelling

EMQ Lecture Notes

CASE 4

A 70 year old man presents 1 month postMI with pain on inspiration, a low-grade
fever and central sharp chest pain, made
better by leaning forwards.
= Dresslers Syndrome
Characterised by pleuritic chest
pain, low-grade fever and
pericarditis, which may be
accompanied by pericardial
effusion
It usually presents two to five
weeks after the initial episode,
with pain and fever that may
suggest further infarction
The pain is the main symptom,
often in the left shoulder, often
pleuritic, and worse on lying down.
There may be malaise, fever and
dyspnoea
Rarely, it may cause cardiac
tamponade or acute pneumonitis
A pericardial friction rub may be
heard. The typical sound of
pericarditis is described as like the
sound of boots walking over fresh
snow

ANTIHYPERTENSIVES

Nina Cooper

EMQ Lecture Notes

GI Medicine
ANATOMY OF ALIMENTARY
TRACT

Liver

Topographical Anatomy
Costal margin: Formed by the
medial borders of the 7th through
10th costal cartilages
Rectus sheath: from xiphoid
process and 5th through 7th
costal cartilages
pubic
symphysis and pubic crest.
Contains rectus abdominis muscle
Linea alba: A slight indentation
that can sometimes be seen
extending from the xiphoid
process to the pubic symphysis
Inguinal Ligament: From ASIS
to pubic tubercle of pelvis. Folded
inferior edge of external
abdominal aponeurosis. Separates
abdominal region from thigh
Umbilicus: At approximate level
of intervertebral disc between the
L3 and L4. Marks the T10
dermatome
Oesophagus
Pierces diaphragm at T10
Stomach
Foregut structure
Found in epigastric/left
hypochondriac region

Lies deep to ribs 5-11


The common hepatic artery runs
in association with the bile duct +
portal vein in the free edge of the
lesser omentum (the
hepatoduodenal ligament part)
The common hepatic is a branch
of the aorta, and then divides into
the R + L hepatic (with the cystic
artery supplying the gall bladder
branching off the R hepatic)

Gall Bladder/Biliary Tree


Gall bladder stores + concentrates
bile
Sphincter of oddi controls passage
of juices through the ampulla of
vater
Spleen
Beneath ribs 9 through 11 on the
left side. 10th rib is axis of spleen
Kidneys
Located in loin region
Left kidney is higher than right
(pelvis at L1/2 on left and L2/3 on
right)
Duodenum
Brunners glands found in
second part of duodenum
C-shape surround head, body and
uncinate process of pancreas
L2 level
Blood supply = foregut (coeliac
trunk) ends at ampulla of vater

Nina Cooper

Ileum/Jejunum
Blood supply = midgut (SMA) arterial arcades, with connecting
vasa recta
The jejunum makes up the
proximal 2/5 of the SI, with the
ileum forming the distal 3/5
The jejunum diameter > ileum
It is difficult to distinguish the
jejunum from the ileum by
diameter alone, but there are
other features, particularly the
arterial arcades + vasa recta
o The jejunum has less
prominent arterial arcades
with longer vasa recta
o The ileum has prominent
arterial arcades with a
shorter vasa recta
Large Intestine
The large intestine consists of the
caecum (most proximal),
vermiform appendix, ascending
colon, hepatic flexure, transverse
colon, splenic flexure, descending
colon, sigmoid colon + rectum
Distinguishing features: fatty tags
(appendices epiploicae), ribbons
of longitudinal muscle (taeniae
coli) + segmented/pocketed
haustra walls
2 openings:
o Ileocaecal orifice acts as
valve preventing
movement of substances
back into small intestine
o Appendix
Blood supply = midgut until
splenic flecture ! IMA/hindgut
Innervation of Gut
Abdominal viscera is supplied by
the autonomic nervous system
Parasympathetic sensory
innervation regulated the reflex
gut function:
o Vagus nerve
o Pelvic splanchnic nerves
S2-S4
Sympathetic sensory innervation
mediate pain:
o Thoracic splanchnic T5T12 (greater, lesser, least)
o Lumbar splanchnic L1+L2

EMQ Lecture Notes

VOMITING
Case 1

80F
2 day hx of nausea and vomiting
Slightly confused
PMHx: AF, osteoarthritis, HTN
(recently diagnosed)
3/52: started on low dose
bendroflumethiazide
Also takes digoxin and codydramol
O/E: 36.8C, 56BPM, irregularly
irregular HR, BP 145/85
Mild epigastric tenderness
Na 138, K 3.1, urea 8.6, creatinine
142

Digoxin Toxicity
Hypokalaemia secondary to use of
diuretic (bendroflumethiazide)
predisposes to digoxin toxicity
Vomiting ! further hyperkalaemia
Patients are nauseated and have a
low HR

Case 2

25 student
8h hx of frequent vomiting,
unable to keep anything down
Cramp-like abdo pain
Pale, clammy, shivering
Hb 14.7, WCC 11.8, Platelet 368,
Na 135, K 3.4, urea 7.7, creatinine
70

Gastroenteritis
Secondary to staphylococcus
aureus
Short history due to preformed
enterotoxin from food
contaminated with staph.
Vomiting starts shortly after
consumption of food
Other causative organisms of
vomiting-predominant food
poisoning include Bacillus cereus,
however the hx is typically shorter
(1-2h) and symptoms start later
after ingestion
Also occurs with Salmonella,
Campylobacter and Shigella, but
the predominant symptoms are
lower abdominal pain, diarrhoea
or dysentery

Nina Cooper

Case 3

4 week old baby


4/7 projectile vomiting of curdled
milk shortly after every feed
2cm palpable mass in epigastric
region

Gastric Outflow Obstruction


Neonatal pyloric stenosis
Occurs at 3-6 weeks old
Characteristic = projectile
vomiting after every meal
Disorder is due to hypertrophy of
circular muscle in pyloric region
Dehydration and severe
electrolyte disturbances eventually
arise
Treated surgically with a ramstedt
pyloromyotomy

HAEMATEMESIS
Case 1

70M
40% burns to body
Several episodes of haematemesis

Gastric Erosion
Stress ulceration ! erosion of
stomach is a complication of
serious burns = Curlings ulcer
Other traumatic causes e.g. sepss,
organ failure and intracranial
lesions (Cushings ulcer)
Risk of bleeding is reduced with
mucosa-protecting agent or
prophylactic antacids

Case 2

32F
1 year of recurrent peptic
ulceration
Taking 40mg of omeprazole
Endoscopy reveals 2cm actively
bleeding ulcer in duodenum
CT shows 2cm mass in pancreas

Zollinger-Ellison Syndrome
Rare disorder caused by gastrinsecreting tumour
Either found in pancreatic islet
cells or duodenal wall
Large amounts of HCl produced
due to gastrin secretion
Diagnosis is suspected in pts with
recurrent ulceration and

EMQ Lecture Notes

Case 3

persistently high serum gastrin


levels
Use of PPIs can suppress
secretion e.g. lansoprazole

45M
Several episodes of vomiting fresh
blood
Drowsy
MCV = 106
Platelets = 167
WCC = 11.7
INR 2.1

Oesophageal Varices
Massive haematemesis + bloods =
likely to point to varices
Raised MCV due to alcoholinduced bone marrow toxicity
Raised INR due to severe hepatic
dysfunction
Normally caused by portal htn
secondary to cirrhosis
Upper GI endoscopy =
confirmation
Need to correct coagulopathy !
FFP + vit K
Sengstaken-Blakemore tube can
be used for balloon tamponade of
bleeding in emergency situations

CONSTIPATION
Case 1

66M
3/12 hx of difficulty passing stool
Previously opened daily
Now straining, producing wormlike stools with mucus
Tenesmus

Carcinoma of Colon/Rectum
Short hx of change in bowel habit
to constipation
Mucus + tenesmus are common in
low rectal tumours

Case 2

28F
Chronic schizophrenia
Abdominal pain, bloating and
constipation
Opens 2x weekly with passage of
hard stool
Also complains of dry mouth

Nina Cooper

Iatrogenic
Antipsychotic treatments (e.g.
chlorpromazine) have anticholinergic side effects
Leads to decreased motility of GIT
and decreased glandular
secretions ! dry mouth
Also can cause difficulty passing
urine, blurred vision + others
Will see iatrogenic constipation
with iron and opiate use

Case 3

92F
Falls and fractures right neck of
femur
Six days post-op, complains of
colicky lower abdo pain
Not opened bowels for 6-days
post-op
Faeces are palpable in left colon
and on DRE
Faecal loading seen on AXR

Simple Constipation
Treatment = alleviate causative
factors e.g. reduced food intake,
low-fibre diet, lack of exercise and
mobility
Common in elderly too
Use of laxatives should be shortterm

Case 4

EMQ Lecture Notes

Case 5

Hypercalcaemia
Likely to have carcinoma of
bronchus (cough + haemoptysis)
Can get hypercalcaemia with lung
cancer due to malignant
infiltration of bone (osteolysis +
Ca release) or due to ectopic
secretion of an PTH-like hormone
Bowel can also be affected by
auto-antibodies to myenteric
neurones in association with SCC
of the lung ! intestinal pseudoobstruction

26F
Constipation
Struggled for a few years
Topical GTN and dietary advice is
not helping

Anal Fissure
Constipation due to pain
Tx
o High fibre, high fluid diet
o Topical lidocaine
o Topical GTN ! headaches
o Botox
o Surgical

Case 6
56M
Short hx of abdo pain and
difficulty opening bowels
Not passed faeces for 6 days and
now struggling to pass urine
6/12 hx of cough with occasional
haemoptysis puts down to being
a smoker
Wife thinks he has lost weight

Groans, moans, stones, bones


Abdo pain, vomiting, constipation
Polyuria, polydipsia
Depression, anorexia, weight loss
Fatigue
Hypertension
Confusion
Pyrexia
Renal stones
Ectopic calcification
Causes
o Malignancy
o Sarcoidosis
o Primary
hyperparathyroidism

8 year old
Constipation
Distended abdomen
Normally fit
Healthy and balanced diet

Aganglionosis
E.g. Hirchsprungs disease
Absence of ganglion cells in GIT
Therefore impaired/absent
nervous control
Usually diagnosed as an infant

DIARRHOEA
Case 1

93F
Severe chest infection
10 days broad-spec abx, nebs +
O2
Develops profuse, offensive,
greenish diarrhoea

Clostridium difficile

Nina Cooper

Case 2

Antibiotic induced diarrhoea


Carried by 3-5% population
Treatment with oral metronidazole
or vancomycin
Enterotoxin is responsible for the
diarrhoea and cytotoxin is
responsible for the
pseudomembrane

22M
Severe abdominal cramps,
vomiting and watery diarrhoea
Several hours after eating poorly
reheated chinese takeway
Stool samples confirm gram
positive, aerobi bacilli

Bacillus cereus
Associated with poorly reheated
rice
Takes between 1-16h for
symptoms to present

Case 3

Middle aged couple


1 week hx of watery, severe
diarrhoea and foul-smelling
belches
Several days after returning from
Russia
Cysts and trophozoites seen on
stool microscopy

Giardia lamblia
Commonly found in Eastern
Europe and Russia
Causes malabsorption and watery
diarrhoea
Characteristic = foul smelling
flatus
Causes villous atrophy in SI
Can exist in cyst and trophozoite
form
Causes acute-phase IgM response
Treat with rehydration and oral
metronidazole

Case 4

31M
HIV +ve
Profuse watery diarrhoea, weight
loss and fever
Dilated descending colon on AXR
Oocysts present on stool sample

Cryptosporidium

EMQ Lecture Notes

Case 5

Self-limiting diarrhoea for up to 10


days
Can cause toxic dilatation of colon
in immunocomprimised patients
Hydration + nutrition + electrolyte
support

65F
Diarrhoea
Struggled with infrequent bowel
motions most of her life

Overflow Diarrhoea
History of chronic
constipation/obstruction
Liquid stool passes obstruction

Case 6

18F
Diarrhoea
Recently started OCP
Similar episodes in the past which
she manages with lifestyle
modification

Lactose Intolerance
Drug adjuvants can cause
problems too
OCP and POP both contain lactose
Intolerance is due to
hypersensitivity (reproducible,
adverse reaction). Non-immune
mediated

Case 7

62M
Bloody diarrhoea
Intense abdominal pain
Therapy for heart palpitations

Ischaemic Colitis

WEIGHT LOSS
Case 1

22F
Hypopigmented patches on
dorsum of hands
Weight loss, loose stools and
oligomenorrhoea
Examination: onycholysis, fine
tremor, resting tachycardia +
warm peripheries

Nina Cooper

Thyrotoxicosis
Hypopigmentation = vitiligo
associated with many autoimmune
disorders
Signs of graves disease
o Acropachy (pseudoclubbing of the nails)
o Orbitopathy (eye disease)
o Pre-tibial myxoedema
Start pts on carbimazole or
propylthiouricil

Case 3

51F
Weight loss, anorexia and swelling
of abdomen
Unwell, pale, signs of a left pleural
effusion, hepatomegaly and
shifting dullness in abdomen
Multiple opacities in both lung fiels

Carcinomatosis
Signs of disseminated malignancy
ascites, hepatomegaly and
multiple pulmonary mets
Most likely cause = ovarian cancer
presents late with intra-abdo
mets

DYSPHAGIA
Case 1

72M
6/12hx of progressive difficulty
swallowing
Now only swallows small amounts
of fluid
Lost 10kg in weight
Wasted appearance

Oesophageal Carcinoma
Rapidly progressing dysphagia
Weight loss + hypoproteinaemia =
highly suggestive
Histological diagnosis confirmed
by oesophagoscopy
Size of lesion seen on barium
swallow ! apple core lesion

Case 2

45M
6/12hx of progressive difficulty
with speech and swallowing
Weakness of facial muscles
bilaterally
Absent jaw jerk

EMQ Lecture Notes

Bulbar Palsy
Palsy of tongue, muscles of
mastication/deglutition and facial
muscles due to loss of function of
brainstem motor nuclei
Signs of LMN disease
Seen in motor neurone disease,
but also Guillain-Barr, polio and
brainstem tumours
NEVER affects extraocular
movements how you distinguish
from myasthenia gravis

Case 3

50F
Chest pain associated with
regurgitation of solids and liquids,
occurring shortly after swallowing
Dilated oesophagus + tapering
lower oesophageal segment
Oesophageal manometry
demonstrates failure of relaxation
of lower oesophageal sphincter

Achalasia
Failure of relaxation of lower
oesophageal sphincter
Presents between ages 30-60
Diagnosis = beak-like tapering
on barium swallow

Case 4

26M
Undergone renal transplant
3 day hx of odynophagia and
difficulty swallowing
Barium swallow + endoscopy
show generalised ulceration of
oesophagus
DHx: oral pred + ciclosporin

Oesophageal Candidiasis
Short hx of symptoms +
immunosuppression
Also seen in AIDS

DISEASES OF THE STOMACH


Case 1

63F
2/12 hx of anorexia, weight loss
and epigastric pain
Iron-deficiency anaemia

Nina Cooper

Endoscopy shows thickened and


rigid gastric wall without an
obvious mass lesion
Biopsies show numerous signetring cells diffusely infiltrating the
mucosa

Adenocarcinoma
Weight loss + anorexia + irondeficiency anaemia (chronic
bleeding)
Leather bottle stomach = gastric
carcinoma which diffusely
infiltrates all layers of the gastric
wall
Signet-ring cells = poorly
differentiated adenocarcinoma

Case 2

51M
3 month hx of weight loss +
dyspepsia
Endoscopy shows heavy mucosal
folds + 2cm antral ulcer
Biopsy shows heavy infiltrate of
atypical lymphocytes with clusters
of intraepithelial lymphocytes

MALT Lymphoma
Lymphoma of mucosa-associated
lymphoid tissue
Normal gastric mucosa is devoid
of lymphocytes any infiltrate
signifies disease
DD lies between gastritis and
lymphoma
Atypical + intraepithelial
involvement + thickened folds +
ulceration = infiltration is
lymphomatous
MALT lymphomas are the most
common type of primary gastric
lymphoma and are usually lowgrade B cell type

Case 3

26M, HIV +ve


2 week hx of dyspepsia and
epigastric pain
Endoscopy shows purple, plaque
like lesions
Biopsies show slit-like vascular
spaces surrounded by proliferating
spindle cells

EMQ Lecture Notes

Kaposis Sarcoma
Endoscopic + histological features
= typical
Also HIV +ve

Case 4

42M
Long hx of epigastric discomfort
related to meals
Endoscopy shows diffuse
erythema in antrum w/o obvious
ulceration
Antral biopsies show infiltrate of
lymphocytes, plasma cells and
neutrophils in the gastric mucosa
Staining shows h. pylori infection

Active Chronic Gastritis


Caused by h. pylori infection
Presence of neutrophils shows
active inflammation
Nb. 90% of h. pylori infections
cause duodenal ulcers

INFLAMMATORY BOWEL
DISEASE
Case 1

34F
Known UC
2 week hx of worsening jaundice
ERCP shows beading of
intrahepatic ducts

Sclerosing Cholangitis
Cholestatic jaundice with relatively
raised alk phos
Beading ! strictures and
dilatations
Principally associated with UC

Case 2

27M
CD
Several small bowel resections
Severe right sides loin pain, rigors
and haematuria
Urine microscopy shows pus cells
and red cells but no organisms

Oxalate Renal Stones


Oxalate is normally bound to
calcium in terminal ileum,
rendering it insoluble in the colon
With multiple bowel resections,
this does not occur and so leads

Nina Cooper

to hyperoxaluria and stone


formation

Case 3

26M
6/52 hx of weight loss, passage of
blood and mucus per rectum
associated with diarrhoea
Complains of red, watery eyes
UC diagnosed

Anterior Uveitis
Related to disease activity and can
be associated with presenting
symptoms
Equal presentation in UC and CD

Case 4

19F
4 week hx of abdo pain and
bloody diarrhoea
Enlarging ulcer on left shin

Pyoderma Gangrenosum
More common in UC
Seen in IBD, vasculitic disorders
and haematological malignancies

JAUNDICE
Case 1

74M
Acute chest infection
Pyrexial, icteral sclerae, RLL
pneumonia

Gilberts Syndrome
Unconjugated hyperbilirubinea
Associated with acute illness
Autosomal dominant inheritance
Acute illness + raised bilirubin
No other signs of deranged LFTs

Case 2

41M
Acute jaundice, nausea, vomiting,
diarrhoea
3 weeks after returning for SE asia
o Bilirubin 43
o AST 432
o ALT 522
o Albumin 35
o INR 1.2
Acute phase IgM response
demonstrated

EMQ Lecture Notes

Hepatitis A
RNA virus spread by faeco-oral
route
Causes acute illness associated
with jaundice, diarrhoea and fever
Anti-HVA IgM response seen and
later IgG seen

Case 3

61F
Known polycythaemia rubra vera
admitted with worsening pruritis
and jaundice
Icteric, tender hepatomegaly,
ascites, peripheral oedema
(marked sacral pad)

Budd-Chiari Syndrome
Hepatic vein obstruction
secondary to thrombosis as a
result of polycythaemia rubra vera
Disorder is known as Budd Chiari
Syndrome
Arises secondary to thrombogenic
disorders
Rarely can be due to congenital
web within HPV
Diagnosis confirmed by
USS/venography of HV and IVC
Treament: anticoagulation and
treatment of underlying cause

Case 4

52F
6 week hx of worsening jaundice,
pruritis and weight loss
Spider naevi, hepatomegaly,
jaundice
Antimitochondrial antibodies
strongly positive
Anti-smooth muscle antibodies
and anti-LKM antibodies - negative

Primary Biliary Cirrhosis


Signs of chronic liver disease
associated with acute jaundice
Autoantibodies are highly
suggestive of PBC
Heralded by onset of pruritis +
antimitochondrial antibody
Liver biopsy to determine stage +
prognosis
Prognostic markers include
albumin, bilirubin, PT, evidence of
cirrhosis and portal HTN
Liver transplantation may be
considered

Nina Cooper

Case 5

EMQ Lecture Notes

Obese 71m
2 week hx of worsening jaundice,
dark urine and pale stools
Dilated CBD and gallbladder, no
gallstones

Carcinoma of the Head of the


Panreas
Symptoms of obstructive jaundice
Painless therefore ASSUME
pancreatic cancer
Other causes include
cholangiocarcinoma, herpatic
tumours and obstructing tumours
at ampulla of Vater
Diagnosis confirmed by ERCP/CT

Case 3

DISEASES OF THE LIVER


Case 1

42M African
LFTs abnormal high
transaminases
Moderate-severe inflammation
seen on biopsy with mild fibrosis
Special stains identify antigens
from a dsDNA virus within the
cytoplasm of many hepatocytes

Chronic Hepatitis B
dsDNA = hepatitis B
Not often preceeded by an acute
hepatitic illness
Leads to hepatocyte damageby
causing expression of viral
antigens on the cell surface !
destruction by immune system

Case 2

51M
Pain in right hypochondrium
Hepatomegaly 2cm below costal
margin
Darkened skin compared to before
LFTs show markedly raised
transaminases
Biopsy shows cirrhosis and heavy
deposition of haemosiderin
within the cytoplasm of
hepatocytes and bile duct
epithelium

Genetic Haemochromatosis
Defect in chr 6, near HLA-A locus
Excessive absorption of iron in SI

Deposited as haemosiderin !
fibrosis ! cirrhosis
Can deposit in pancreas ! DM
and heart ! HF
Darkened skin is due to raised
levels of melanotrophin

17M
Symptoms and signs of chronic
liver disease
High transaminases
Chronic inflammation in portal
tracts + moderate fibrosis on
biopsy
Prominent accumulation of
copper-associated protein in
periportal hepatocytes
Low serum caeruloplasmin + Cu

Wilsons Disease
Inherited autosomal recessive
disorder
Cooper accumulates in liver and
basal ganglia of brain
Chr 13 defect ! abnormal coppertransporting ATP-ase and failure
of liver to excrete Cu in bile
Accumulation leads to chronic
hepatitis and cirrhosis
In the brain ! neurological
disability
Children present with hepatic
problems, adults present with
neurological disease
Pathognomonic sign = Kayser
Fleischer ring = Cu deposition in
cornea
Causes of Ascites
Abdominal trauma
Acute pancreatitis (high serum
amylase)
Alcoholic cirrhosis (high yGT)
Bacterial peritonitis
CCF
Hepatic vein occlusion
Malignant mesothelioma
Nephrotic syndrome (assoc. with
hypercholesterolaemia)
Ovarian carcinoma (see
adenocarcinoma cells in ascitic
fluid)
Tuberculosis (granulomas in
ascitic fluid)

Nina Cooper

EMQ Lecture Notes

HEP B SEROLOGY

Resolved
Infection
Acute
infection
Vaccinated
Low infectivity
carrier
Naive

sAg
-

eAg
-

eAb
-

cAb
+

+/-

+/-

+/-

+
+

sAg: surface antigen; eAg: e antigen; eAb: e


antibody; cAb: core antibodu

DNA virus
Signs: fever, malaise, jaundice,
hepatomegaly, arthralgia,
urticarial, deranged LFTs
Long term consequences:
o Fibrosis
o Cirrhosis
o Hepatocellular carcinoma

DISORDERS OF THE PANCREAS


Case 1

Case 3

36F
Long hx of gallstones
12hr hx of severe abdominal pain
radiating to back
Tachycardic, tachypnoeic,
hypotensive
Mildly raised bilirubin
6x serum amylase level
USS shows stone in CBD

75F
Painless obstructive jaundice
5cm mass on head of pancreas
Malignant glandular cells on
aspirate

Adenocarcinoma
Aspirate = diagnostic

56M, chronic alcoholic


Repeated attacks of abdo pain
precipitated by bouts of heavy
drinking
Loose, pale, greasy stools
Calcification in peritoneal cavity

Chronic Pancreatitis
Upper abdo pain which develops
gradually after meals/drinking
Pain radiates to back and is
relieved by sitting rowards
Calcification = chronic calcifying
pancreatitis ! intraductal calculi
Can lead to pancreatic
insufficiency

Case 4

Acute Haemorrhagic Pancreatitis


Sudden onset abdominal pain +
signs of shock
Common causes = alcohol and
gallstones
Diagnosis depends on serum
amylase
Raised amylase is also seen in
other abdo emergencies (e.g.
perfed DU) but if >5x normal =
likely to be acute pancreatitis

Case 2

Most common pancreatic


neoplasm and occurs in the elderly
Presents with painless jaundice
Weight loss + anorexia also occur

20M
Fainted during rugby match
Dizzy, palpitations then blacked
out
Previous episodes related to
sport/exercise

Insulinoma
Pancreatic islet-cell tumours than
secrete insulin
Hypoglycaemia associated with
exercise/fasting
Mostly cytologically benign

DRUG SIDE EFFECTS


Drug
Isoniazid
Ethambutol
Rifampicin
Pyrazinamide
Streptomycin

Side Effect
Peripheral neuropathy
Colour blindness
Hepatic enzyme
inducer, liver toxicity,
OCP failure
Liver toxicity
Ototoxicity

Nina Cooper

EMQ Lecture Notes

GI Surgery
UPPER GI DISEASE

Case 1 (Gallstones)
41F
Fever, abdo pain, nausea
RUQ pain
Intense shaking
Icteral sclera

Jaundice
Increase in serum bilirubin
Heme in RBCs broken down by
macrophages in spleen/bone
marrow ! unconjungated
bilirubin
Conjugated in liver
Excreted into GI tract via biliary
tree
Converted into urobilinogen in
terminal ileum
Excretion: kidneys (via blood),
sterabilogen (via faeces) and reexcreted by liver
Causes
Pre-hepatic: unconjugated
hyperbilirubinaemia e.g.
hereditary spherocytosis
Hepatic: mix ! problem with liver
excretion e.g. hepatitis, cirrhosis,
drugs, humours
Post-hepatic: conjugated
hyperbilirubinaemia !
obstruction;
gallstones/tumours/infection
Gallstones/Biliary Colic
Contraction of gallbladder against
an obstruction
Triggered by eating fatty foods
Severe pain radiating to back
Associated with nausea
Positive murphys sign
Tachycardia, non guarding
Female, fat, fertile, forty
Stagnant bile ! ASCENDING
CHOLANGITIS
Charcots triad (fever, rigors,
jaundice)
Gram ve (e.coli)
Urgen investigations watch BP

Cholecystitis can occur in absence


of gallstones
Investigations include USS and
ERCP
Treat via interventional elective
cholecystectomy

Case 2
21
Severe burns
Vomiting blood
Pyrexial
= Curlings Ulcer

Case 3
73
Persistent unexplained
dyspepsia
Dysphasia
Melaena
= Gastric Carcinoma
Red flags = anorexia, melaena,
progressive symptoms, age >55

Case 4
56
Hyperpigmented skin in left
axilla (acanthosis nigrans)
Progressive anaemia and
dyspepsia
= Gastric Carcinoma
Acanthosis nigricans
Microcytic anaemia
Dyspepsia, nausea, anorexia,
weight loss, satiety
Virchows node
Persistent unexplained dyspepsia
Vomiting of fresh blood
Leather bottle stomach, rigid thick
gastric wall
CT/MRI staging
Signet ring cells on biopsy
Treatment: partial/complete
gastrectomy

Nina Cooper

Case 4
32
CLO test positive
Abdominal pain after meals
= Peptic Ulcer

Case 5
45 GI surgeon (?stressful)
Raised INR
Haematemesis after stag night
= Oesophageal Varices
Oesophageal Carcinoma
Progressive, rapid dysphagia, weight loss,
cachexia
Low serum protein
Diagnosis via oesophagoscopy
Size assessed via barium swallow
apple core appearance
Achalasia
Lower oesophageal sphincter
failure
Food regurg
Looks like beak line on swallow
Barretts Oesophagus
Metaplasia of distal oesophageal
mucosa, consequence of GORD
Diagnosis with endoscopy, biopsy
Treatment with regular
surveillance, PPI or mucosal
ablation
Can progress to gastric carcinoma
(30-40x more likely if untreated)
Peptic vs. Duodenal Ulcer
Peptic
Duodenal
Pain immediately
Pain 2-3h after
after meals
meals/at night

Chronic Gastritis vs. MALT


Lymphocytes in stomach is bad
Chronic gastritis = h. pylori,
plasma cell and lymphoid follicles,
differentiated, erythematous,
typical lymphocytes
MALT = B cell type lymphoma,
intraepithelial involvement,
thickened folds, atypical
lymphocytes

EMQ Lecture Notes

Malignant vs. Chronic Ulcers


Malignant ! ulcerating, heaped
up epithelium around crater,
rolled, raised, everted
Chronic ! boundaries, fibrous
scar tissue, inflammation, necrosis
Kaposis sarcoma ! AIDS, purple
plaques/lesions in fundus, spindle
cells
Pyloric Stenosis
Projective vomiting
Succession splash
Hypokalaemia
Metabolic alkalosis
Seen in babies
Oesophageal Stricture
Impact pain
Regurgitation
Barium swallow; benign ! PPI,
dilatation; malignant ! resection
Oesophageal Varices
Haematemisis + macrocytic
anaemia (direct effect on bone
marrow)
Raised INR (liver toxicity)
Seen in alcoholics portal HTN
secondary to liver disease
Mallory Weiss Tear
Students, beer race
Tear in mucosa at G-O junction
Repeated retching and vomiting
Bloods normal
Self-limiting
Acute Erosive Gastritis
NSAIDs, alcohol, chemotherapy,
stress
Melaena and anaemia seen
Burns ! curlings ulcer
Intracranial lesion ! cushings
ulcer
Zollinger-Ellison Syndrome
Recurrent peptic ulceration
Haematemsis
Gastrin-secreting tumour from the
G cells of the pancreas (also in the
stomach and duodenum)
Increased HCl in gastric antrum
Multiple ulcers throughout upper
GIT
Distal ulceration ! diarrhoea

Nina Cooper

Investigation
Raised gastrin levels in serum
US, CT, angiography
60% malignant; mets to local LNs,
liver
Treatment
Lansoprazole, surgical resection
MEN1 = multiple endocrine
neoplasia 1 (ZES presents as part
of it)

Case 5
40 banker
LUQ pain
Radiating to back
Hypotensive and tachycardic
= Acute pancreatitis
Triad = HTN, low BP + resp failure
Single best marker of prognosis?
= Serum urea
Amylase = raised 5x ! likely to be
pancreatitis but can be duodenal ulcer

Case 6
52 chronic alcoholic
Presents with fullness,
epigastric pain and nausea
Jaundice
Palpable GB
= Pancreatic Carcinoma
Causes = IGETSMASHED
Acute Pancreatitis
Alcohol abuse
Epigastric pain, radiates to back,
relieved by sitting forwards
Clinical dehydration !
hypokalaemia
Haemorrhagic ! anaemia
Retching
Inv: bloods and USS of liver
WCC: raised, neutrophilia
Modified Glasgow criteria =
prognostic (PANCREAS)
A score >= 3 indicates Acute Severe
Pancreatitis
A score < 3 indicates Acute Mild
Pancreatitis

EMQ Lecture Notes

PaO2 < 8kPa (60mmhg)


Age > 55 years
Neutrophils: (WBC >15 x109/l
Calcium < 2mmol/l
Renal function: (Urea > 16mmol/l)
Enzymes: (AST/ALT > 200 iu/L or LDH >
600 iu/L)
Albumin < 32g/l
Sugar: (Glucose >10mmol/L)
Pancreatic Pseudocyst
Possible complication of
pancreatitis
Fever from infection and
haemorrhage into lesser sac
Symptoms of pancreatitis and
fullness
Insulinoma
Pancreatic islet cell tumour
Hypoglycaemia upon
fasting/exercise
Mostly benign
Palpitations, dizziness, fainting,
diplopia, confusion
Your typical young athlete
Pancreatic Adenocarcinoma
Painless, progressive jaundice
Bad prognosis
Risk factors: DM, smoking, alcohol
5th most common cause
Courvoisiers law: if the gall
bladder is palpable and the patient
is jaundiced, obstruction of the
bile duct is unlikely to be a stone
(i.e. a tumour!)
Stone ! inflammation ! fibrosis
! non-distensable GB
Older population, dark urine, pale
stools, ALP>AST (obstructive)
Inv: tumour marker CA19-9 (also
increased in biliary tract
malignancies), CT
Palliative treatment
Whipples
pancreatoduodenectomy:
resection of head of pancreas +
CBD + GB + distal stomach +
duodenum

Abdominal Masses
Case 1
60M

Nina Cooper

AAA

Severe acute onset abdo pain


BP 90/60, HR 150
Expansile umbilical mass
Screening for men >65
<4cm = reviewed annually
4-5.5cm = reviewed every 6
months
Indications for elective repair
o >5.5cm
o Growth >1cm/year

Case 2

30M, Nigerian
Abdominal mass and high fever
Mass in LUQ
Moves with respiration
Cannot palpate above it

Splenomegaly
Haematological e.g. CML
Portal HTN e.g. cirrhosis
Storage diseases e.g. Gauchers
disease
Systemic diseases
Infections

Case 3

72F
RIF mass
Firm, irregular and 5cm in
diameter
Lower edge is palpable
Hb 9.5, WCC 5, MCV 70, Pk 150

Caecal Carcinoma
Caecal lesions = weight loss and
anaemia
Sigmoid colon/rectal lesion
change in bowel habit/PR bleed
Be suspicious if >40yo presenting
with acute appendicitis

Case 4

45M, Indian
Tender swelling below right
inguinal ligament
Right hip held in flexed position
Complains of excrutiating pain
despite taking paracetamol
earlier

Psoas Abscess
Secondary to lumbar TB

EMQ Lecture Notes

Case 5

Presents with abdo pain, fever,


mass
If caused by spinal TB, also back
pain
Differential = femoral hernia
Affected hip often held in flexed
position

65yo alcoholic
Epigastric tenderness
Palpable mass
CT shows fluid filled round
mass in epigastrium

Pancreatic Pseudocyst
Late complication in 20%
pancreatitis, especially alcoholic
Typically 6-8 weeks after episode
Thick wall surrounding fluid
Can occur in pancreas or lesser
peritoneal sac
Complications: abscess, vessel
erosion, duodenal obstruction,
rupture
CT/US guided drainage if large

Case 6

50M
2 year history of dyspepsia
Sudden onset, severe
epigastric pain
Worse on movement
One episode haemetemisis
O/E = cold, sweaty, shallow
breaths
Abdo is rigid and bowel sounds
absent
CXR shows air under the
diaphragm

Perforated Peptic Ulcer


Free air under diaphragm =
perforation
Patient is shocked with abdo pain
+ worse with movement
If diaphragmatic irritation,
referred pain to shoulder tip
Do erect CXR
Immediate treatment = drip and
suck, analgesia, antibiotics, PPI

Case 7

28M
Hx of UC

Nina Cooper

Acute abdo pain, nausea,


vomiting and bloody diarrhoea
Febrile, tachycardic and
distended abdomen
AXR shows transverse colon
with diameter of 6.5cm

Toxic Megacolon
Can lead to perforation, sepsis
and shock
>6cm dilatation = diagnostic
Toxic megacolon can be treated
with intravenous fluids, antibiotics
and steroids
Colonoscopy contraindicated
Appendicitis
Clinical Diagnosis
Inflammation of vermiform
appendix
Causes = faecolith, lymphoid
hyperplasia, infection
(parasitic/viral)
Tx: non operative vs. operative
McBurneys point
Rovsings sign

Case 8

65M
Referred by GP with altered
bowel habit and rectal bleeding
over 8 weeks
Blood is bright red and painless
Reports tenesmus and weight
loss

Investigation: Flexible sigmoidoscopy


Colorectal Cancer
Predisposing factors: IBD, FAP,
polyps, smoking, low fibre diet
Genetics: 1st degree relative 1:17,
two 1st degree relatives = 1:10
Tx: must stage (TNM) !
chemoradiation/surgery
Flexi sig visualisation rectum
and sigmoid, allows for biopsy

Case 9

83F
Tiredness and weight loss over
6 weeks
Mass in RIF
Loss of 10kg

EMQ Lecture Notes

Investigation: colonoscopy
(likely bowel cancer)
Haemorrhoids
Disrupted and dilated anal
cushions
3, 7, 11 oclock when in lithotomy
position
1st degree - internal
2nd prolapse on defecation,
spontaneously reduce
3rd prolapse but require digital
reduction
4th persistent prolapse
Tx: conservative, sclerosing
agents, banding,
haemorrhoidectomy

Case 10
75M
Hx of AF
Abdo pain, vomiting and rectal
bleeding
Blood is dark with altered
constituency
O/E: pale, tachycardic, cold
peripheries
Tender abdomen
Bowel sounds difficult to here
Mesenteric Ischaemia
Hallmark: persisting abdo pain
that is out of proportion for the
clinical findings in high-risk
patients
Mesenteric angiography to
differentiate between
occlusive/non-occlusive causes
Treatment: surgical,
revascularisation or thrombolysis
Embolus ! embolectomy
Thrombus ! aorto-mesenteric
bypass

Case 11
45F
RUQ and R shoulder pain
Fever
Murphys sign positive
Investigation: USS

Case 12
23M

Nina Cooper

Abdominal pain
Started over umbilicus and now
in RLQ
Tachycardic, tender RIF and
Rovsings sign postive

Investigation: Predominantly clinical,


USS in females

EMQ Lecture Notes

GI##INFECTIOUS#DISEASES#
CAUSATIVE#ORGANISM#
Bacteria##

SPECIES#
Clostridium+difficile+
+
+
Salmonella+typhi+
+
+
+
+
Bacillus+cereus+
+
+
Campylobacter+jejuni+
+
+
+
+
Cholera++
+
Listeria+

Virus#

Noro/rota+virus+
+
Giardia+(protozoa)+
+
+
+
Enteramoeba+histolytica+
+
+
Cryptosporidium++
+
Toxomplasmosa+Gondii+

Parasite##

IDENTIFYING#FEATURES#
Caused+by+antiobiotics+
Plaques+on+colonoscopy+
+
TYPHOID+
Diarrhoea+
Rose+spots+on+chest+
Dry+cough+
+
Diarrhoea+
Reheated+rice+
+
Diarrhoea+
Unpasteurised+cheese+
Ascending+weakness+(GBS)+
Spiral+shaped+organism+on+LM+
+
Rice+water+diarrhoea+
+
Spontaneous+abortion+
+
Diarrhoea,+rapid+spread+
Pasty,+bulky+diarrhoea+
Flatulent++
Ukraine+
+
Diarrhoea+
Liver+cysts+
+
Diarrhoea+in+immunosuppressed+
+
Toxoplasmosis++
Kitten+faeces+
Encephalitis,+brain+abscess,+diarrhoea+

Nina Cooper

EMQ Lecture Notes

Neurology
Case 1:
Left sided weakness
Unable to move arm
2hrs since onset
Management of Stroke
Consider thrombolysis until 4.5h
AFTER onset of symptoms:
alteplase or streptokinase
Contraindicated in
o Haemorrhagic stroke
o Improving symptoms
o Severely impaired or
altered mental state
o At high risk of bleeding
o Pregnancy
o Recent trauma, surgery or
lumbar puncture
Alteplase = recombinant tissue
plasminogen activator
If contraindicated use aspirin

Case 2
Able to speak, but sentences
make little sense
Types of Stroke
Middle cerebral artery affected
Wernickes area =
UNDERSTANDING
Brocas area = speech formation
broken speech

Artery Regions and Symptoms


MCA

ACA

Rare
Contralateral weakness
Greater in legs than
arms/face

PCA

Receptive vs. Expressive Dysphasia


Receptive:
o Wernickes aphasia
o Left tempero-parietal
damage
o Can speak fluently but
doesnt make sense
o Grammatical mistakes
o Jumbled sentences
o Potential inability to
comprehend language
Expressive:
o Brocas aphasia
o Left frontal lobe
o Cannot initiate language
o Few disjointed words
o Unable to construct
sentences

Very common
Contralateral weakness
Greater face/arms than legs
Dysphasia
Neglect

Varied visual defects

Vertebral Artery
More common than PCA
Causes lateral medullary
syndrome
AKA Wallenbergs syndrome
DANISH
Acute vertigo (damage to
VESTIBULAR NUCLEUS)
Cerebellar signs ataxia/pastpointing/dysdiadochokinesia =
damage to cerebellar penduncle
Ipsilateral horners syndrome
o PTOSIS
o ANHIDROSIS
o MIOSIS
Posterior Inferior Cerebellar Artery
Less common than VA

Nina Cooper

Lateral medullary syndrome


Acute vertigo
Cerebellar signs
Ipsilateral horners syndrome

EMQ Lecture Notes

1 = moderate risk: warfarin (INR 2-3) or


novel drug treatment (dabigatran)
>2 = high risk: DABIGATRAN (or warf)
CHF or LVEF <40% = 1
HTN = 1
AGE >75 = 2
DIABETES = 1
STROKE/TIA/THROMBOEMBOLISM = 2
VASCULAR DISEASE = 1
AGE 65-74 = 1
SEX CATEGORY: FEMALE = 1

HEADACHE

Case 3:
57yo M
T2DM
Confusion, dizziness, anxious,
loses consciousness
Regains consciousness after 5
minutes
Most likely to be HYPOGLYCAEMIA
Autonomic Signs:
Sweating
Anxiety
Tremor
Palpitations
Tremor
Dizziness
Neuroglycopenic signs
Confusion
Drowsiness
Visual problems
Seizures
Coma
Hemiparesis (rare)

Case 4: Who should you


anticoagulate?
CHA2DS2-VASc SCORING
0 = no therapy

Case 1:
18F
2/12 pain above eye since
started revising
Feels sick
Lies in dark room to recover
Migraine
>2:
Unilateral/bilateral headache
Pulsating
Worsened by physical activity
+ >1 Nausea/vomiting
Photophobia/phonia
1/3 occur with aura
Visual
Sensory
Speech
Triggers: stress, depression, menstruation,
menopause, food
Tension Headache
Band across forehead
Associate with stress
Classically EMQ of student doing
exams

Case 2:
74M
Recent onset, severe headache
Fatigue
Weight loss
Sore throat
Red face

Nina Cooper

Temporal Arteritis
Granulomatous infiltration of small ->
large size arteries
Most common in ELDERLY
Decreased pulsatility of temporal
pulse
ESR >50
New onset of headaches
See abnormal
MULTINUCLEATED GIANT
CELLS
Jaw claudication
Scalp tenderness
Associated with polymyalgia
rheumatic
Rarer manifestations
10% show respiratory symptoms
Weight loss
Fever
Fatigue
Bruits of subclavian, axillary and
brachial arteries
Temporal redness

Case 3:
58M
Indian
Electric shooting pain when
brushing teeth
Thinning beard
Trigeminal Neuralgia
Paroxysmal attacks, varying in 220 minutes in length
Sharp/intense/burning/stabbing
sensation
Trigger factors in trigger area e.g.
brushing teeth in V3
Tic douloueux
Caused by compression of the
trigeminal nerve root (15%
tumour)

Case 4:

82F
Progressively worsening
headache
Too much alcohol
More forgetful than usual
Skull
= middle meningeal artery (extradural)
Dura Mater
= venous (subdural)
Arachnoid Mater

EMQ Lecture Notes

Pia Mater
Extradural Haemorrhage
Sharp Bang
Loss consciousness > lucid period
> compression of brain > coning
of brainstem > vomiting >
drowsiness > neurological
symptoms > death
Subdural Haemorrhage
Seen in ELDERLY and
ALCOHOLICS brain shrinks,
veins stretch and can rupture
Slow bleeding
Symptoms progressively develop
over weeks/months
Classic = fall, progressively
worsening headache >
neurological deficits
Also more likely in anticoagulated
patients

Case 5:
34M
45 minute headaches every
night
Bangs head on floor to relieve
severe pain
Cluster Headaches
Severe orbital, supraorbital or temporal
pain
Lasts 15min-3hr
Occurs in clusters
Often wakes people up at night
Agitations
Cranial

Autonomic Features
Ipsilateral lacrimation
Ipsilateral conjunctival infection
Ipsilateral rhinorrhoea
Ipsilateral ptosis
Ipsilateral oedema of eyelid

Case 6:
19M
Pakistani parents
2 months of weight loss and
headaches
Worse since getting a cold
Waking in the night with pain
Space-Occupying Lesion
Progressively worsening over time

Nina Cooper

Progressively worsening
neurological signs
Heaches which is present on
waking
Worse on coughing, sneezing,
defaecating, bending over

DEMENTIA
Case 1:
65M, HTN
Severe memory loss and
slowness of thought over
weeks
Wife reports memory problems
for one year
Marked deterioration 2/12
= Vascular Dementia

Case 2:
85F
Increasingly forgetful over past
few years
Deteriorated fails to
recognise immediate family on
occasions
Often doesnt make sense
when speaking

EMQ Lecture Notes

Passed away 3 months later

= CJD
Rapidly progressive -> death
within months
Myoclonus
Memory loss, personality change,
hallucination
PRION proteins -> sponge-like
degeneration of brain tissue
Commonly sporadic (sCJD) but
can be variant (vCJD) due to
eating contaminated meat (mad
cow disease)

Case 5:
Wife of 75M reports frequent
episodes of slowness and
inattentiveness for hours
Patient tells you he has been
experiencing vivid visual
hallucinations
Patient has increased tone in
both arms
Slow to follow motor
instructons
= Lewy body Dementia (Parkinsonian
features)
Alzheimers

Vascular
Dementia

= Alzheimers

Case 3:
61M
Recently become increasingly
aggressive
Shouting inappropriate remarks
in public
Increasingly cold and irritable
towards own family
= Picks disease (fronto-temporal
dementia)

Case 4:
72F
Rapid decline in cognitive
function over days
Memory/speech affected
Jerky myoclonic movements of
her limbs

Memory loss
Impaired speech/verbal
ability
Loss of executive function
Visuo-spatial symptoms
(gets lost)

Gradual

Stepwise

Old age, FHX

CV risk
factors +
previous
stroke

Picks
Disease
(Frontotemporal)
Disinhibition
Personality
change
Inappropriate
behaviour
Memory
often
preserved

Earlier onset
(<65)

Most
Common

Case 6:
Ex-American Football player
Deterioration in memory

Lewy Body

Hallucinations
(visual)
Sleep
disturbance
Resembles
Alzheimers
Memory,
language and
understanding
all affected
Fluctuating
impairment
Parkinsonism:
tremor,
rigidity,
bradykinesia
Histology
shows lewy
bodies

Nina Cooper

Poor mood
Struggles to find right words
when speaking
Continues to worsen over years

= Chronic Traumatic Encephalopathy


Athletes e.g. boxers, football
players who experience recurrent
head trauma
Develop features of alzheimers in
their 40s

COLLAPSE
Case 1:
21M
Large amounts of dilute urine
Tremor and dizziness after
sport, then collapses
= Hypoglycaemia
DIABETICS
Incorrect medication dose, missed
meals or exercise
Tremor, sweating, dizziness

Case 2:
36F
Collapse after finding out about
death of close family member
Unconscious for a few minutes
Limbs continue to move,
making frequent, erratic
movements
= Vaso-vagal syncope
Emotion/pain/fear/standing for too
long
Nausea, going pale
Clonic jerking (no stiffness)
Stokes-Adams Attacks
ARRHYTHMIA -> reduced CO and
LOC
Pale, slow/absent pulse ->
flushing upon recovery
Drop Attacks
Elderly woman
Weakness of legs -> fall with no
LOC or confusion

EMQ Lecture Notes

Carotid Sinus Syncope


Shaving, head turning -> dizziness
and syncope
Epilepsy
AURA
Tonic-clonic sequence
Tongue-biting, incontinence
Post-ictal confusion
Generalised Seizures
No localizing features
Tonic-clonic = loss of
consciousness, increased tone and
jerking
Absence = brief pause midsentence
Myoclonic = single sudden jerk of
limb/trunk
Simple Partial Seizure
Restricted to one part of brain, no
LOC
Can vary from motor disturbance
(Jacksonian) to a funny turn or
strange sensation
No post-ictal confusion or amnesia

Case 3:
28F
Smacking lips and strange
chewing movements at work
Stares blankly at colleagues
during episodes
Patient confused afterwards
with no recollection
Felt dj vu earlier in the day
= Complex partial seizure
Typically start with an aura, which
is the simple partial component
-> Automatisms such as lip
smacking, wandering around
Awareness is impaired

Case 4:
16M
Stiff and shakes violently
Right arm was shaking before
collapse
= Partial seizure with secondary
generalisation
Begins as a partial seizure
(sensory/motor/automatism)
Develops into LOC and
convulsions

Nina Cooper

Status Epilepticus
Seizures lasting >30 minutes with
no consciousness regained
inbetween
Tonic-clonic
Treatment
Open and maintain airway, 100%
O2
IV Lorazepam bolus + 50%
glucose
o Rectal diazepam or oral
midazolam if IV access
impossible
If seizures continue, IV
Phenytoin
If still no response -> general
anaesthetic

WEAKNESS
Approaching Weakness: categorise into
UMN and LMN, or
Brain/SC/PNs/NMJ/muscle
UMN

Spasticity
Hyper-reflexia
Babinksi

LMN

Muscle wasting
Floppy paralysis
Fasciculations

Brain

UMN signs
Focal signs according to lesion
e.g. hemiparesis, haemianopias
General signs e.g. headache,
vomiting

Spinal Cord
E.g. MS, transverse myeltitis,
trauma, malignancy
UMN signs
Focal lesions e.g. vision (optic
neuritis, nystagmus),
bladder/bowel incontinence,
bulbar signs (swallowing, speech)
Peripheral Signs
Spinal root compression, plexus
injuries
Look for LMN signs

EMQ Lecture Notes

NMJ

E.g. myasthenia gravis, Lamberteaton syndrome,


organophosphate poisoning,
botulism
Affects multiple muscle groups
LMN signs

Muscle
E.g. muscular dystrophy,
hypothyroidism
Tend to result in muscle wasting
and specific functional difficulties
relating to proximal weakness
Signs of proximal myopathy
include difficulty in getting out of
chairs and difficulty climbing stairs

Case 1:
65F
Sudden-onset right sided
weakness of face and arm
History of HTN takes
amlodipine
Right arm is rigid and hyperreflexic
= CVA

Case 2:
50M
Smoker
6 month hx of SOB and 2 day
hx of haemoptysis
Weight loss of 1 stone in 1
month
Peripheral nerve examination
shows weakness in arms/legs,
but improved when repeated
later
= Lambert Eaton Syndrome
Paraneoplastic (assoc. with small
cell lung cancer) or autoimmune
Antibodies against presynaptic
calcium channel
Proximal muscle weakness and
gait abnormalities

Case 3:
40F
Droopy eyelids
Hard to swallow, cannot
complete meal

Nina Cooper

EMQ Lecture Notes

Hyperthyroidism
Voice trails off at end of
sentences

= Myaesthenia Gravis
Antibody against ACh receptor
Oculomotor/bulbar signs
predominate

Happened 6 months ago but


resolved spontaneously
Reduced fine touch sensation
in both feet + weakness

= Multiple Sclerosis
Optic neuritis is most common
presentation
Weakness
Fatigue
Paraesthesia
Paralysis
Relapsing-remitting, secondary
progressive, primary progressive
RR: get better, then relapse, then get
better
Eventually can progress to secondary
progressive
PP: death at 5-10 years

ALCOHOL

LE
Improves with
exercise

MG
Worsens with
exercise

Less Ca in cleft,
therefore requires
multiple APs before
muscle moves

More weakness over


time as need Ache to
break down the ACh
to free up receptors

Case 4:
4M
Not able to run like peers
Waddle when walks
Falls over frequently
Difficulty climbing stairs
Positive Gowers sign
= Muscular Dystrophy

Case 5:
30F
2 day hx of pain and loss of
vision in left eye

Impacts on
Brain
Liver
Nutrition
Trauma
Social
Other substance abuse
Immunosuppression

Case 1:
45F
Post-op (gall bladder)
Agitated, shouting incoherently
and convinced she is covered
in spiders
= Delirium Tremens
Alcohol withdrawal state
Agitation, confusion, fluctuating
consciousness, seizures,
arrhythmias, visual/tactile
hallucinations
Treat with benzodiazepines
(CHLORDIAZEPOXIDE)

Case 2:
Known alcoholic
Slurred speech, cannot walk
straight, floppy sat in chair
Blood alcohol level = 0

Nina Cooper

EMQ Lecture Notes

= Cerebellar Syndrome
DANISH
Dysdiadochokinesia
Ataxia
Nystagmus
Intention tremor
Slurred speech
Hypotonia

Case 3:
39M
40 units a week for 5 years
Numbness/tingling in toes for 4
months
Impotence
= Peripheral Neuropathy
Commonest cause are DIABETES
and B12 DEFICIENCY
Glove & stocking distribution
Usually symmetrical
Can also cause impotence and
autonomic dysfunction (B12 def)

Case 4:
60 year old publican
Increasingly confused over
weeks
Vomiting for few days
Intermittent consciousness
= Subdural haemorrhage
Cerebral atrophy (alcohol!)
Frequent falls (drunk!)
Clotting disorder (liver failure!)
DD: CVS, other intracerebral
bleed, SOL, meningitis,
encephalitis)

Case 5:
50M, Irish
Mumbling incoherently
Not sure where he is
Complains of double vision
Recalls the prime minister as
Bertrand Ebstein
Claims came on moped which
he borrowed from Robbie
= Wernicke-Korsakoffs Syndrome
Due to untreated Wernickes
Anterograde amnesia
Confabulation

Alcoholics also get other types of


dementia (10% causes)

Wernickes Encephalopathy
Confusion
Ataxia
Opthalmoplegia
TX: IV Pabrinex (thiamine
[B1], riboflavin [B2],
nicotinamide [B3], pyridoxine
[B6], ascorbic acid [vit C]

Case 6:
60M
Reducing GCS
Swollen abdomen
Yellow sclera
Sickly sweet smell of breath =
FETOR HEPATICUS
= Decompensated Liver Failure
Leads to hepatic encephalopathy
Inverted sleep-wake cycle = first
sign
Due to ammonia and other waste
substances crossing BBB
Lethargy, personality change
Worsening confusion and LOC
Signs of liver failure e.g. fetor
hepaticus, jaundice
Cure = transplant

NERVE INJURY
Nerve
Median Nerve (C6-T1)

Feature
Wasting of thenar eminence
Loss of sensation in lateral palmar surface of 3 and
a half digits
Test for weakness in abductor pollicis brevis
Frequently affected in carpal tunnel syndrome

Ulnar Nerve (C8-T1)

Wasting of hypothenar eminence


Sensory loss over medial one and a half digits
Test for weakness of abductor digiti minimi
Claw hand

Radial Nerve (C5-T1)

Weakness of wrist extension ! wrist drop


Anaesthesia over fist dorsal interosseous muscle

Klumpkes Palsy

Paralysis of intrinsic muscles of hand


Loss of sensation in ulnar distribution
Horners syndrome sometimes present

Erbs Palsy

Loss of shoulder abduction and elbow flexion


Arm held internally rotated
Waiters tip

Common Peroneal Nerve (L4-S1)

Weakness in dorsiflexion and eversion of the foot


Sensory loss over dorsum of foot
Hit in side of knee

Tibial Nerve (L4-S3)

Inability to invert food or stand on tip-toe


Sensory loss on sole of foot

Stroke: Symptoms and Signs


Site of Stroke

Symptoms/Signs

Anterior Circulation

Unilateral weakness/sensory deficit


Homonymous hemianopia
Higher cerebral dysfunction e.g. dysphasia/neglect

Posterior Circulation

Cranial nerve palsy/cerebellar signs


Vertigo, dysarthria, ataxia, choking
Isolated homonymous hemianopia

Dominant Frontal Lobe


(Brocas Area)

Patient can understand you


Replies in broken speech
= Brocas (expressive) dysphasia

Dominant Temperoparietal Lobe


(Wernickes Area)

Patient has impaired comprehension


Speech is fluent with jargon
= Wernickes (receptive) dysphasia

PICA Thrombosis
(Lateral Medullary Syndrome)

Vertigo, vomiting, dysphagia


Ipsilaterally: ataxia, Horners, nerve V/VI palsy
Contralaterally: loss of pain/temperature/sensation in
face

CAUSES!OF!MENINGITIS!

ORGANISM!

SPECIES!

CNS!FLUID!!

! Treatment!

Bacterial!

S.!pneumonia!
N.!meningitides!
H.!influenzae!
Listeria!
!
M.!tuberculosis!

! Ceftriaxone/cephalosporin!(IV)!
Benzylpenicillin!!
!
Give!dexamethasone!(steroid)!for!
streptococcus!and!in!children!

Fungal!

Cryptococcus!neoformans!

Viral!

HSA!

Neutrophilia!
Low!Glucose!
High!protein!
!
!
Lymphocytosis!
Low!glucose!
High!protein!
!
Lymphocytosis!
Low!glucose!
High!protein!
!
Lymphocytosis!
Normal!glucose!
Normal!protein!
!

!
!
Management:!1st!line:!blood!culture,!gold!standard:!LP!

! Varies!!

! Acyclovir!!

HEADACHES: SUMMARY
Name

Site

Character

Severity

Time

Migraine

Unilateral or
bilateral

Can be pulsation

Moderate
severe

4-72 hours

Tension
Headache
Temporal
arteritis

Temporal Band

Band-like

Mild

N/a

Temporal

General pain

Moderate severe

New onset

Trigeminal
Neuralgia

V2/3

Sharp, stabbing,
burning

Intense

2s-20min

Cluster
Headaches

Unilateral, orbital
or supraorbital

Hot metal poking


into eye

Extremely severe

15min 3hrs

Subdural

General

General

Progressively
worsens

Worsens over
weeks/months

Extradural

General

General

Severe

Worsens over
hours

SOC

General

General

Increases in
severity can be
very painful

Progresses over
months

Associated
Symptoms
Nausea,
vomiting,
photophobia,
photophonia
None at all

Predisposing
Factors
Caffeine,
chocolate

High Risk
Groups
Women

Stress

Students

Polymyalgia
rheumatica,
respiratory
symptoms, jaw
claudication
Caused by
shaving,
brushing teeth,
wind on face etc.
Running, red and
swollen eye
Runny nose
Worsening neuro
signs

Parvovirus B19

>50 years old

None

15% have
tumour

None

All ages

Fall or head
injury

Elderly,
anticoagulated,
EtOH

Worsening neuro
signs after lucid
interval
Worsening neuro
signs

Large head injury

Worse in
morning,
coughing,
sneezing,
bending over

Nina Cooper

EMQ Lecture Notes

Renal Medicine
PHYSIOLOGICAL KIDNEY
FUNCTION
Main role is waste removal
Occurs in glomerulus
Mainly creatinine, urea and drugs
Also involved in reabsorption of
Na and H2O
Rapid exchange of Na and K in the
DCT
Renin-Angiotensin-Aldosterone Axis
Involved in regulating BP
Renin coverts ang ang I
Then converted to ang-II by ACE
Results in water retention via
aldosterone
Vitamin D Regulation
Kidney controls how much Ca is
reabsorbed
Phosphate
High phosphate ! PTH secretion
Therefore must regulate
phosphate excretion to stop bone
reabsorption and high blood Ca
Acid-Base Balance
H+ and HCO3- regulated by
kidney
RBCs

EPO produced by kidney


Influenced RBC formation (and
some platelets)

CHRONIC RENAL FAILURE


Case 1
17 child
Chronic renal failure
Severe anaemia
Iron studies within normal
limits
Most appropriate treatment
Recombinant erythropoietin
injection

Case 2

54M
Stage 3 CRF
Bone pain
Ca low
Alk phos high

Test to confirm pathology


DEXA scan
Diagnosis = renal osteodystrophy
Renal Osteodystrophy
High phosphate due to inadequate
excretion
Low Ca due to low calcitriol and
poor renal reabsorption
Increased PTH
Increased bone reabsorption
Osteoporosis on DEXA = bone
density scan
Give Ca and Vit D supplements to
reduce PTH levels

Case 3

35F
Kidney transplant 6 months
ago
Swelling in neck
Constipation, low mood
Calcifications in kidney
GFR normal

Diagnosis
Tertiary hyperparathyroidism
Hyperparathyroidism
Primary = problem with
parathyroid (adenoma)
autonomous secretion of PTH
Secondary = physiological reaction
to low Ca or high Phos ! high
PTH
Tertiary = prolonged secondary
hyperparathyroidism, so
parathyroidism becomes
autonomous ! inappropriate PTH
secretion
o Used to being in CRF
therefore gland doesnt
know what to do after
transplant as Ca is
normal!

Nina Cooper

She has stones + groans + moans


= hypercalcaemia
Swelling in neck suggests a
thyroid/parathyroid problem it is
hyperplasia

CRF Overview
Drugs contraindicated in renal failure
Potassium sparing diuretics
NSAIDs
Gentamicin
CRF

Causes

GFR <60ml/min for >3 months


5 stages
End stage is <15ml/min

EMQ Lecture Notes

Eventually
Haemodialysis
Peritoneal dialysis
Transplant

HAEMATURIA
Case 1

HTN
DIABETES
GLOMERULONEPHRITIS
Renal vascular disease
Recurrent pyelonephritis

Symptoms and Signs


Fatigue, anorexia, weakness
Vomiting, metallic, pruritis
(hyperuricaemia)
Bone pain
Dyspnoea, ankle swelling
Pallor, easy bruising
HTN (secondary to fluid retention
+ decreased renin)
Investigations
Urine dip: protein + blood
Low Hb (normocytic
normochromic anaemia also
seen in acute blood loss)
Raised urea and creatinine
Hypocalcaemia and
Hyperphosphataemia
o Secondary
hyperparathyroidism
Hyperkalaemia
Usually small kidneys on USS
can be polycystic
Renal osteodystrophy on DEXA
scan
Management
Careful fluid balance
Sodium restriction (K restriction if
hyperkalaemia)
Phosphate restriction
Vitamin D and Ca supplementation

ACE-I to control BP and save renal


function
Statins and aspirin to reduce CVS
risk
Erythropoietin or iron
supplementation for anaemia
Avoid nephrotoxic drugs e.g.
NSAIDs

60M with a 40 pack year


history
Frank haematuria

Likely diagnosis
Transitional cell carcinoma

Case 2

23F
Burning on passing urine
Dipstick +ve for blood,
leukocytes and nitrites

Best Treatment
Nitrofurantoin
Commonest pathogens
E-coli
Klebsiella
Proteus
Gut bacteria are mainly gram negative
therefore abx that can be used are
Nitrofurantoin
Ciprofloxacin
Augmentin = penicillin +
clavulanic acid (B-lactamase
inhibitor)
Trimethoprim

Case 3

18 gap year student


4 week history of terminal
haematuria
Intermittent fevers and malaise
Eosinophil markedly raised
Returned from Malawi a month
ago

Nina Cooper

EMQ Lecture Notes

RUQ pain
Thinks passing blood in urine

Likely diagnosis
Schistosomiasis
Key is terminal = lower down in
urinary tract (malaria causes
haemolysis therefore blood would
be present throughout)

Best way to confirm diagnosis


Abdo ultrasound (she has
gallstones)

Schistosomiasis
Lots of different species
S. Haematobium causes urinary
schisto (commonly in Africa and
the Middle East)
Transmission: drinking/swimming
in water with schisto eggs
Increased risk of squamous cell
carcinoma of the bladder

Differential Diagnosis
Bilirubinuria (in obstructive
jaundice)
Myoglobinuria
Food e.g. beetroot
Drugs e.g. rifampicin,
nitrofurantoin
Porphyria (urine darkens on
standing)

Case 4

MIXED RENAL SBAs/EMQs

35 year old man comes to A&E


with severe left flank pain
Comes in waves

Most appropriate next step


CT KUB
Renal Colic
Ask about
o Loin to groin pain
(writhing in agony!)
o Haematuria
o Frequency
o Dysuria
o Rigors
o Fever
o Hx of stones or recurrent
UTIs
Abx: nitrofurantoin, ciprofloxacin,
augmentin (if infection present)
Differentials for Renal Colic
Biliary colic
Pyelonephritis
Acute pancreatitis
Acute appendicitis
Dissecting aortic aneurysm
Epidiymo-orchitis
Management
Analgesia and anti-emetic
CT KUB
If septic, consider surgical
drainage

Case 5

45F
BMI 35

Case 1

50M
ITU admission
Severe sepsis
Develops acute RF

Indications for Dialysis in ARF


Persistent hyperkalaemia
Volume overload refractory due to
diuretics e.g. pulmonary oedema
Severe metabolic acidosis
Uraemic complications e.g.
encephalopathy, pericarditis
Urine Dipstick Testing
False-negatives for haematuria are
rare, hence a dipstick that is
negative for haem theoretically
excludes haematuria
Haematuria +ve test =
haemoglobinuria, myoglobinuria,
haematuria
UTI +ve but ve nitrate ! not all
organisms have nitrate reductases
OR the urine dwell time in the
bladder is too short for conversion
Dipstick insensitive to non-albumin
protiens e.g. Bence-Jones proteins
(seen in myelomas ! light chains)
Glycosuria due to renal
glycosuria can be due to genetic
conditions, DM or pregnancy

Case 2
Patient in ITU
Granular muddy brown casts in
urine

Nina Cooper

Acute Tubular Necrosis


Most common cause of ARF in
hospital
3 stages
o Initiation: Hypovolaemia,
myoglobinuria, toxins
o Maintenance: oliguria,
hyperkalaemia, fluid
overload, increasing
creatinine/BUN
o Recovery: gradual
increase in urine output
leading to high-volume
diuresis. Can get
hypokalaemia and other
electrolyte imbalances
Urinary Casts
Means it is a tubular problem
Cylindrical structures formed in
the tubular lumen therefore
pathology is intrarenal
Red blood cell casts
o Glomerulonephritis
White blood cell casts
o Acute pyelonephritis
o Tubulointerstitial nephritis
Granular casts
o Acute tubular necrosis

Case 3

Paediatric FY1 writing


discharge summary
Young boy has just recovered
from acute pyelonephritis

Scan he will need for follow up in 4-6


weeks = DMSA Scan
Basic Genitourinary Imaging
Plain abdo x-ray
o Uric acid stones are
radiolucent
USS: UT obstruction,
hydronephrosis, renal mass and
PKD
CT KUB: Gold standard for kidney
stones
MRI: Renal vein thrombosis, RCC,
replacing renal angiography
IV Urogram
Kidney anatomy
Detects renal stones
US taking over

EMQ Lecture Notes

Renal Arteriography
MRI/CT taken over
EXCEPT in polyarteritis nodosa
Multiple aneurysms and irregular
constrictions in the alrger vessels
with occlusion of the smaller
penetrating arteries
DMSA

Radionuclear
Looks for scars therefore have
to look 4-6months after
atypical/recurrent UTIs

MCUG
Micturating cystourethrogram
Catheter administered contrast
Can be used in paeds for vesicoureteral reflux and gynae for
urodynamic studies

Case 4

24
Severe gm-ve sepsis

Becomes oliguric with rising urea


and creatinine
Difficulty hearing everyone

Caused by gentamicin
Kidney damage + hearing loss
8th CN damage ! irreversible
hearing loss and vertigo
Think of 2 boxers punches
kidneys first then your head off
Chloramphenicol
Not used in western world
Causes anaemia, aplastic
anaemia and gray baby
syndrome
Doxycycline
Photosensitivity reactions
GI upset
Tooth discolouration
Erythromycin
GI discomfort
Skin rashes
Imipenem
GI upset
Skin rash
Seizures

Nina Cooper

EMQ Lecture Notes

Nephrotic vs. Nephritis


Nephrotic
Proteinuria (>3.5g/24h)
causes frothy urine
Hypoalbuminaemia (<25g/L)
Generalised oedema (periorbital)
Also risk of
o Thromboembolism renal
vein thrombosis (loss of
antithrombin and proteins
C/S ! procoagulant)
o High infection risk (loss of
Ig)
o Hyperlipidaemia liver
tries to produce more
albumin and at same time
pumps out lipids
Nephritic
Inflammatory process
Haematuria (RBC casts)
HTN
Azotemia (high urea, high
creatinine but no clinical
symptoms)/oliguria
Proteinuria <3.5g/day
Nephrotic
Acute poststreptococcal
glomerulonephritis

Both
Diffuse
prolipherative
glomerulonephritis

Rapidly progressive
glomerulonephritis

Mebranoproliferative
glomerulonephritis

Bergers IgA
glomerulonephropathy

Nephritic
Focal segmental
glomerulonephritis
Membranous
glomerulonephritis
Minimal change
disease
Amyloidosis

Alport syndrome
Diabetic
glomerulonephropathy

Membranous GN - most common cause of


nephrotic syndrome in adults
Minimal change disease most common in
children
Bergers most common cause of acute
GN worldwide

Case 5

12M
FHx renal disease males
more than females

Auditory nerve deafness,


corneal dystrophy and ocular
lens dislocation
Urinalysis shows microscopic
haematuria

Alports Syndrome
X-linked, type 4 collagen mutation
major structural component in
kidneys, eyes, ears
Haematuria, sensoorineural
hearing loss, severe ocular
abnormalities

Case 6

54M
Haematuria
Abdominal discomfort
CT scan of the abdomen shows
PKD and liver cysts

Autosomal Dominant PKD


Adult (autosomal dominant)
Multiple, large, bilateral cysts
gradually destroying kidneys
Presents with progressive UTIs,
haematuria, HTN, flank pain
Muts in PKD1/2
Associated with polycystic kidney
disease, berry aneurysms and
mitral valve prolapse
Autosomal Recessive PKD
Can cause RF in utero
Potters syndrome
Neonatal period ! HTN, portal
HTN and progressive renal failure
Associated with congenital hepatic
fibrosis

Case 7

13F, Caucasian
Discoloured urine
3 weeks earlier had facial
impetigo that was treated
Urinalysis shows haematuria,
mild proteinuria and occasional
RBC casts

Post-streptococcal GN
Discoloured urine
3 weeks ago typical presents 1-4
weeks later

Nina Cooper

Facial impetigo/URTI infection !


both caused by staph
RBC cast = nephritic syndrome
Immune complexes deposit in
glomeruli
Frequently seen in children
Resolves spontaneously

Bergers (IgA Nephropathy)


Acute gastroenteritis or URTI
Usually 1-3 days after start of
infection
IgA deposits in mesangium
causing micro/macroscopic
haematuria
Resolves within 2-3days and
usually benign
30% can progress to ESRF over
20 years

Case 8

26M
Haemoptysis
Condition worsens ! few
weeks later has renal failure
Significant haematuria and
HTN
Renal biopsy shows linear
immunofluorescence

Goodpasteures Syndrome
Middle aged, haemoptysis, linear
immunofluorescence
Antibodies formed against alveolar
and glomerular basement
membrane (type 4)
Whole of glomerular membrane is
destroyed ! linear pattern
Wegeners Granulomatosis
(Granulomatosis with polyangitis)
Small vessel vasculitis (c-ANCA
+ve)
URTI nasal septum perf, chronic
sinusitis, mastoiditis
Renal: haematuria, red cell casts
LRTI haemoptysis

Case 9

7M, Caucasian
Stung by bee 2 weeks ago
Generalised urticarial
Required hospitalisation
Later developed a puffy face
and now reports foamy urine

EMQ Lecture Notes

Renal biopsy was taken !


podocyte infusion on EM

Minimal Change Glomerulonephritis


Usually post URTI but can be
caused by immunisation and
allergy
Light micros = normal
EM = podocyte fusion
Responds to corticosteroids
Most common cause of GN in
children

Case 10
20F
Butterfly rash
Puffiness around eyes, mouth
and ankles
HTN
Blood/protein in urine
Lupus Nephritis
Primary = involves only glomeruli
Secondary = involves glomeruli
and other organs
Immune complexes circulate in
blood and deposit in glomeruli
causes an inflammatory process
Spectrum from nephrotic !
nephritic ! RF

Case 11
58M
Obese
Blood in urine
Abdo CT shows right sided
renal mass
Biopsy ! rounded and
polyglonal cells with abundant
clear cytoplasm
Grawitz Tumour
Man + old + obese = risk for
renal cell carcinoma
RCC also known as clear cell
carcinoma and grawitz tumour
Clear cell = loads of fat ! look
like yellow/gold upon excision
70-90% renal tumours, mean age
55
Arises from renal tubule
epithelium
Other RF = von-Hippel Lindau and
smoking
Metastasise to lung and bone

Nina Cooper

Haematuria + abdo (flank) pain +


renal mass

Case 12
76M
Progressive lower back pain for
2 months
Nagging and constant pain
not relieved by rest/position
change
Urinary urgency, notcutria,
frequency and hesistancy
Prostatic Carcinoma
Mets to bone ! pain
Adenocarcinoma
80% peripheral zone
presents as above with weight
loss, bone pain, raised ALP = mets
DRE = craggy, nodular, loss
midline sulcus
BPH: hyperplasia of transitional zoneincreases over time naturally
Irritative symptoms = nocturia,
frequency
Obstructive symptoms = poor
stream, hesitancy, dribbling,
terminal haematuria, overflow
incontinence, UTIs

Case 13
3 year old
Large abdo mass and
haematuria
Wilms Tumour
Commonest intra-abdo tumour
<10yrs
Peak 2-4 years
Presentation = large flank mass
+/- haematuria
Nephroblastoma (mesoderm:
kidney, fat, bone)
Rapidly growing

Case 14
62M
Used to work in rubber
industry
Recurrent UTIs and painly
haematuria
Transitional Cell Carcinoma
M:F 4:1
Peak 60-70

EMQ Lecture Notes

Painless haematuria (NO casts)


Recurrent UTIs
Voiding irritability
Pee SAC
o Phenacetin
o Smoking
o Aniline dies
o Cyclophosphamide

Squamous Cell Carcinoma


10% cancers
Associated with chronic
inflammation/irritation
Developed = long term catheters
Developing = schistosomiasis

ACUTE RENAL FAILURE


Pre Renal
Renal
Hypoperfusion

Renal
Glomerular
disease

Volume depletion

Tubular injury
e.g. ATN

Post-Renal
Obstruction
Calculus

Oedematous
states

Enlarged
prostate

Vasculitis
Bladder tumour

Hypotension
Cardiovascular

Acute interstitial
diseases e.g.
NSAIDs

Blood clot
Urethral stricture
normally due
to cystoscope

Case 1
90F
Complains of back pain
GP prescribes diclofenac
5 days later ! A&E
Generally unwell and confused
In ARF
Intersitial Renal Failure
NSAIDs
Diclofenac strong NSAID usually
prescribed for renal colic and
muscular pain
Inhibits prostaglandins required
to maintain renal perfusion
pressure
Analgesic nephropathy ! more
long term, aspirin/paracetamol,
renal scarring

Case 2

12M
D&V

Nina Cooper

Urea 23.1, creatine 220, K 5.7,


Na 134
Stool sample shows E. Coli
O157

Haemolytic uraemic syndrome


ARF
Microangiopathic haemolytic
anaemia
Thrombocytopenia
Nephrotoxins ! RBC and platelet
destruction
Supportive treatment: ? temporary
dialysis

Case 3

68M
History of IHD
Generally unwell after new
tablet from GP for HTN

70M
A&E ! confusion and abdo
pain
Large suprapubic mass
Urea 16
Creatinine 130

Post-renal failure
Prostatic hypertrophy
Leads to urinary retention
High pressure in renal pelvis !
decline in renal function
Treat: urinary catheter

Case 5

30M
Ambulance after motorcycle
accident
Large blood loss
Urea 20
Creatinine 220
K 5.7
Na 138

CT shows no visible damage to


kidneys

Hypovolaemia
Causes pre-renal failure

RENAL INVESTIGATIONS
Case 1

Pre-renal Failure
Renal artery stenosis
Block ACE ! reduced Ang-II !
effert arterioles cannot constrict
! decrease GFR ! ARF
ACEi and ARBs are contraindicated
in patients with RAS
Doppler US check for RAS
Gold-standard = angiography

Case 4

EMQ Lecture Notes

25M
Severe flank pain and vomiting
4 episodes today
Pale, sweaty and tender in
right loin
Urine dipstick
o +++ Blood
o + Protein
o No nitrites/leukocytes

Renal Colic
CT KUB = first investigation
Loin ! groin pain
Vomiting, pallor, sweating, fever,
haematuria
90% radiopaque
Key sites: renal pelvis, PUJ, VUJ
Cannot see urate/xanthine stones
Give diclofenac analgesia
Associated with zidovudine and
other HIV meds
Treatment
o Hydration
o NSAIDs
o If large ! shockwaves,
surgery

Case 2

34F
2 day hx pain passing urine,
frequency and offensive smell
+ve nitrates and leukocytes
+ Blood, + protein

UTI

Case 3

MSU MC&S
First line ! dipstick
Before starting abx, send of MSU
then start broad-spectrums
Once sensitivities are known,
adjust abx therapy

16M
Renal transplant

Nina Cooper

Decline in renal function over 2


weeks
Pain around transplant site
US shows no significant
abnormalities

Transplant Rejection
US guided renal biopsy
Used for diagnosis and severity
Acute = steroids, chronic = re-list
for transplantation

Case 4

2M
Recurrent UTIs
Failing to thrive
Strong FHx for vesico-ureteric
reflux

Vesicoureteric Reflux
UTI in kids
Urine enters ureters and renal
pelvis during micturition
Leads to renal scarring and failure
First to USS
Also DMSA scan and micturating
cysturethrogram

Case 5

27M, Indian
8 week history of weight loss,
night sweats, loin pain and
dysuria
Urine dip shows blood and
leucocytes
Urine sample shows no
growth

TB

Three early morning urine samples


for MC&S and Ziehl Neelson
staining
Sterile pyuria always renal TB
See leucocytes with negative
culture
Non-specific symptoms may
resemble UTI
Positive mantoux

EMQ Lecture Notes

Nina Cooper

EMQ Lecture Notes

Haematology & Rheumatology


RHEUMATOLOGY
Sjogrens Syndrome
Schirmers test filter paper left
for 5 minutes, dry if got sjogrens
Xerophthalmia (dry eyes,
conjunctivitis sand in my eyes)
Arthritis
Xerostomia (dry mouth, dysphagia)
Temporal Arteritis
Jaw claudication
Unilateral headache
Irreversible blindness due to retinal
artery occlusion
Treat with steroids

Scleroderma
CREST syndrome
Calcinosis calcium deposits in
skin
Raynauds
Esophageal dysfunction
Sclerodactylyl thickening and
tightening of the skin on fingers
and hands
Telangiectasia red marks on skin
surface

Dermatomyositis
Heliotrope rash
Gottrens papules
Autoimmune combining
inflammatory myopathy and
cutaneous involvement
Myopathy ! prox muscle
weakness
>50 years old = cancer
CK elevated

Osteoarthritis
Heberdens and bouchards
B Below Bouchards

Rheumatoid Arthritis
Swan neck and boutonniere
deformities
Z-thumb
Dorsal interossei wasting
Joint subluxation
Rheumatoid nodules
Ulnar deviation

Nina Cooper

Scars
Prominent ulnar styloid

EMQ Lecture Notes

Nails of hand are pitted and


lifted of skin at distal edges
Scaly rash

Psoriatic Arthritis
Asymmetric and patchy
involvement
DIP involvement
Dactylitis (sausage fingers)
5% of those with psoriasis

Tophaceous Gout
Multiple large tophi affecting the
wrists and several finger joints

Case 3:
31M
Red/itchy eyes for 8 hours
Pain on urination and diffuse
joint pain for one month
Negative test for
gonorrhoea/chlamydia
Negative for rheumatoid factor
but HLA-B27 positive
Diarrhoea 2/12 ago
Reiters Syndrome

MISC. RHEUM
Case 1:
75 woman
Sudden onset pain in right knee
Red, swollen and painful
Temp = 38.6
Gout

Needle shaped
Negatively birefringent
X-ray finding shows
chrondrocalcinosis

Acute Hot Swollen Joint


Trauma, septic arthritis and crystal
arthropathy (gout/pseudogout)
Septic: may see pus, culture
s.aureus/streptococcus/gonorrhea
Pseudogout = positive
biregringent, rhomboid shaped
weakly

Case 2:
56f
Pain in hands and swelling of
fingers
Mainly in DIPs

Case 4:
27M
6/12 hx of lower back pain
Stiffness wakes him up in the
night
Worse in morning
Severe lumbar spine limitation
Negative for ANA/RF but HLA
B27 positive
Ankylosing Spondylitis
Disease of spine and sacroiliac
joints
Fusion of joints = bamboo spine
Extra articular features = 5 As
o Anterior uveitis
o Achilles tendonitis
o Apical lung fibrosis
o Aortic incompetence
o Amyloidosis in kidneys
Seronegative Spondyoarthropathies
Arthritis without RF
Strongly associated with HLA-B27
More common in men
Enteropathic Arthritis
o Crohns and UC
Reactive Arthritis
o After acute GI/chlamydia
o CANT SEE: Conjunctivitis
and anterior uveitis

Nina Cooper

EMQ Lecture Notes

o
o

CANT PEE: urethritis


CANT CLIMB A TREE:
arthritis

HAEMOLYTIC ANAEMIA

= Premature RBC haemolysis


Normal RBC lifespan = 120 days
Classification = intrinsic or extrinsic
o Intrinsic = RBC problem
" Hereditary
spherocytosis
" PK/G6PD
deficiency
" HbC
" SCA
" Paroxysmal
nocturnal
haemoglobinuria
o Extrinsic
" Autoimmune
" MIHA
(microangiopathic)
" MAHA
(macroangiopathic
haemolytic
anaemia)
" Infection (malaria)
Confirming haemolysis
o LOW Hb
o Unconjugated bilirubin is
HIGH
o Urine urobilinogen is HIGH
o LDH HIGH (found in
RBCs)
o Blood film typical signs
for each
o Haptolobin LOW if
intravascular mops up
free Hb

Case 1:
7F
Nigeria
Screaming that bones in
hand/leg are excruitatingly
painful
Hb = 90
Sickle

Cell Anaemia
Abnormality in B-globin chain
Glu!Val at position 6
Causes of sickling
o Hypoxic conditions Hb
stick together in RBC =
high altitude, anaesthesia

and excessive exercise


Dehydration increased
MCHC, increased chance of
sickling
o Infection and inflammation
Trait: symptoms only in extreme
hypoxia
Diagnosis: electrophoresis and
neonatal screening
o

Case 2:
Child admitted with septicaemia
Platelets 30, APTT 60s, PT 20s
and low fibrinogen
DIC

Microangiopathic Haemolytic
Anaemia
o Thrombocytic
thrombocytopenia purpura
(TTP)
o Haemolytic uraemic
syndrome
o DIC
DIC
o Widespread activation of
coagulation system
o Common in obstetric
complication
o Septicaemias
o Cancers (pancreatic
carcinomas or AML)
TTP
o Widespread activation of
platelets
o Genetic
o Autoimmune process
o Pentagon:
" CNS abnormalities
" Fever
" Renal pathology
" Thrombocytopenia
" MIHA
HUS
o Widespread activation of
platelets
o E-coli infection
TTP vs. HUS : both
thrombocytopenia but HUS more
renal and not CNS
DIC vs. TTP/HUS: doesnt have
thrombocytopenia but raised DDimers

Nina Cooper

Case 3:
24 teacher
Atypical pneumonia
Yellow sclera
Patient has a positive coombs
test
Autoimmune Haemolytic Anaemia
Coombs test: binds to
autoantibodies for RBCs
Myocoplasma pneumonia ! cold
agglutinin haemolytic anaemia
>50% of patients with
mycoplasmic pneumonia develop
cold agglutinin HA

EMQ Lecture Notes

FBC shows mild anaemia


Osmotic fragility test positive
Negative Coombs test

Hereditary Spherocytosis
1/5000
North Europeans
Autosomal dominant
Weak scaffolding in RBC
membrane structure membrane
gets damaged causing a spherical
RBC
Gets stuck in spleen
Osmotic fragility: hypertonic
solution, water should move into
RBC ! no extra membrane,
cannot expand

LEUKAEMIA AND LYMPHOMA

Case 4:
28 Mediterranean couple
Honeymoon to Namibia
Husband becomes unwell
Blood film shows bite cells and
Heinz bodies
G6PD Deficiency
5-7% worlds population
Mediterranean and African
populations
X linked recessive
Pathology: hexomonophosphate
shunt pathways, no production of
NADPH, no free radicals mopped
up but GSH, therefore damage to
RBCs from free radicals (oxidative
attack and haemolysis)
Oxidative crisis: infection, drugs
(primaquine, quinine), fava beans
Symptoms usually 2-3 days
following oxidative stress and self
limiting
Heinz bodies: identified with a
supra vital stain
Bite cells: due to macrophage
removal of membrane

Case 5:
Northern European boy
Enlarged spleen

Case 1:
5 year old boy
Seizures and lymphadenopathy
Investigations reveal
pancytopenia (everything down
in bloods)
ALL

Case 2:
40M
Severe haemorrhage
Blood films shows schistocytes
(broken down RBCs)
APML

Case 3:
72M
Weight loss, fever and night
sweats
Symptoms get better with
Imatinib
CML

Case 4:
16F
Afro-Caribbean
Mass on jaw
Starry sky appearance on jaw
Burkitts Lymphoma

Nina Cooper

Case 5:
51m
Pain on drinking alcohol
Reed Sternberg Cells
Haematology starts him on
radiotherapy in combination
with ABVD
Nodular Sclerosing Hodgkins
Lymphoma

EMQ Lecture Notes

BLOOD CELL LINEAGE

AML

Should have no blasts on films live in BM


LEUKAEMIAS
Progeny of single transformed cells
Acute lymphoblastic leukaemia
Acute myeloid leukaemia
Chronic myeloid leukaemia
Men > Women
L comes before M in alphabet, AML
can be a form of CML
Therefore age of presentation
o Kids for ALL
o AML and CML in older
patients
ALL

Increased risk in Downs


More common in age 1-3
80% 5 year survival if young
Presentation: B SYMPTOMS,
anaemia, lymphadenopathy, bone
pain, easy bruising, multiple
infections
CNS involvement: headache,
vomiting, meningism, cranial nerve
palsy
Testes in Boys

GOLD STANDARD BM aspirate,


20% blasts
Investigations
o FBC: pancytopenia,
increase in WCC
o Definitive diagnosis if
>20% blasts in BM
aspirate (Jamshidi needle,
PSIS)
Management
o Supportive: blood
products, broad spec abx,
co-trimoxazole, hydration,
allopurinol, rasburicase
o WBC >200 !
hyperviscosity,
leukophoresis
o Renal failure ! dialysis
Treatment
o Systemic chemotherapy
o In AML, give for 3 years as
have to target BBB/testes
o Stem cell transplant
Mean age 60-70 (OLD PEOPLE)
Tranlocation of t(8,21)
Acute promyelocytic leukaemia =
subtype of AML, medical
emergency as high risk of bleeding
and DIC, septic shock and
hyperviscosity syndrome
Presentation: older men, short
presentation of B symptoms,
anaemia, easy bruising, multiple
infections
Investigations
o Pancytopenia (aleukaemic
leukaemia)
o Auer rods and lysine
acetylation (APML =
GIANT CELLS,
schistocytes)
o Definitive diagnosis =
>20% BM cells in aspirate
o Immunophenotyping/cytog
enic analysis for risk
stratification
o Myeloperoxidase, sudan
black B, non specific
esterase
APML ! give ATRA (or arsenic
trioxide), only 30% cure
AML ! supportive therapy,
systemic chemotherapy via central
venous catheter

Nina Cooper

CML

Philadelphia chromosome t(9,22)


! BCR-ABL fusion protein leading
to increased tyrosine kinase
activation
In old people
Precursor to AML
Presentation: old man with
incidental finding or long
presentation of B symptoms
Splenomegaly, bruising and
bleeding, gout, hyperviscosity,
retinal haemorrhage
Bi/triphasic disease
o Chronic (5% blast cells)
o Acceleration (10-19%)
o Blast crisis (20% - Auer
rods)
Investigations
o FBC ! anaemia of
chronic disease, increased
WCC
o Blood film shows
increasing blast cells
o FISH analysis checks for
fusion genes
Management
o Imatinib if Philadelphia
chromosome positive and
in chronic phase inhibits
tyrosine kinase
o Blast crisis ! allogenic
SCT
o Still refractory: donor
lymphocyte fusion

LYMPHOMA
Cancer
degree
by

of the lymph nodes controlled


of genomic instavility accentuated
Inherited
Viral; EBC, HTLV
Enviromntal; mutagens and
chronic immune stimulation
Iatrogenic; radiotherapy,
chemotherapy
Immunosuppression

EMQ Lecture Notes

80% non-Hodgkins, 20% Hodgkins


More lazy the lymphoma, the
worse the prognosis
Hodgkins
One site of swollen lymph nodes
Spreads adjacently
Male > female
Bimodal distribution ! classical
20s, nodular lymphocyte
predominant in 50s
o Classical: nodular
sclerosing (related to EBV)
o Lymphocyte depleted
bad prognosis
Present with B symptoms, rubbery
neck node, pain on drinking
alcohol and tobacco intolerance
Nodular Sclerosing: Reed
Sternberg/Hodgkin cells,
Owls eyes inclusions
Lymphocyte Predominant: L and
H cells, can return as B cell
lymphoma
Staging: 1-4, A/B with PET
o 1 = lone LN
o 2 = spread to neighbour
LN
o 3 = spread below
diaphragm
o 4 = extends beyond LNs
Manage with ABVD and extensive
radiotherapy
Non-Hodgkins Lymphoma
Can be a B cell or T Cell
Divided into low grade and high
grade
High grade = good prognosis due
to fast dividing rate, can be
targeted
B cells are CD3 negative, T cells
are CD3 positive
B Cell Lymphoma: LOW GRADE
CLL: chronic lymphocytic
leukaemia (actually a lymphoma!)
o Occurs in elderly
o Fragile
o Smudge/smear cells
under microscopy
o Treatment: GO-GO
(rituximab), SLOW-GO,
NO-GO (supportive
treatment, chlorambucil)

Nina Cooper

EMQ Lecture Notes

MALT
o

o
o
o

Mantle
o
o
o

Has CD5+ve marker as so


premature
Chronic antigen stimulation
e.g. chronic h. pylori
infection
Associated with Sjogrens
and thyroiditis
Only lymphoma that can
be treated without chemo
Give triple therapy: PPI
and 2 x abx
Zone Lymphoma
Males
Affects GI tract
Cyclin D1 + CD5+ve

B Cell Lymphoma: HIGH GRADE


Diffuse large B Cell lymphoma
Burkitts Lymphoma
o Mass in jaw
o Associated with EBV
o Give R-CHOP
T Cell Lymphoma
CD3 +ve
Large numbers of aggressive T
Cells
Enteropathy associated: Crohns

Nina Cooper

Year 3 Notes

Haematology Overview
ANAEMIA

Normal for = 110-140, = 130-150


Indication for transfusion if
symptomatic and <80

Symptoms
Tiredness
SOB
Examination
Conjunctival pallow
Tachycardia hyperdynamic state
causing flow murmur
Classification: via MCV
Normal = 80-100
Microcytic = <80
Macrocytic = >80
Microcytic Anaemia
IRON DEFICIENCY
Dietary ! vegetarian/vegan
Increased loss
o Crohns/Coeliac/Short
bowel syndrome !
Malabsorption
o Loss in bowel e.g.
bleeding
o Menstruation
Treat with ferrous
sulphate/fumarate will see
darkened stool and causes
constipation
HERIDITARY: Thalassaemia
Seen in mediterraneans
/ types
o 4 alleles and 2 alleles
Alpha severity depends on
number of genes mutated
o One gene = no effect
o Two genes = symptoms
of mild anaemia = alpha
thalassaemia trait
o Three genes = HbH
disease require lifelong
chronic blood tranfusions
o Four genes = death in
utero
Beta

Major: require lifelong


blood transusions
o Intermedia: nontransfusion dependent
Only cure = BM transplant and
cord blood transplantations
Chronic blood transfusions ! iron
overload
Manage with iron chelation
(desferioxamine)
Signs
o Hepatosplenomegaly
o Extramedullary
erythropoiesis ! jaw
protrusion abd big
forehead
o Hair-on-end XR
appearance
o

= LOW concentration of haemoglobin

Macrocytic Anaemia
B12/FOLATE DEFICIENCY
Lack in diet
Lack absorption
B12/Folate
Required for purine synthesis,
which is key for RBC production
Therefore abnormal RBCs are
produced
Need to check for coeliac in young
check for anti-endomysial
antibodies
Pernicious Anaemia
Lack of intrinsic factor binds to
B12/folate
Can be due to antibody against
parietal cells (which produce IF)
or antibody directly against IF
itself
Nb. Absorption occurs in the ileum
Use schilling test to diagnose
pernicious anaemia
o Give vitamin B12 to
saturate stores
o Radiolabelled vitamin B12
given alone, if this is low
in the urine then there is
a malabsorptive problem
o Radiolabelled IF/Vit B12
are given ! if the urine
level increases then there

Nina Cooper

Year 3 Notes

is a lack of IF and so it
must be replaced

o
o

Other Causes
Alcohol
o Simple = macrocytic
o Can develop into
megaloblastic
Hypothyroidism
Liver disease
Drugs
o Methoxtrxate
o Phenytoin
o Leads to functional folate
deficiency
Megaloblastic Anaemia
Large erythroblasts with immature
nuclei in BM
Hypersegmented neutrophils
Haemolytic Anaemia
Same symptoms + signs as usual
anaemia PLUS
Jaundice (prehepatic) due to
increased unconjugated bilirubin
Can be intravascular or
extravascular
Causes of Haemolytic Anaemia
SICKLE CELL
o HbS
o Typically African/AfroCaribbean child
o Trait ! few symptoms,
only at altitude or extreme
hypoxia
o Risk with parvovirus: RBCs
have a shorter lifespan in
SSA, and you cannot
synthesize enough RBCs
quickly enough to recover
! manage with abx,
transfusion, oxygen and
analgesia
o Deep seated bony pain =
BM infarction
o Give SC morphine
HEREDITARY SPHEROCYTOSIS
o Mutation in spectrin !
spherical RBCs
o Buzzword = osmotic
fragility test
ENZYMATIC DEFICIENCY
o E.g. G6PD

G6PD
o
o
o
o

X-linked recessive seen


in men more
Cannot protect cell from
reactive oxidative species
Triggered by
" Moth balls
" Fava beans
" Nitrofurantoin
" Anti-malarials
" Infection
Deficiency
RBC haemolysis
G6PD = protective against
oxidative stress
Occurs after exposure to
dapsone/antimalarials
Denatured Hb ! HEINZ
BODIES

ACQUIRED
Autoimmune Haemolytic Anaemia
Use Coombs Test/DAT
Check for RBC antibodies
Cold-activated = mycoplasma
infection
Warm-activated =
lymphoproliferativedisease e.g.
CML/lymphoma + SLE
Paroxysmal cold haemoglobinuria
= Donalth-Landsteiner Test
Paroxysmal Noctural
haemoglobinuria = Hams Test
Non-autoimmune = Microangiopathic
Haemolytic Anaemia
Diseases of small BVs !
haemolysis
HUS: Haemolytic Uraemic
Syndrome
o E-coli ! blood cell
haemolysis
TTP: thrombotic thrombocytopenic
purpura
o Clots and low platelets
o Rash
o Affects brain
DIC: disseminated intravascular
coagulation
o Septic/post-natal/vascular
insult
o Low platelets
o Pro and anti thrombotic
state
o Low fibrinogen and clots
o Manage with fluids, O2,
abx and stop the bleeding

Nina Cooper

Metal Heart Valve


RBCs hit metal and get
fragmented
Causes haemolytic anaemia
Normocytic Anaemia
Chronic disease
Liver disease
Blood loss (acute)
Anaemia of Chronic Disease
Rheumatoid arthritis ! can get
folate/B12 deficiency secondary
to methotrexate use
Cancer
Chronic inflammation
Vague symptoms
Aplastic Anaemia
Pancytopenia
Hypocellular BM
Absent reticulocytes

MYELOPROLIFERATIVE
DISORDERS
Polycythaemia Rubra Vera
Myeloproliferative disorder
V167F mutation in JAK2 kinase
causes uncontrolled stem cell
proliferation, usually of the RBC
line
Present with headaches,
dizziness and stroke
Hyper mast cell degranulation can
cause severe pruritis,
characteristically after hot
baths/showers (aquagenic) and
peptic ulcers
Patients may also present with
gout due to elevated cell turnover,
splenomegaly and plethora

Year 3 Notes

Essential thrombocythaemia
Platelets >600
Usually old/middle-aged patients
Increased clot risk

LEUKAEMIA
ALL: Acute Lymphocytic Leukaemia
Commonest cancer of childhood
Assoc. with Downs Syndrome
AML <3 + downs
ALL >3 + downs
Presentation
Enlarged LNs
Hepatosplenomegaly
Prone to infections
Low platelets = bleed
Low RBCs = anaemic
Blood Film: Blast cells
Treat with chemotherapy and BM
transplant
AML: Acute Myeloid Leukaemia
More common in adults
Although also seen in those under
3 with downs
Auer rods seen on film
Subtype: acute promyelocytic
anaemic (APML)
o RARA-PML is a fusion
protein
o Treat with transretenoic
acid (vitamin A)
CLL: chronic lymphocytic leukaemia
More likely to die with than from
Normally incidental finding
Chronically raised WBC count
Smear cells seen on film

Secondary Polycythaemia
Condition causing excessive RBC
production
COPD ! chronic low levels of
oxygen
EPO-secreting tumour

CML: chronic myeloid leukaemia


Seen in those aged 25-40 years
Due to Philadelphia
chromosome = BCR-ABL mutation
(9,22)
Treat with imatinib

Idiopathic Erythrocytosis
Isolated erythrocytosis with no
change in WVC or platelets
High risk of AML progression

LYMPHOMA

Either Hodgkins or Non-Hodgkins


Hodgkins Lymphoma
Reed-Sternberg/owl-shaped
cells seen

Nina Cooper

Continuous spread
Painful LNs after ethanol
Lymphadenopathy
Hepatosplenomegaly
Infection
Bleeding
Anaemia
Treat with ABVD chemotherapy

Non-Hodgkins
Non-continuous spread
Burkitts lymphoma
o Black people
o Mass on jaw
o Fast growing and
disfiguring
Treat with R-CHOP chemotherapy
Different Subtypes and Management
Use B staging
A = no sweats, weight loss of
fevers
B = B symptoms
o FLAWS
o Fever, Lethargy, Appetite
Loss, Weight Loss,
Sweating

Year 3 Notes

<1 = greater chance of


clot
o >1 = greater chance of
bleeding
Aim for 2-3 in most patients, but
for 3-4 in metallic heart valve
If INR is too high, give vitamin K
o INR>6: ?bleeding
" Yes: IV vitamin K
+ FFP
" No: Oral vitamin K
Monitors EXTRINSIC pathway (a
pEt)
o

For PE/DVT
X1, known cause ! 3 months
warfarin
X1, unknown cause ! 6 months
Recurrent ! 1 year then review
Heparin
Protamine sulfate = reverser of
unfractionated heparin
Nothing easily reverse LMWH
Monitor w APTT = INTRINSIC
(apInt)

MYELOMA
Cancer of plasma cells in BM
Lack of antibody production
Unexplained backache
Pathological fractures
Recurrent bacterial infections

Haemophilia
X-linked, only seen in males
A: factor 8 (stabilizes vWF,
therefore loss ! decreased vWF)
B: factor 9 (AKA Christmas
Disease)
Causes hypocoagulable state

COAGULATION

Von Willebrand Disease


Males and females
VWF starts the coagulation
cascade
Causes excessive bleeding

Warfarin
Inhibits vitamin K
Blocks clotting factors II, VII, IX,
X and C&S
C/S are anticoagulant and get
blocked quicker than the clotting
factors, so you initially get a
hypercoagulable state
Give LMWH with warfarin to
prevent clots, until INR is in target
range
Given for PE/DVT, AF, metallic
heart valve
INR Targets
PT is affected by warfarin
INR shows how the clotting of a
patient compares to someone else
in their age group (i.e. is a ratio)
o 1 = normal

Vitamin

K Deficiency
Seen in liver disease
Vitamins ADEK are fat soluble
Therefore seen in pancreatitis and
biliary tree problems too

Low platelets and clotting factors leads to


an increased of bleeding
Low platelets
o Fine bleeding
o Gums/mucosa
o Petechiae
Low CFs
o Haemarthroses
o Delayed bleeding

Nina Cooper

Protein C/S deficiency

Essential Thrombocythaemia
Clonal proliferation of
megakaryocytes
Epistaxis (nosebleeds)
Bleeding from gums and GIT
Picture of thrombosis and bleeding
Idiopathic Thrombocytopenic Purpura
(ITP)
Platelet dysfunction
Menorrhagia + epistaxis +
purpura
Antibody of platelet leads to
removal by reticuloendothelial
system, reducing platelet lifespan
to just a few hours

Year 3 Notes

Disorder

INR

APTT

Thrombin Time

Platelet Count

Bleeding Time

Liver Disease

N/

N/

N/

Platelet Defect

()

Vitamin K
Deficiency

Haemophilia

Von Willebrands

()

Heparin
DIC

!
!

Notes

AST

HYPOCOAGULABLE
Name

Description

HYPERCOAGULABLE
Features

PLATELET DISORDERS:
low platelets ! fine bleeding from gums/mucosa, petechiae
Decreased BM
Aplastic anaemia, megaloblastic anaemia, marrow
production of
infiltration (leukaemia/myeloma), marrow
platelets
suppression e.g. cytotoxic drugs, radiotherapy
Essential
Clonal proliferation of
Epistaxis (nosebleeds)
Thrombocytaemia
megakaryocytes
Bleeding from gums
and GIT
Picture of thrombosis
and bleeding

Name

Description

Features

CONGENITAL
Factor Von Leiden

Mutation of factor V !
resistance to protein C

Protein C Deficiency

Congenital absence of
Protein C

Increased risk of DVT


5% Caucasians
suspect if <45
Increased risk of VTE

ITP
Idiopathic
Thrombocytopenic
Purpura

Platelet dysfunction:
Antibody to platelet
leads to removal by
reticuloendothelial
system, reducing
platelet lifespan to just
a few hours

Menorrhagia
Epistaxis
Purpura
megakaryocytes in
bM

Protein S Deficiency

Vitamin K-dependent
anticoagulant, causing
activation of factor V
and VIII

DVT
DIC
PE

HUS
Haemolytic Uraemic
Syndrome

Initially endothelial
damage triggers
thrombosis, platelet
consumption and fibrin
strand deposition,
mainly in the renal
vasculature. This leads
to decreased platelets
and increased bleeding
risk.

Abdominal pain
Bloody diarrhoea
ARF

Blood Group O

Lower levels of factor


VII and VWF naturally

4x increased risk

Bernard Soulier
Syndrome

Glycoprotein Ib
deficiency = receptor
for VWF

Glanzmanns
Thrombaesthenia

Glyocoprotein IIb/IIIa
deficiency = receptor
for fibrinogen

CLOTTING FACTOR DISORDERS:


Low CFs ! haemarthroses and delayed bleeding
Haemophilia
A: factor 8
(Stabilises VWF)
B: factor 9
(Christmas disease)
Liver Disease
Decreased clotting
factor production
Decreased absorption
of vitamin K
Abnormalities of
platelet function
Malabsorption
Vitamin K deficiency !
reduced synthesis of
factors II, VII, IX, X

Von Willebrand
Disease

VWF starts clotting


cascade ! absence
leads to
hypocoagulable state

Bleeding gums
Menorrhagia
Epistaxis
Bleeding time
Bleeding gums/GIT
Menorrhagia
Epistaxis
Bleeding time
ACQUIRED
X-linked
Males only
Crippling arthropathy
Haemathroses
Alcoholic/chronic liver
disease patient
Bleed a lot

Disease

Malignancy, sickle cell anaemia, nephortic


syndrome, polycythaemia vera, essential
thrombocytosis

Antiphospholipid
Syndrome

Autoimmune
hypercoagulable state
caused by
antiphospholipid
antibodies

Have a malabsorptive
picture +
hypocoagulable state
Treat with FFP/vit K
Vitamins ADEK are fat
soluble, therefore also
see if have pancreatitis
or a biliary tree
problem
Males and females
Normal PT + platelets
Abnormal PTT
Abnormal bleeding time

Iatrogenic

OCP use

Other

Obesity

Arterial and venous


thrombosis
Multiple miscarriage

Year 3 EMQ Notes

Haematology

Nina Cooper

LEUKAEMIA, LYMPHOMA & MYELOMA


Name
Myeloma

B Cell Lymphoma

Description
Myeloma =
Cancer of the bone marrow
Affects the plasma cells
(antibody producing cells)
Non-Hodgkins
Seen in older people

Hodgkins Lymphoma

T Cell Lymphoma

Reed-Sternberg Cell (special


type of B cell)
Continuous LN spread
Non-Hodgkins

Symptoms
Unexplained backache
Pathological fractures
Recurrent bacterial infections
Malaise
Widespread, intractable itch
Appetite loss
Weight loss
Lethargic, low mood
Fevers
Night sweats
Enlarged, rubbery LN
Pain in LN after drinking
Hepatomegaly/splenomegaly
Infection
Bleeding
Anaemia
Fatigue
Weight loss
Night sweat

Diagnostics
Bone Marrow Aspirate
showing increased plasma
cells
Ig profile
Biopsy LN

Treatment
Chemotherapy with
autologous stem cell
transplantation

Owl-shaped cells (ReedSternberg Cells)

ABVD chemotherapy

Lymphoblastic change
Invades CNS

Remission induction with


chemotherapy

R-CHOP

Consolidation:
Maintenance by clearing
CNS of residual cells
Stem cell transplant

Can causes rashes/lumps if


cutaneous TCL
Can also be caused by NK cells
Chronic Myeloid
Leukaemia

Philadelphia chromosome
(9,22) translocation = BCRABL

Insidious onset
Aged 25-40 years

Hypercellular BM
Cytogenetic testing

Imatinib

Year 3 EMQ Notes

Chronic lymphocytic
leukaemia

Acute Myeloid Leukaemia

Acute Lymphocytic
Leukaemia

Haematology

Seen in elderly
More likely to die with than
from

Often asymptomatic

Adults
Seen in <3s with Downs
APML = subtype, treat with
vitamin A (transretinoic acid)
RARA-PML fusion protein

General weakness
Fatigue
Weight loss
Blood in urine/stools
Bruising easily
Hepato/splenomegaly
Enlarged LNs
Hepatosplenomegaly
Prone to infections
Low platelets = bleed
Low RBCs = anaemic

Cancer of childhood
Associated with Downs

Nina Cooper

Chronically raised WCC


Smear cells seen

Depends on stage
If asymptomatic, dont
treat

Auer Rods

Blast Cells

Can treat with chemo


Chemotherapy
Radiotherapy
Radiotherapy
BM transplant
Chemo
BM transplant

Year 3 EMQ Notes

Nina Cooper

CLINICAL HAEMATOLOGY
Blood Film Sign

Cause

Sickle Cells

Sickle Cell Anaemia

Spherocytes

Hereditary Spherocytosis

Dacrocytes

Myelofibrosis

Target Cells

Obstructive jaundice
Liver disease
Haemoglobinopathies
Hyposplenism

Rounded Macrocytes

Alcoholic liver disease

Oval Macrocytes

Vitamin B12/folate deficiency

Polychromatic Macrocytes

Young RBCs

Irregularly Contracted Cells

Oxidant damage

Elliptocytes

Hereditary

Pencil Cells

Iron Deficiency (elliptocytes)

Fragments

Mechanical stress

Rouleaux

Plasma protein abnormalities

Agglutinates

Antibody causing agglutination

Howell-Jolly Body

Nuclear remnant from cell, due to poor splenic


function

Left Shift

Severe bacterial infection

Toxic granulation

Infection, inflammation, necrosis


Pregnancy

Hypersegmented neutrophil

Lack of vitamin B12/folate

Heinz Body

Haemolytic anaemia

Bence Jones Protein (urine)

Light chain, related to lytic lesions

Fibrin Split Products

DIC

Tests

Hams test: paroxysmal noctural haemoglobinuria


Coombs test: autoimmune haemolytic anaemia
Metabisulfite test: mimics accelerated deoxygenation, for sickle cell
Osmotic fragility test: hereditary spherocytosis
Reticulocyte test: aplastic anaemia, hypersplenism

Year 3 EMQ Notes

Nina Cooper

Stains
Stain
Sudan Black
Myeloperoxidase
Tartrate-resistant acid
phosphatase
Leukocyte alk phos

Mechanism
Preferentially stains myeloblasts
against lymphoblasts
n/a
n/a

Clinical Use
Differentiates AML and ALL
Extramedullary leukaemia
Hairy cell leukaemia

Elevated in polycythaemia vera, essential thrombocytosis and


myelofibrosis
Depressed inparoxysmal nocturnal haemoglobinuria and CML

Blood Transfusion Reactions

Tissue related lung injury


o 2-6hr after transfusion, inflammatory process causes sequestration of neutrophils
within the lungs
o Antibodies form against donor WCs and attack the patients lungs (which share the
HLA antigens)
o Symptoms include fever, hypotension, cyanosis and pulmonary oedema
Immediate haemolytic transfusion reaction
o Immune destruction of transfused cells
o Hypotension, tachycardia, nausea, abdo pain, loin pain
o Occurs within 24hrs
Delayed haemolytic transfusion reaction
o Same symptoms, >24hrs
IgA deficiency ! anaphylactic-type reaction if patient is re-transfused at later date
o Antibodies to IgA are produced during first exposure
o Retransfusion leads to bronchospasm, laryngeal oedema and hypotension
Febrile non-haemolytic transfusion reaction
o WC antibodies react with leukocytes in blood transfusion
o Usually have hx of previous transfusions
o Fevers, rigors and discomfort

Coagulation
Disease

Abnormality

PT

PTT

PLATELETS

Lack of factor V ! no
input from extrinsic
pathway

normal

BLEEDING
TIME
normal

Factor V
deficiency
Warfarin/
Vit K Deficiency

Inhibition of factors
II, VII, IX, X

normal

normal

Glanzmanns
Thrombaesthesia

Platelets lack GLP


IIb/IIIa, so fibrinogen
binding is disrupted

normal

normal

normal

Bernard Soulier
Syndrome

GLP Ib (receptor for


VWF) is deficient

normal

normal

normal

Nina%Cooper%

Year%3%Notes%

ANAEMIA&
CLASSIFICATION&
Microcytic%
%
%
%
%
Macrocytic%

SUBTYPES&
Iron%Deficiency%
%
%
Hereditary:%thallasaemia%
%
B12/Folate%Deficiency%

Haemolytic%

Inherited%
%
%
%
Acquired%

Normocytic%

Acute%
%
Chronic%

CAUSE&
Dietary/Increased%loss%
(Crohns/Coeliac/Menstruation/Bleed)%
%
Alpha/Beta,%Major/Minor%
%
Pernicious:%IF/Parietal%Cell%antibody%
Alcohol%
Hypothyroidism%
Liver%Disease%
Iatrogenic:%methotrexate,%phenytoin%
%
Sickle%Cell%
Hereditary%Spherocytosis%
G6PD%Deficiency%
%
Autoimmune%Haemolytic%Anaemia%
Microangiopathic%Haemolytic%Anaemia%
HUS%
TTP%
DIC%
Metal%Heart%Valve%
%
Blood%Loss%
%
Liver%Disease%
Anaemia%of%Chronic%Disease%(cancer,%
chronic%inflammation,%Rheumatoid%Arth)%
%

Nina Cooper

Year 3 Notes

Rheumatology Overview
ARTHRITIS
RHEUMATOID ARTHRITIS
>30 minutes of stiffness in the
morning
Deforming polyarthropathy
Proximal IPJs affected
Nodules on elbows/tendon
sheaths

o Urethritis
o Arthritis
Most commonly post-GU/GI
infection
See penile ulcers + plaques on
soles/palms
Seronegative
Also get enteropathic arthritis
secondary to IBD

Rheumatoid Hands
Ulnar deviation of fingers
Subluxation of MCPs
Z shaped thumbs
Boutonnieres deformity
Swan necking

ANKYLOSING SPONDYLITIS
Young men
Bamboo spine fusion of the
vertebrae
95% HLA-B27 +ve
? posture

Bloods: RhF (IgM against IgG), anti-CCP

SEPTIC

Treat with DMARDs


Biologics include infliximab
Methotrexate
Etanocept
Affects other parts of body too
Fibrotic lung disease
Seen as part of Feltys syndrome
o Rheumatoid Arthritis
o Splenomegaly
o Neutropenia
Caplans Syndrome
o Rheumatoid Arthritis
o Pneumoconiosis
o Lung Nodules on XR

ARTHRITIS
Inflammation of joint
Will be just ONE swollen hot joint
S.aureus or GU infection on
aspirate

CONNECTIVE TISSUE DISEASE


SLE

Autoimmune
Drug induced: penicillamine/gold
Malar rash
Fatigue
Joint pain

Bloods: anti-dsDNA, anti-histone, antismith

PSORIATIC ARTHRITIS
Looks like rheumatoid but has
skin/nail changes
Affects people who have psoriasis

Management: immunosuppression
Hydroxychloroquine
Biologics
Acute: steroids

OSTEOARTHRITIS
Worst at night
Degenerative
Seen in weight-bearing joints
Affects distal IPJs
Heberdens (distally) and
Bouchards (proximally) nodes

SJOGRENS
Autoimmune attack on exocrine
glands
Schirmers test: for dry eyes
o See lacrimation
o Positive if not producing
enough tears
Also get dry mouth/vagina
Bloods: anti-Ro, antil-La
Can be primary or secondary to
SLE, RA
Symptom constellation = sicca
o Xerostermia

REACTIVE ARTHRITIS
Reiters Syndrome
Cant see, Cant Pee, Cant Climb a
Tree
o Uveitis

Nina Cooper

Year 3 Notes

o
o
o
o

Keratoconjunctivitis
Chronic bronchitis
Vaginal dryness
Signs of arthritis

SYSTEMIC SCLEROSIS
Cutaneous
Anti-centromere
CREST syndrome
o Calcinosis
o Raynaulds
o Esophagitis
o Scleradectomy (thickening
of skin, beak nose, small
mouth)
o Telangiectesis
Diffuse: anti-SCL70 = anti-DNA
topoisomerase
Get CREST syndrome + organ
fibrosis
POLY/DERMATOMYOSITIS
Muscle inflammation + skin
involvement
Proximal muscles affected
Anti-JO1, raised CK
Associated with malignancy
Gottrons papules
Heliotrope rash (purple around
eye)
Shawl sign

CRYSTALS
Gout

Middle aged men


1st MTP of big toe
Swollen, hot, painful
Uric acid crystals
Normal gout
o Negatively birefringent
o Needle shaped
Triggers: meat, alcohol, thiazide
diuretics
Management
o Acute: NSAIDs, colchicine
o Chronic: allopurinol
! Interacts with
azathioprine

Pseudogout
Occurs in knee (and elsewhere)
Calcium pyrophosphate crystals
Pseudogout
o Positively birefringent
o Brick shaped

VASCULITIS
LARGE VESSEL
Giant cell (temporal) Arteritis
o Ix: biopsy showing skip
lesions
o Mx: high dose steroids
o Associated with
polymyalgia rheumatic
Takayasu arteritis
o Japanese women
o Fibrosis of aortic branches
o Pulseless limb
MEDIUM VESSEL
Kawasakis
o Children
o Coronary artery
aneurysms
o Skin manifestations
Polyarteritis nodosa
o Associated with HBV
SMALL VESSEL
Wegeners
Churg-Strauss
o P-ANCA positive
o Asthma
o Eosinophilia
o SE of monteleukast
Microscopic polyangitis
o P=ANCA
o Pulmonary and renal
symptoms
Henoch-Schonlein Purpura
o Children
o Triad
! Purpura on
buttocks/legs
! Abdo pain
! Renal failure

Nina Cooper

EMQ Lecture Notes

Oncology
Carcinoma
Tumour from epithelial surface
Ulcerated mass
Rolled edge
Tissue invasion
Necrosis
Bleeding
Obstruction/perforation of a lumen

Case 4
6M, Ethiopian
4 week hx of swelling jaw
Progressively enlarging
Hx of EBV infection
Minor disruption of teeth
No laryngeal obstruction

Local Excision
For benign/minimally invasive
malignant tumours
Basal cell carcinoma
Small SCC
Pleomorphic adenoma
Lipoma

Burkitts lymphoma

EPONYMOUS NEOPLASMS
Case 1
2M
Abdominal distension,
haematuria
Large nodular mass on left
kidney
Wilms Tumour

Case 2
32M architect, homosexual
2 week hx of indigestion and
dysphagia
Multiple purple bruise-like
lesions on R arm
Kaposis Sarcoma

Case 3
55M
Progressive weakness of L
hand
Wasting of intrinsic muscles of
hand (T1 nerve root)
Mild ptosis and meiosis on L
face
Pancoast Tumour

Case 5
35F
Bulky mass in left breast
Overlying skin is red and
tender
Contour of breast is distorted
Mixed connective tissue and
epithelial elements on biopsy
Brodies Tumour
AKA Phyllodes tumour
Mostly benign, good prognosis
OTHERS
Ewings sarcoma: rare bone
cancer in children (blue cell)
Grawitz tumour: RCC

TUMOUR MARKERS
Case 1
45F
Irregular 4cm lump in breast
Fixed to overlying skin
Bloody discharge from nipple
Ca15-3
High levels suggest metastatic
breast disease

Case 2
56M
Anorexia and jaundice
Bilirubin 350 umol/L, AST 55
IU/L and ALP of 750 IU/L
Gallbladder is palpable
Ca19-9
Pancreatic cancer
Monitors response to therapy

Nina Cooper

EMQ Lecture Notes

Case 3
70M
Difficulty passing urine
Haematuria
Back pain
DRE shows irregular, hard
mass
PSA

Measure of prostate cancer


Treatment success and
progression

Case 4
82F
Constipation, tenesmus and
lower abdominal pain
Lost 5kg in 1 month
CEA

Carcinoembryonic antigen

Case 5
50M, hep C
Jaundice, RUQ pain
Weight loss and appetite loss
Alpha-Fetoprotein
Hepatocellular carcinoma
Malignancy
Hepatocellular Ca

Carcinogen
Aflatoxin B1

Mesothelioma

Abestos

Prostate Ca

Cadmium

Bladder Ca

Aniline dyes/rubber
industry

Endometrial Ca

Oestrogen

GI Lymphoma

Coeliac

Gastric Ca

Pernicious anaemia

Cervical Ca

Unprotected sex
with multiple
partners

Lymphoma

EBV

Nina Cooper

EMQ Lecture Notes

Breast, Lumps, Vascular


GENERAL LUMPS
Case 1
35F
Lump on scalp
Painless, attached to skin
Has a punctum
Sebaceous Cyst
Occurs anywhere with sebaceous
glands (not palms/soles)
Intradermal
Easily infected
Tx: incision

Lymph Node
Inflamed due to infection or
infiltration
Infection
o EBV
o TB
o HIV
o Syphilis
Infiltration
o Malignancy
o Sarcoidosis

Case 5

Case 2

68F
Firm, painless lump on elbow
PMHx: Arthritis and pulmonary
fibrosis

Rheumatoid Nodule
Subcutaneous, collagenous
granulomes
Found on extensor surfaces
(elbows/hands)
In established rheumatoid arthritis
Up to 25% RA patients
Treat RA with methotrexate !
pulmonary fibrosis OR she gets
pulmonary fibrosis due to
inflammation from RA

Case 3

45M
Lump on a scar on abdomen
Recently had a stoma reversal
Present all the time
Solid + itches sometimes

Keloid
Hypertrophy of vascularised
collagen
Extend outside the scar
Tx: excision or steroids
(topical/injections)

Case 4

22M
Lump on jaw
Firm, not fixed to skin

Recently diagnosed with EBV

21M
Painless lump on top of foot
Appeared a few weeks ago
Transilluminates

Ganglion (cyst)
Dorsum of hand or foot
Herniation of synovial joint or
tendon sheath
Thin-walled cysts
Contain mucinous fluid
Bible cyst
Tx: leave it, aspiration or excision
Examining a Lump
6 Ss
Site
Size
Shape
Surface
Smoothness
Surroundings
3 Ts

Tender
Temperature
Transilluminate

Pulsatile

NECK LUMP INVESTIGATIONS


Case 1
27M
Pain in jaw
Hurts when eats

Nina Cooper

EMQ Lecture Notes

Swollen, tender and red


No fever

Sialolithiasis
Stone in Salivary Gland
Sialogram = best investigation
Recurrent, unilateral
Pain, worse on eating

Case 2
66F
Pain in jaw
Small, hard lump adjacent to
earlobe
Getting bigger over past few
months

Parotid Tumour
Unilateral lump near ear
Displaces the earlobe = classic
EMQ
Facial pain = ? parotid tumour
compressing facial nerve
Best investigation = excision
biopsy
Salivary gland tumours
o 80% parotid
o Of these, 80% are
pleomorphic adenomas
o 80% are in superficial
lobes
Tx: surgery +/- radiotherapy
beware of facial nerve (VII)

Acute
Chronic

Unilateral
Mumps/HIV
Stone
Tumour
(fixed)
ALL, sarcoid
etc.

Bilateral
Mumps/HIV
Autoimmune
disease

Case 3
18M
Lump in neck
Anterior border of right
sternocleidomastoid, half way
up neck
Growing slowly, causing no
trouble
Branchial Cyst

Congenital malformation of
branchial arches = therefore
<20yo
Dont tend to be problematic
unless fistulate or become infected
Cholesterol crystals can be found
on FNA
Tx: leave/excise (close to
carotids/jugulars therefore risky)
Best investigation = final needle
aspirate

Case 4
23M
Painless and mobile from sideto-side but not up and down
Sits anterior to
sternocleidomastoid
Carotid

Body Pathology
Either aneurysm or rupture
Best investigation = angiography
Duplex USS cannot differentiation
RARE
Usually firm, but can be soft and
pulsatile
Classically move sideways but not
up/down

Case 5
35F
Uneven borers, smooth,
slightly fluctuant
Not fixed to skin or deeper
structures
Lipoma
Reassure patient
Benign, fatty lump
Occurs wherever fat can expand
(not scalp, palms)
Symptomatic only with pressure
Malignant change is rare

ULCERS
Case 1
65M
Painless wound in R leg
Present for 2 months
3cmx4cm ulcer covering medial
malleolus
Shallow bed is covered in
granulation tissue
Surrounded by sloping edges

Nina Cooper

Venous Ulcer
Caused by superficial/deep venous
incompetence
Gaiter region, covering the medial
malleolus
Lipodermatosclerosis
Brown discoloration (haemosiderin
deposition due to pooling of blood
and iron breakdown)
Extensive, can be painless

Case 2
60M
T2DM
Large ulcerating lesion over left
toes
Lesion has well demarcated
adges
Leg is cool to touch
Hairless
Dorsalis pedias and posterior
tibial pulses are absent
Arterial Ulcer
AKA ischaemic
Caused by large vessel disease
(atherosclerosis) or small vessel
disease (vasculitis)
Located distally
Punched out appearance
Absent pulses
Low ABPI
Shiny, hairless skin
Arteriopath, smoking, diabetes,
CVD
VERY painful

EMQ Lecture Notes

Case 3
27F
Altered bowel habit and rectal
bleeding
Large ulcerating lesion on left
leg with dark red edges
Pyoderma Gangrenosum
Recurring nodulo-pustular ulcers,
with tender red/blue necrotic edge
Purulent
Associated with IBD, but also
alcoholic hep, Wegeners,
myeloma

Case 4
60M, obese
Ulceration on sole of right foot
Deep and penetrating skin
Appears well perfused
No pain
Ankle reflexes are absent
bilaterally
Neuropathic
Repetitive trauma with
absent/reduced sensation
Diabetes
Lack of ankle reflexes
Seen on weight-bearing areas

Nina Cooper

Case 5
66M
Retired marines officer
Large scar from burn sustained
in a war
Painless 3cm area of irregular
ulceration within scar
Surrounded by raised edge
Blood-stained discharge from
base
No regional lymphadenopathy
Marjolins Ulcer
Eponymous name for a SCC
arising in an area of previous
inflammation e.g. long-standing
venous ulcer or third degree burn
(clue: WAR VETERENS)
Features include
o Painless
o Blood-stained discharge
o Raised edge
o LN spread rare

NECK LUMPS
3 Main Locations for Neck Lumps
Anterior Triangle
o Branchial cyst
o Carotid body tumours
o (ABC)
Midline
o Thyroiglossal cust
o Thyroid goitre
o Dermoid cyst
Posterior Triangle
o Cervical ribs
o Cystic hygromas
o Pharyngeal pouches
o (2C, 2P)
Anywhere
Lymph nodes
Lipoma
Sebaceous cysts

EMQ Lecture Notes

Carotid

Body Tumour
Babies
Tumour of paraganglion cells
Moves side/side but not up/down
Pressure ! dizziness
Pulsatile and has a bruit
Diagnosed: carotid angiography

Case 1
30M
Lump in posterior triangle
Enlarging slowly for 2 months
Feverish, itchy, sweaty
Lost 8kg unintentionally
Lymphoma
Lethargy
Weight loss
LN painful after alcohol
Night sweats
Itchy

Case 2
25M
Painful swelling bulging from
under anterior border of
sternocleidomastoid
Soft, fluctulant but tender on
examination
Present since was a child
Started causing trouble after
recent pneumonia
Branchial Cyst
2/3 up sternocleidomastoid
Remnant of second branchial cleft
Smooth, non-tender fluctuant
lump
Enlarged + painful after
respiratory infection
Present from childhood
Diagnosed: FNA showing yellow
fluid with cholesterol crystals

Case 3
70F
Mass in right jaw
Slowly growing for 6 months
Past week unable to move
right side of face
Hard, irregular lump extending
behind angle of mandible
Salivary Duct Carcinoma
Hard, irregular pass

Nina Cooper

Facial nerve involvement

Case 4
28M
Lump in posterior triangle of
neck
Difficulty finishing meals
Regurgitates food at night
Pharyngeal Pouch
Weakening and protrusion of
pharyngeal wall through Killians
dehiscense
Elderly man
Dysphagia, regurgitation of food
Gurgling sensation
Aspiration at night
Bad breath

Case 5
3 month old baby
Lump on base of neck,
posterior to stenocleidomastoid
Transilluminates
Cystic Hygromas
Congenital collection of lymphatic
sacks
Clear fluid
Soft, fluctuant

VASCULAR SURGERY
Case 1
Conditions met to qualify for
elective AAA repair
AAA Repair
Greater than 5.5cm + otherwise
fit for surgery, OR
Smaller aneurysm that is now
causing symptoms e.g. back pain,
OR
Smaller aneurysm growing at
>1cm/yr
<5.5cm = monthly ultrasound
surveillance
Repair can be open/endovascular

Case 2
56M
Homeless
Severe pain in right leg
Pale compared to left leg, from
below knee to toes

EMQ Lecture Notes

Cold, pulses absent (popliteal,


posterior tibila and dorsalis
pedis)
Fixed flexion deformity
cannot flex knee or toes
Apyrexial, HR 85

Acute Limb Ischaemia


Surgical emergency requiring
revascularisation within 4-6h to
save limb
Mainly due to in situ thrombosis or
emboli
Signs 6 Ps
Pale
Pulseless
Painful
Perishing cold
Paraesthesia*
Paralysis*
*indicates amputation necessary
Management Options
Percutaneous transluminal
angioplasty: short stenosis in large
arteries
Surgical bypass: extensive
atheromatous disease with good
distal run off e.g. femoral popliteal
bypass, femoral distal bypass
Acutely: surgical embolectomy or
thrombolysis
Amputation: useless limb e.g.
flixed flexion deformity OR dead
limb e.g. extensive necrosis
In addition to other signs, LOSS
OF SENSATION or PARALYSIS
point to gangrene
Ankle Brachial Pressure Index (ABPI)
Measured by divided ankle systolic
BP by brachial systolic BP
Should be >1
>1.3 = walls are so calcified that
they cannot be compressed by a
BP cuff
0.6-0.9 = peripheral vascular
occlusive disease e.g. intermittent
claudication
0.3-0.6 = critical limb ischaemia
<0.3 = impending gangrene
Buergers Test
Used to assess arterial supply to
lower limbs

Nina Cooper

Raise patients legs to 45 degrees


and held there for 2 minutes
The time taken for the legs to
show signs of pallor is noted
o Rapid = poor arterial
supply
Legs are then allowed to hang
independently off the side of the
bed
o Rate at which colour
returns = rate of venous
filling
If there is arterial insufficiency,
after 2-3 mins the foot on the
affected side will develop a
reactive hyperaemia ! crimson
coloured foot (due to vasoldilation
from anaerobic metabolics built up
during elevation)
Can be positive in one or both of
the lower limbs

Amaurosis Fugax
Transient monocular loss of vision
Result of atherogenic emboli
arising from the carotid artery
territory, which unilaterally
obstruct the lumen of the retinal
artery circulation
Other causes include cardiac
emboli and temporary vasospasm
of retinal artery (in young people
during exercise)
Giant cell arteritis can cause
chronic granulomatous
inflammation of the central retinal
artery which produces similar
symptoms to amaurosis fugax
Trendelenburgs Tourniquet Test
SFJ is occluded using a tourniquet
Assesses where the level of
venous insufficiency occurs
If the varoscosities do not fill,
then the defect lies above the
tourniquet
Carotid Endarterectomy
Patients with symptomatic carotid
artery stenosis >70-80% are
candidates for carotid
endarterectomy
Simply the procedure to unblock
the carotids
Post operative complications
o 66% get post-op HTN

EMQ Lecture Notes

o
o

5-8% stroke risk (carotid


artery stenosis or
haemorrhagic)
5% CN injury
1% get patch rupture or
infection

Varicose Veins
Asymptomatic ! treat
conservatively
o Avoid long periods of
standing
o Compression stockings
o Lose weight
o Walk more
Or injection sclerotherapy ! for
below knee varicosities
o Increase the amount of
intravascular granulation
tissue
If QOL is severely aggected e.g.
due to pain, ulceration,
thrombophlebitis, then surgery is
an option
o You can ligate the
saphenofemoral/saphenop
opliteal junctions
o Or multiple avulsions of
the varicosities and
endoluminal
radiofrequency ablation of
long saphenous
Low molecular weight heparin has
been shown to be effective for the
use of DVT prophylaxis in those
undergoing surgery
Necrotising Fasciitis
Infection of deep layers of the
skin and subcutaneous tissues
which spreads within the fascial
plain of the tissue
Group A strep = most common
Usually starts at the site of local
trauma
Features include swelling,
erythema, vesicle and bullae
formation on the skin
Very ill with fever +- diarrhoea
and vomiting
Confirm diagnosis with blood
culture and aspiration of pus
Requires debridement
emergency

Nina Cooper

Aortic Dissection
Defined as a surgical emergency
Split in the intimal lining of the
tunica media of the aorta !
double lining or rupture into the
pericardium ! cardiac tamponade
CT scan used for rapid diagnosis
MRI = higher sensitivity and goldstandard, but takes longer
Raynauds Syndrome
Digital ischaemia due to reflex
vasospasm of normal arterioles
Digital pallow ! cyanosis !
reactive hyperaemia
Secondary Reynauds can occur as
a result of scleroderma, SLE,
polyarteritis nodosa, RA, cervical
rib, cryoglobulinaemia,
polycythaemia and drugs such as
B blockers and ergot alkaloids
Popliteal Aneurysms
80% of all peripheral aneurysms
More likely to have an aortic
aneurysm as well
Branches of the external carotid
Superior thyroid
Ascending Pharyngeal
Lingual
Facial
Occipital
Posterior auricular
Maxillary
Superficial temporal

Some Aggressive Lovers Find Odd


Positions More Stimulating

BREAST

EMQ Lecture Notes

Case 1
24F
Breast lump
1.5cm discrete, smooth mobile
lump
Fibroadenoma
Benign neoplasm
Arises from stroma
Most common breast lump in
under 30s
Hard, smooth and mobile
Breast mouse
Fibroadenosis
AKA fibrocystic changes
Lumpy/bumpy chest
Multiple cysts and nodules
Menstrual cycle
Sometimes pain
Oestrogen-induced hyperplasia
Small cysts dilation/unfolding
lobules
Fibrosis rupture of cysts !
fibrosis
Adenosis increase in acini

Case 2
34F
Large lump, hot and painful
Gave birth 6 weeks ago
Acute mastitis
Infection of the breast
Often in the first month of
breastfeeding
Bacteria from skin ! spread to
ducts
Caused by staph aureus, strep
pyogenes, empidermidis
Causes inflammation, pus and
sometimes fever
Abscess more inflammation and
fever, not helped by abx

Case 3
56F
White nipple discharge
Poorly defined periareolar mass
Duct Ectasia
Dilatation
Blockage
Chronic inflammation and fibrosis
Poorly defined palpable periareolar
mass

Nina Cooper

Mostly painless
White cheesy discharge
5th-6th decade
Often multiparous

Periductal Mastitis
Painful erythematous palpable
subareolar mass
Poorly defined periareolar mass
No discharge
Metplasia of squamous cells !
abscess formation ! abscess
formation and inflammation
Not associated with age/parity
>90% smokers

Case 4
31F
Pain in breast
Same time each month
Cyclical Breast Pain
Associated with menstrual cycle
Occurs in luteal phase (after rising
of progesterone)

Case 5
52F
Irregular lump on left breast
Inverted nipple
Unilateral bloody discharge
Skin puckering
Invasive breast carcinoma
Most common cancer in UK
1 in 8 women
Ulceration
Skin retraction
Peau dorange
Swelling
Discharge
Eczema
Nipple retraction or distortion
Lump will be hard, fixated and can
cause skin tethering

Case 6
65F
Itchiness and discharge from
nipple
No lump
DCIS

No palpable lump

EMQ Lecture Notes

Pagets

Disease
Itchy
Erythema
Scale formation
Erosion
Nipple discharge
Occurs in 1-4% breast cancers
60% with Pagets have invasive
carcinoma, 40% have DCIS
(cannot palpate lump)

BREAST INVESTIGATIONS
Case 1
32F
5cm unilateral irregular breast
lump
USS

Anyone under age of 40

Mammography
Anyone over 40
USS + Mammography
Lump and are 40 or over

Case 2
56f
5cm unilateral irregular breast
lump
USS + Mammography
Less dense tissue than younger
woman

Case 3
36F
Hot, red, painful breast lump
Antibiotics
Likely to be acute mastitis
Might be abscess will require
draining if abx dont work

Case 4
26F
Painful breasts at same time
each month
Multiple hard lesions less than
2cm in diameter
Reassure
Can give ibuprofen

Nina Cooper

Case 5
47F
White, low viscosity liquid
discharge from her nipples
Prolactin
White, low viscosity = breast milk
Prolactinoma ! galactorrhoea

EMQ Lecture Notes

Nina Cooper

EMQ Lecture Notes

Endocrinology and Poisoning


ENDOCRINE INVESTIGATIONS
Case 1
20M athlete
Weight loss and fatigue
Unable to gain weight
Drinking water non-stop
Polyuria
Random Blood Glucose
T1DM
Lack of insulin leads to release of
glucagon, corticosteroids,
adrenaline and GH
This results in gluconeogenesis
and glycogenolysis in the liver,
releasing more glucose into the
bloodstream
This process is fed by breakdown
of fat/muscle to fuel the liver
The glucose gets so high that it is
lost in the urine, causing an
osmotic diuresis
This makes you drink more,
causing the polydipsia
When the liver cannot keep up
with the gluconeogenesis and
glycogenolysis, it begins to
produce ketones (acetone smells
like pear)
Ketones = acids, and so trigger
vomiting via the CTZ
This causes Kussmal breathing to
blow off excess CO2 as well

Case 2

65M
Difficulty climbing stairs
Exertional dyspnoea
Has centripetal obesity,
bruises, darkening palmar
reases and recent onset DM

Cushings Syndrome
Due to a SCC links to problems
breathing
Therefore order high dose
dexamethasone test
Acts like cortisol to depress ACTH
in Cushings this doesnt happen
Disease pituitary adenoma

Failure to suppress on
high-dose dexamethasone
and exaggerated cortisol
response to CRH
Cushings syndrome = ectopic
ACTH secretion
o Failure to suppress on
low-dose dexamethasone
o Loss of diurnal variation in
cortisol secretion
o Loss of cortisol response
to an insulin tolerance test
Psuedo-Cushings Syndrome
o Failure of suppression of
serum cortisol on lowdose dexamethasone +
characteristic body
habitus of Cushings
o Elevated MCV and serum
GGT show cause =
alcoholism
o Normalises on cessation of
drinking
o

Case 3

25M
3/12 hx polyuria and polydipsia
PMHx ITU stay at HH following
RTA
Glucose 5.8

Diabetes Insipidus vs. Psychogenic


Polydipsia
Use water deprivation test
DI: get up at night to drink water
CENTRAL = pituitary tumour,
trauma [car crash!] or tumour
treated by desmopressin
NEPHROGENIC = hereditary or
secondary hypercalcaemia,
lithium and demeclocycline
Treatment:

Nina Cooper

Central = intranasal
desmopressin, nephrogen =
thiazide diuretic

Case 4

EMQ Lecture Notes

65M
Hyponatraemia
No medications
Adrenal insufficiency
High urine osmolality

Diagnosis: SIADH
Opposite to DI
Order CXR and head MRI
Symptoms: generalised weakness,
hyporeflexia, ataxia, lethargy,
confusion
Suspect if patient has
hyponatraemia, hypo-osmolality
and a high urine osmolality
Indvestigations
o Decreased osmolality
o Inappropriate urine conc
o Excess Na excretion in
urine
Treat: saline, demeclocycline
Treating Na by >10mmol causes
quadriplegia, pseudobulbar palsy,
seizure, coma, death dont rush
treatment
Use demeclocycline and
tolvapatan if water restriction is
sufficient

Neurones derived from olfactory


epithelium patients typically
present with anosmia or hyposmia
Frequently demonstrate colour
blindness, cleft palate/lip
Treatment = pulsed GnRH

Case 5
65m
Coarse facial features
Widely spaced teeth
Headaches
Ring keeps getting stuck on
fingers
Acromegaly
Visual disturbance
Slanting forehead
Protruding jaw
Oral glucose tolerance test =
diagnosis
o GH drops in increased
glucose
o Wont decrease in
acromegaly
Due to a pituitary tumour
Serium IGF=1: screening test for
acromegaly, levels directly relate
to amount of GH secretion of
previous 24hrs
Treatment = surgery

Case 6
Young female
Low mood, weakness, tearful
Mild GI irritation
Darkened skin creases
Addisons Disease
Short synACTHen test
Inject ACTH ! increased cortisol
Damage to adrenals means
cortisol will NOT increase
GI disturbance, weight loss,
postural htn
High potassium + low sodium due
to lack of aldosterone

Kallmanns Syndrome
Isolated hypogonadotrophic
hypogonadism
Caused by failure of development
of hypothalamic neurones

Case 7
30F
Generalised weakness and
paraesthesia
Severe HTN
Raised sodium, low potassium,
raised bicarbonate

Nina Cooper

Darkened skin creases

Conns Syndomre
Diagnose with plasma
renin:aldosterone level
Hypokalaemia, metabolic low
alkalosis
Can be unilateral or bilateral
Renin low due to excess
aldosterone

Case 8
35F
Visual disturbance
Cross street, cannot see cars
on either side
PMHx hypercalcaemia treated 3
years ago with neck surgery
Pituitary studies are ordered
What else?
MEN1

3 Ps: parathyroid tumour,


pancreatic tumour, pituitary
adenoma
Pancreatic US (Zollinger-Ellison
Syndrome stomach tumour
causing duodenal ulcers)
MEN2b = signs of Marfans
(Lincoln = men2b)
Men2a = phaeochromocytoma,
amyloid producing medullary
thyroid tumour

THYROID DISEASE
Functions of Thyroid
Increase BMR
Physical and mental growth
Maintain responsiveness of body
to adrenergic drive

Case 1
35F
Insominia, palpitations
Lost 5kg over 2 months despite
eating whole KFC bucket every
night
Irritable
Wear shorts
Unsightly rash on shins
Hyperthyroidism

EMQ Lecture Notes

Exophthalmos and pretibial


myxoedema GRAVES
Autoimmune
Overstimulation of TSH-R

Case 2
53F
Weight gain, lethargy,
constipation
Started 5 months ago
Mild, diffusely enlarged thyroid
Hashimotos
Most common cause of
hypothyroidism in western world
Examination: moderately enlarged
non-tender thyroid
Anti-thyroglobulin
Anti-microsomal
Anti-thyroid Peroxidase
Anti TSH-R

Case 3
40F
Mood swings, problems with
attention, hand tremor
Discomfort in neck
Radiates to ears when
swallowing
Flu-like illness recently
De Quervains Thyroiditis
Clue = flu-like illness
Viral infection e.g. flu, coxsacchie,
mumps
Causing inflammation and release
thyroid hormone
Pain worse with swallowing
Fever
Hyperthyroid -> hypothyroid ->
euthyroid
Dx: clinical and high ESR
Treatment: none, NSAIDs

Case 4
61F
Difficulty breathing and
swallowing
Midline neck mass increasing in
size over few months
Thyroid is hard, immobile and
enlarged
No tenderness
USS-guided biopsy shows
increased fibrotic tissue

Nina Cooper

Riedels Thyroiditis
Idiopathic fibrosis of thyroid
Slow growing (rock like)
Biopsy to distinguish from cancer
TFTs normal, some develop
hypo/hyperthyroidism
None, palliative surgery if
affecting oesophagus etc.

Case 5
84F
Lump in neck, growing rapidly
for past month
Problems swallowing
Hard irregular mass on right
side, fixed to overlying skin
Anaplastic Carcinoma
Rare
Elderly
Die within 1 year
Hard, rapidly enlarging
Spread
Causing tracheal compression of
RPN injury

Case 6
25F
Left sided neck lump
2cm smooth, regular, firm
lump that moves upwards
when swallowing but not with
tongue protrusion
Cervical LNs present on left
side of neck also
Papilary Carcinoma
90%
Adolescents and young adults
May spread to LNs
Orphan Annie eyes (cleared-out
appearance)
Psammoma bodies (sady)
Treatment = thyroidectomy +radio-iodine
TSH-dependent so thyroxine for
rest of life
OTHER CANCERS
Follicular Carcinoma
20%
Middle aged
Spreads to lung and bone

EMQ Lecture Notes

Medullary Carcinoma
Calcitonin = tumour marker
Rare 5%
Sporadic or part of MEN2
syndome

CALCIUM METABOLISM
Case 1
49M
Increased tiredness, irritability
and abdo pains
Admitted to hospital with
bones
Raised serum PTH
Primary Hyperparathyroidism
Bones, stones, moans, groans =
hypercalcaemia
Polyuria and polydipsia
Hyperplasia or adenoma or
parathyroid gland
Increased PTH leading to
increased Ca
Causes of Hypercalcaemia
MALIGNANCY: Myeloma and bony
mets also increase serum calcium
VIT D INTOXICATION
SARCOIDOSIS
Causes

of Hypocalcaemia
CKD
Gland failure = autoummune
DiGeorge syndrome
Post-thyroidectomy
Hypomagnesaemia

Pseudohypoparathyroidism
Target cell resistance to PTH
Low Ca and PTH is appropriately
high

Case 2
21F
Thyroidectomy
Numbness in extremities and
muscle spasm
Wrist flexes when taking BP
Hypoparathyroidism
Trousseaus sign = cuff leading to
spasm = carpopaedal spasm
Weakness, crampings, normal
Also Chvosteks sign

Nina Cooper

Case 3
63M
CKD
Tiredness and muscle cramps
Chvosteks sign elicited
Raised PTH
Secondary Hyperparathyroidism
PTH increased due to low serum
Ca
Usually in CKD

Case 4
Same 63M
Admitted to hospital a year
later with severe loin to groin
pain
Increasingly depressed and
irritable
Tertiary Hyperparathyroidism
Prolonged period of secondary
disease
Parathyroids undergo hyperplasia
due to loss of feedback control !
hypercalcaemia

Case 5
56 smoker
SOB, weight loss, haemoptysis
Polyuria and bone pain in left
leg
PTHRP

Secretion
Causes ectopic secretion of PTH
Signs of hypercalcaemia
Pain due to bone breakdown

Case 6
35F
Symptoms of hyperkalaemia
Lump in right neck
Frequent headaches and
palpitations
Normal stature
MEN2a
See primary hyperparathyroidism
Palpitations due to
phaeochromocytoma

EMQ Lecture Notes

MIXED EMQs
Case 1
40M
Weight gain
Stretch marks
Cut wont heal
Cortisol suppressed by high
dose dexamethasone only
Cushings Disease
Disease = ACTH secreting
pituitary adenoma
Syndomre = ectopic sources e.g.
adrenal adenoma, SCC

Case 2
Female
Received several units of blood
Amenorrhoea
Inability to lactate
Sheehans Syndrome
Ischaemic necrosis of pituitary
Hypopituitarism

Case 3
26F
HTN
Muscle cramps
Weakness
Serum K = 2.9
Conns Syndrome

Case 4
60f
oVERWEIGHT
Loss peripheral vision
Straie and alrge scars on lower
abdomen
Had recent surgery not sure
what for
Nelsons Syndrome
Bilateral adrenalism for Cushings
! existing pituitary adenoma
grows rapidly (no negative
feedback) ! visual problems,
hyperpigmentation due to
increased ACTH production

Nina Cooper

Case 5
30F
Weight loss, tachycardia
Feels hot all the time
Clubbing
Painful swelling of digits
Eyes are bulging out of orbits
Graves Disease
Thyroid acropachy
Exophthalmos

Case 6
49F
Lump in neck
Hoarse voice
Riedels Thyroiditis
See above

Case 7
22M
Sore throat recently
Feels heart racing
Weight loss
Painful lump in neck
De Quervains
See above
Toxic Multinodular Goitre
Lumpy irregular swelling of thyroid
No eye sides
Can cause Plummers Syndrome:
autonomous hyperfunctioning
nodules
Simple Colloid Goitre
Increased gland production of
colloid due to increased thyroxine
demand
Low T4 and high TSH
Simply due to iodine deficiency or
pregnancy/puberty

Case 8
64M
2 month hx of weight loss
Hoarse voice
Difficulty swallowing
Lymphadenopathy
Orphan Annie Eyes on
histology

EMQ Lecture Notes

Papillary Tumour
Younger patients

MANAGEMENT OF DIABETES
Case 1
26F
A&E via LAS
Clutching stomach, doesnt
know where she is
Eyes are sunken
IV Saline
Dehydration
DKA ! characterised by hyperglycaemia,
ketoacidosis, ketonuria
Mortality of 3-5%
Complete lack of insulin !
increase in glucagon, cortisol,
growth hormone, epinephrine
Stimulates hepatic
gluconeogenesis, glycogenolysis,
lipolysis
Lack of insulin leads to fatty acid
metabolism and causes increased
ketone production
Presentation
Nausea and vomiting due to
hydroxybutyrate
Sweet smelling breath due to
acetone
Severe dehydration - glycosuria,
osmotic diuresis
Ketoacidosis due to ketones
Confusion: hyperosmolarity,
dehydration and acidosis
Diffuse abdominal pain
Management
IV normal saline DEHYDRATION
kills
One drip is set up, then start
insulin infusion
Correct hypokalaemia

Case 2
54M
Diabetic
HbA1C of 8.5% six weeks after
being started on metformin
Sulphonylurea
HbA1C >6.5 ! metformin

Nina Cooper

Sulfonylurea given if not


overweight e.g. glibenclamide
(urea pisses off B cells and causes
them to produce more insulin)
If HHbA1C>6.5 still !
sulphonylurea + metformin
Still not good and >7.5 !
thiazolidinediones or insulin
Then if >7.5 = insulin +
metformin + sulphonylurea

HbA1c
Chromatography of Hb gives HbA
and HbA1 (4-6%)
HbA1c = glycosylated form of
HbA1
Reflects blood glucose over 120
day life span of erythrocyte
50% comes from last month, 25%
from month before
6.5 MAGIC NUMBER

Case 3
65M, white
BP 155/85 on 2 consecutive
visits to GP
Calcium Channel Blocker
Over 55/black = Ca Channel
blocker,
Under 55 = ACE inhibitor
Step 2: A+C
Step 3: A+C+D (thiazide-like
diuretic)

Case 4
67F
DM
Suddenly becomes aggressive,
shouts at nurses for
imprisoning her

EMQ Lecture Notes

Calms down after few minutes


and appears to be falling
asleep

Hypoglycaemic give IV glucagon


ABCDEFG
Neuroglycopenia
Irritable, tired, drowsy etc.
3 states
o Conscious, orientated,
able to swallow ! 15-20g
carbohydrate
LUCOZADE
o Conscious but confused,
disorientated, unable to
cooperate, aggressive but
are able to swallow !
dextrogel squeezed into
mouth, IM glucagon
o Unconscious and
or/having seizuires and/or
very aggressive ! IV
Glucose or IM glucagon

POISONING
Case 1
78F
Attempted suice by OD on
prescription medicine
Fatigued, bradycardic and can
see yellow-green halos
Digoxin OD
Affects cardiac ion
channels increases IC
calcium
Increased contractility
and increases vagal tone
to heart
Dig ! AVN block
Enhanced automaticity !
arrhythmias and supraventricular
tachycardia OR bradycardia due to
HB
Yellow-green visual halos =
xanthopsia
Digibind = antidote

Case 2
14F
Paracetamol OD
10 x 250mg tablets

Nina Cooper

EMQ Lecture Notes

Weighs 50kg

Reassure and Discharge


For paracetamol toxicity must be
<75mg per kilo, 75-150mg per kg
= borderline, 150mg per kg =
hepatocellular damage
This pt has taken 2500mg overall,
which is 50mg per kilo therefore
she is not at risk
Otherwise you would give Nacetylcysteine

Took mothers anxiety


medication
Ends in pam
RR is 6

Benzodiazepine
Flumezanil
-Ams bind to inhibitory GABA-A
receptors
Solitary OD causes sleepiness
With alcohol it causes severe resp
depression

Paracetamol
Primary cause of liver failure
First few hours = nausea and
vomiting, over 24-48hrs you get
hepatotoxicity
Signs = tenderness, jaundice,
asterixis, foetor hepaticus,
haemorrhage (II, VII, IX, X)
95% paracetamol detoxified by
phase II enzymes and excreted by
urine
5% converted to NAPQI via P450
enzymes! conjugated with GSH
Too much paracetamol enzymes
are all occupied and so excess
NAPQI is produced ! destructive
in liver as not enough glutathione
to bind
Therefore need to give NAC to
allow for more glutathione
production and to conjugate the
NAPQI
NAC is most effective in first 8
hours

Case 4
4 yr old child
15 year old brother notes
brother drank bottle of
mothers bright green
medication
Has respiratory depression and
pinpoint pupils

KNOWN
DOSE
<1 hr
CHARCOAL

Aspirin (salicylate)
Stimulates resp centres to
hyperventilate ! resp alkalosis
Aspirin = acid therefore metabolic
acidosis
Vomiting
Dehydration
Hyperventilation
Give ACTIVATED CHARCOAL
straight away
Measure urine pH
Give IV sodium bicarbonate until
becomes alkaline
Haemodialysis if renal failure,
heart failure, seizures

<8hrs:
measure
levels, NAC
if above line

UNKNOWN
DOSE
<8 hours,
measure
levels, treat
if above line
8-36hrs
TREAT
>36hrs,
treat if
above line

Case 3
16F
Drunk

UNKNOWN
Treat

Naloxone
Prescription meds such as
tramadol can be used for OD
Bright green liquid = methadone
Resp depression
Pinpoint pupils

Case 5
75M
OD on prescription medication

ABG shows mixed respiratory


alkalosis and metabolic acidosis
Seizure

Nina Cooper

Poison

EMQ Lecture Notes

Digoxin

Key Features
Tachycardia, bradycardia, yellow halo

Antidotes
Digibind

Paracetamol

Tell you/ hepatic failure

N-acetyl-cysteine

Benzodiazepines

Drowsiness, respiratory depression

Flumezanil

Opiates

Resp. depression, pinpoint pupils

Naloxone

Salicylates

Tinnitus, vertigo, hyperventilation


Metabolic acidosis, mixed respiratory alkalosis and
metabolic acidosis

First 2 hours: Charcoal


Urinary alkalisation:
sodium bicarbonate

Organophosphorous
insecticides

Farmer, inactivate cholinesterase: SLUD =


salivation, lacrimation, urination, diarrhoea

Atropine

Heparin

Bleeds

Protamine

Warfarin

Bleeds

Vitamin K

Beta blocker

Severe bradycardia or hypotension

Atropine and Glucagon

Carbon monoxide

Low sats but normal blood gas

100% oxygen
Hyperbaric chamber

Paraquat

Weed killers, alveolar failure

Activated charcoal

Nina Cooper

EMQ Lecture Notes

Infectious Diseases & Eponymous


Syndromes
INFECTIOUS DISEASES
Case 1:
Elective in India
Profuse rice watery stools
Cholera
Rice water stools
Gram -ve
DIARRHOEA
Shigella
Bloody diarrhea
Acute presentation
Gram ve
E-Coli

Common diarrhea
Enterotoxigenic
Most common in traveller

Salmonella typhi
Gram ve motile bacilli
Gradual onset
Rose spots
C. Diff
Gram +ve
Pseuodomembranous colitis
Campylobacter
Self-limiting
Hx of poultry or unpasteurized milk
Gram ve flagella, corkscrew
Yersinia
Plague
Gm ve facultative
Anaerobe
Enteroamoeba
Parasite
Giardia
Long-term diarrhea
Water contamination

Case 2:
Elective in Cape town
Mild fever, headache

Malaise

Influenza
Cape Town in August = their
winter
Therefore, no mosquitos &
increased risk of flu
Dengue
Retrobulbar headache
Rash
Viral (RNA)
Can cause facial flushing
Dengue shock syndrome !
haemorrhagic fever
Malaria
Falciparum = worst
Non-specific symptoms: headache,
fever, vomiting, diarrhoea
Plasmodium Ovale/vivax stay in
system V&O do not go
Influenza
Non-specific symptoms: headache,
fever, vomiting, diarrhoea
A: birds
B: seals
C: pigs
TB

Latent infection

Herpes virus
HSVI/II: cold sores/genital
III: varicella zoster chicken pox
and shingles
IV: EBV, infectious mononucleosis
V: CMV, immunocompromisied:
systemic symptoms: fever,
pneumonitis, colitis, hepatitis,
retinitis, CVS
VI/VII: rosida infection (children)
VIII: Kaposis sarcoma (popular
blotches): AIDS defining illness

Case 3:
Travelling businessman
Returns from Africa
Headache and malaise

Nina Cooper

No response to chloroquine,
responds to quinine

Malaria
Quinine/chloroquine are
antimalarials
2-3 treatments
o Old: chloroquine (falc. now
resistant)
o Quinine = new, best
treatment
o Artesanate since 2011/12

Case 4:
23F
Gap year
Returns to UK
Presents with bloody diarrhoea
Shigella

Case 5:
Professor of ID
Returns from South America
3x3 erythematous ulcer on left
forearm, raised edge
Leishmaniasis
Cutaneous: itchy papules, becomes
erythematous ulcers with raised
edges
Visceral (Kala Azar): spreads via
lymphatic and reticuloendothelial
system; lymphadenopathy and
constitutional system
Mucocutanoues: lip, nose, pharynx
etc.
Protozoa
Spread by sandlifes
Schistosomiasis
Lyme Disease

EMQ Lecture Notes

Case 7:
Elective in Cape town
HOSPITAL ACQUIRED INFECTIONS
Klebsiella
Gram ve rod
Pneumonia: currant jelly sputum
Renal calculi (staghorn)
Others: proteus (gm ve bacilli),
pseudomonas (gm ve)
C. Difficile
Gm +ve
Abx use
Toxic megacolon
Pseudomembranous colitis
PCP

Pneumoncystis jivoreci
Pneumonia in CD4<200; ITU
intubated
CXR unrevealing
Histoplasmosis: ddx

Case 8:
6 year old
Itchy popular rash
skin burrows in hand palms and
finger webs
Scabies

Case 9:
Gardener presents to A&E with
inability to eat
Greets you with sardonic smile
Tetanus

Case 10:
42yo South African man
Blotches over legs
Enjoys sexual intercourse

Case 6:
21yo Rockstar
Acute pain in right knee
Hot and swollen to touch
Gram ve diplococci

HIV ! Kaposis sarcoma

Neisseria gonorrhoea
Rockstar: promiscuous?
Chlamydia/gonorrhea = most
common STDs

Case 11:
HIV +ve man
Headache and fevers
Ring enhancing features on CT

JC Virus: causes PML, AIDS-defining, hand


moves around without him realising

Nina Cooper

Toxoplasmosis
ANTIBIOTICS
Lung Pathogens
Typical
o Strep. Pneumoniae
o Pneumococcus
o Rusty brown sputum
o Penicillin: amoxicillin
Atypical
o Legionella
" LFTS: rise
" Low Na
o Mycoplasma
o E.g. dry cough, walking
pneumonia
o Macrolide:
clariothromycin,
erythromycin
Exceptions
C. diff: metronidazole
Pneumonocystis carinii: cotrimoxazole
HIV+ or immunosup.
Boat shaped organisms on BAL
with silver stain
UTIs

Trimethroprim
Nitrofurantoin

Case 1:
25m
Previously well
1 week history of SOB, fever
and productive cough
Dullness to percussion
Increased tactile vocal fremitus
in right base
Amoxicillin
Streptococcal infection
Nb. Flucloxacillin is for staph.
aureus infections of the skin

Case 2:
38 prostitute
1 week hx of dry cough and
SOB on exertion
Normal CXR
Co-trimoxazole
Pneumocystis carinii
Prostitute ! HIV
Normal CXR

EMQ Lecture Notes

Case 3:
29 businessman
3 day business trip
Unwell for a week with muscle
cramps and SOB
Wife is upset as he forgot her
birthday and never normally
does
Na = 128, K = 4.6
Clarithromycin
Legionella
In Edinburgh: probably in a hotel
Can get CNS abnormalities:
forgetful
Low Na

Case 4:
49F
1 week hx of lower abdominal
pain
Pain on urination
Finding job increasingly difficult
as she has to urinate so often
Nitrofurantoin
HEPATITIS B SEROLOGY
Used to measure immunity following HVB
immunization: Hepatitis B surface
antibody (HBsAb)
Used to indicate acute HVB infection:
Hepatitis B surface antigen (HBsAg)
Used to indicate chronic HVB infection if
detected 6 months after original infection:
Hepatitis B surface antibody (HBsAb)
Used to indicate high infectivity in a
chronic HVB carrier: Hepatitis B e
antigen (HBeAb)
High levels indicates low infectivity in a
chronic HVB carrier Hepatitis B core
antibody (HBcAb)

Nina Cooper

Surface antigen: outside, first to be shed


If antigen is found in someones
blood, theyve got Hep B
First way to detect a positive
infection result
If under 6 months, acute
If over 6 months, chronic
If VACCINATED, you will find the
surface antibody only
Core antigen: inside
The core antigen should never be
found in the blood
The core antibody will be present
in the blood and shows infection
present after 4 weeks
o IgM: acute
o IgG: chronic
o If HBcAB is high, you are
mounting a good response
and youre not highly
infective
E antigen: extra, not always there
Indicates a high level of infectivity
Requires aggressive treatment
Absence = low infectivity
TB DRUGS: RIPE
RIPE: for 2 months
RI: for following 4 months
Isoniazid: peripheral neuropathy
(give with pyridoxine vit B6)
Rifampicin: orange tears/urine,
hepatitis, drug interactions
Pyrazindamide: hepatotoxicity
Ethambutol: optic neuritis (redgreen colour blindness)
DIARRHOEA CTD.

Case 1:
32M
Profuse, watery diarrhoea
Abdo cramps during trip to
mexico
Resolves without antibiotics
E. coli

(travellers diarrhea)
Can infect adults/kids
Virulence factors affects severity
ETEC: enterotoxigenic travellers
diarrhea, no invasion/inflammation
EIEC: invasive ! necrosis and
inflammation, blood and pus in
stool
EHEC: causes HUS

EMQ Lecture Notes

Case 2:
22 student
Abdo cramps, D&V
Eaten a chinese take away from
the night before
Gm +ve bacilli
Bacillus

cereus
1-6h intubation period
Reheated rice
Sudden vomiting, minimal nonbloody diarrhea
Self-limiting

S. aureus
Similar
Gram +ve clusters
1-6h incubation
Prominent vomiting

Case 3:
82F
Hospitalised for CAP
Fever and profound diarrhoea
after 3 days in hostpital
White/yellow membrane-like
plaques on colonic mucosa
Biopsy shows plaques made of
fibrin and inflammatory cells
C. Difficile
#1 HAI
Anaerobic, spore-forming, gm +ve
Follows cephalosporins/
fluoroquinolones
2 exotoxins
o A: diarrhea
o B: cytotoxic to colonic cells
Diagnostic in stools
Treat with oral/IV metronidazole
Oral vancomycin if doesnt work

Case 4:
Couple
1 week hx of watery diarrhoea,
foul smelling stools and
gas/abdo cramps
Cysts and trophozoites
Giardia

Eastern Europe/Russia
Hikers
MSM
Bloating, abdo pain, malabsorption
of protein

Nina Cooper

Fat and fould smelling non-bloody


diarrhea
Pear shaped trophozite cysts found
in stool
Treat with metronidazole

Entamoeba Histolytica
Flask-shaped ulcers
Bloody diarrhea
RUQ pain (liver absecesses)

EPONYMOUS SYNDROMES
Case 1:
20yo man
Hemisection spinal cord
Cannot move right leg but
responds to pain
Cannot feel pain in left leg
Brown Squard Syndrome
Spinothalamic tract: crosses at
level of SC
Contralateral loss of fine
touch/pain/temperature

Case 2:
49M
Increased urination and thirst
Weakness and muscle cramps
Conns Syndrome
Cushings Syndrome
Syndrome: outside pit
Disease: within pituitary
Nelsons
Cushings within pituitary
Post bilateral adrenalectomy for
pituitary adenoma
High cortisol levels pre-surgery !
suppressed CRH
Lowered cortisol ! increased CRH
Unchecked growth of pituitary
adenoma
Hyperpigmentation (POMC) !
alpha
Wermers: MEN1
Sipples: MEN2

Case 3:
30F
A&E

EMQ Lecture Notes

Feltys

Stiff hands, wrists and feet


Pain worse in morning
Palpable spleen
Low neutrophils
Raised CRP
Syndrome
Rheymatoid arthritis
Splenomegaly
Neutropenia

Case 4:
55M
A&E
Confused
Gaze palsy
Thiamine started immediately
Condition improves significantly
days later
Wernickes encephalopathy
Chronic alcoholism/severe
nutritional deficiency
Opthalmoplegia (horizontal
nystagmus)
Confusion
Ataxia
Because thiamine is a cofactor for the
enzymes responsible for glucose
metabolism, infusion of glucose will
exacerbate the pre-existing thiamine
deficiency precipitates Wernickes
encephalopathy.
Korsakoffs Syndrome
Irreversible
Anterograde and retrograde
amnesia
Confabulation

Case 5:
34 M asthmatic
Wrist drop
Eosinophilia and autoantibodies to neutrophil
myeloperoxidase
Churg-Strauss Syndrome
Small vessel vasculitis
Palpable purpura
Peripheral neuropathy (wrist, foot
drop)
Eosinophilia
Late onset asthma

Nina Cooper

Histology: granulomatosis
vasculitis
P-ANCA (myeloperoxidase)

Case 6:
42F
Spinning sensation
Nausea and ringing in left ear
Always uses right ear when
using phone
Feels fine between episodes
Mnires Disease
Inner ear disorder
Increased volume of endolymph
Spontaneous episodic vertigo
Unilateral tinnitus
Unilateral sensorineural hearing
loss
Treat by relieving pressure

Case 7:
32M
Weakness and tingling
Started in feet, now at knees
Getting worse
2/5 strength in both legs,
absent reflexes in knees
PMHx: resp infection
Guillain Barre Syndrome
1-28 days post resp/GI infection:
CMV/EBV/campylobacter
Molecular mimicry to myelin =
autoimmune response to
peripheral nerves
Motor > sensory
Ascending symmetrical paralysis
Can affect autonomic NS ! cardiac
irregularitis, HTN/HypoTN
LP find albumino-cytological
dissociation
Mechanical ventilation, treatment
with IVIG/plasma exchange

Case 8:
2 weeks post MI
Central chest pain, worse lying
down and on inspiration
Friction rub on auscultation
Dresslers Syndrome
Autoimmune
1 week ! few months post MI

EMQ Lecture Notes

Pericarditis: sharp, pleuritic chest


pain, pericardial friction rub and
pleural effusions on XR
Give aspirin

Nina Cooper

EMQ Lecture Notes

Emergencies & Harder EMQs


ULCERS
Case 1
65M
Painful wound on leg
Knocked on coffee table 4
weeks ago
ABPI = 1.1
Appears to have eczema
Venous

Painful
Shallow
Flat
Pink base
Granulation tissue
Normal ABPI
Due to venous insufficiency (deep
or superficial)
Lipodermatosclerosis +
haemosiderin deposition

Case 2
65F
Wound on heel
Punched appearance
0.5cm deep
Doesnt hurt
Neuropathic
Alcoholic, diabetes
Microvascular angiogpathy and
peripheral neuropathy
Punched out
Painless
Deep
Pink base
Tips of toes/heels/malleoli

Case 3
70M
Painful wound on dorsum
Smoked for 40 years
Punched out appearance
Intermittent claudication
Arterial

Painful
Cold
Pulseless
Fibrotic base

ABPI

Same sites as neuropathic


Distal sites
Paucity of hair, shiny skin, pale,
nail dystrophy
>1.0 normal
<0.3 = impending gangrene
0.3-0.6: pain at rest
0.9-0.6: claudication
Falsely high ABPI if incompressible
vessels

Case 4
45m
Sore on finger
Painless
Well defined borders
Small flat spot that grew and
became an open wound
Infective
Typical of syphilis
3-90 day incubation period
Chancre of primary syphilis
Painless
Well-defined
Singular
Firm
Lesion appears at point of contact

Case 5
17M
Melaena
Admitted to hospital with
severe burns
Curlings Ulcer
Acute duodenal ulcer
Mucosal sloughing after plasma
volume depletion

EMERGENCIES
Case 1
65M
Headaches and tender scalp
Stiff and painful shoulders in
the morning for 2 weeks
Jaw hurts when chewing

Nina Cooper

Prednisolone 40-60mg/d PO
Temporal arteritis
Pain due to assoc. with
polymyalgia rheumatic
Risk of sudden blindness
Raised ESR, CRP, platelets
Temporal artery biopsy
Tx: decrease prednisolone after 57 days; continue for 2 days

Case 2
22M
Known epilepsy
Having seizure
Started convulsing 30 mins ago
>1 seizure, did not regain
consciousness
100% oxygen
Always do ABC
Status epilepticus: seizures >30
mins without intervening
consciousness
Mortality and brain damage
increase with attack length
Aim to stop in <20mins
Remember, not only tonic-clonic

Same 22 year old man is still


convulsing
! Lorazepam 2-4mg IV bolus

Wait 2 minutes and repeat previous


treatment - still convulsing
! Phenytoin IV

Anaesthetist arrives
! General Anaesthetic

Case 3
33F
A&E after RTA
Chest pain
Right side is hyperresonant and
reduced breath sounds
Trachea is deviated to the left
Large-bore cannula to 2nd ICS, midclavicular line

EMQ Lecture Notes

Case 4
55M
Massive haematemesis
Low BP, tachycardic
5th finger of hand is bent close
to palm
Terlipressin 2mg SC qds
Oesophageal Varcies
Shocked? (HR>100, BP <100,
postural drop, urine output
<30ml/hr)
Yes: haemodynamically stabilise
then endoscopy
No: 2 big cannulae, resuscitate,
endoscopy
Terlipressin: vasopressin analogue
splanchnic vasoconstrictor
Then balloon tamponade

ACUTE ASHTMA MANAGEMENT


Case 1
13M
6/12 hx of coughing at night
and wheeze
Otherwise well
Small for his age
Inhaled short-acting beta 2 agonist

Case 2
25F
Known asthma
A&E ! SOB
Widespread wheeze
Too breathless to talk
HR 115, RR 36
Oxygen + short-acting B2 agonist
(nebulised) + steroids
Severe asthma
60% oxygen via facemask
Back to back nebs
Systemic steroids
Monitor PEFR every 15 mins
Serial ABGs

Case 3
12F
Poorly controlled asthma

Nina Cooper

Managed with LABA,


monteleukast and high dose
steroids
Mother asks if there is any
other medication she can take?

Oral Steroids
Moving from step 4 ! step 5

Case 4
25F, pregnant
Mild asthma
Refuses to put any chemicals
into her body as she has read
it might harm the baby
Breathing techniques

Case 5
35M
A&E with SOB
Barely talk, silent chest, RR:36
Improvement with oxygen,
salbutamol nebs and steroids
Peak flow still only 30% of best
IV magnesium sulphate
Acute severe and not responding
to treatment
Or if life-threatening attack

CALCIUM/PARATHYROID
Case 1
50M
Polyuria, constipation, lethargy
Corrected calcium 2.8 (2.2-2.6)
PTH is significantly raised
Primary Hyperparathyroidism
High PTH, high Ca
PTH can be normal, but this is
inappropriate if the Ca is high
Usually benign parathyroid
adenoma
Classical bone disease: osteitis
fibrosa cystica

Case 2
10F
Chronic renal failure secondary
to PKD
Corrected Ca of 1.9
Phosphate of 2.5 (0.7-2.4)

EMQ Lecture Notes

Significantly raised PTH

Secondary Hyperparathyroidism
Poor kidney function ! calcium
loss
Loads of PTH produced but
kidneys still cannot function well
enough to retain Ca
Therefore high PTH and low Ca
Assoc. with renal osteodystrophy

Case 3
40F
Thyroidectomy yesterday
Tingling lips
General weakness
Ca 1.8
Hypoparathyroidism
Trousseaus and Chvosteks sign
High phosphate, low Ca

Case 4
75M
Weight loss, haemoptysis
Smoking hx
Corrected calcium of 2.8mmol/l
Low PTH
PTHrp secretion
Assoc. with squamous cell
tumours
Common cause of raised calcium
in malignancy
Treat with fluids and
bisphosphonates
If refractory: use calcitonin

Case 5
25F
CRF has a corrected Calcium of
2.6mmol
Phosphate of 2.1
Significantly raised PTH
Tertiary Hyperparathyroidism
History of CRF
PTH significantly raised, raised Ca
Autonomous function of
parathyroid glands after secondary
stage

Nina Cooper

SBAS
Case 1
Benzodiazepine OD
Flumazenil
Do not give if mixed BZD+TCA, as
TCAs reduce seizure threshold

Case 2
Afebrile pt ! blood results
Markedly raised ALP
Ca is 2.5
Pagets

disease of the bone


Headache
Back pain
Hat doesnt fit
Isolated raised ALP

Other sources of ALP


ALP: osteoblast action, liver,
placenta, kidneys
Signs of biliary obstruction (ALP >
AST/ALT)

ECG SIGNS
Case 1
Sinus tachycardia
S I, T III, Q III
Pulmonary Embolus
Pain, SOB, tachycardia

Case 2
Flattened T waves
Wide PR and U waves
Also ST depression
Hypokalaemia
Hyperkalaemia: tall, tented T waves, flat P
waves, wide QRS
Hypocalcaemia: prolonged QT, symptoms
of low Ca (tetany, carpopedal spasm)

Case 3
ST elevation in all leads
Pericarditis
Saddle shaped in all leads

EMQ Lecture Notes

Case 4
Irregular rhythm
Sawtooth pattern
Atrial Flutter
Re-entry circuits in atria
Shows a heart block (1:2/3/4)

Case 5
Tall prominent P in leads II, III
and aVF
Right atrial hypertrophy
P pulmonale
Due to cor pulmonale
Tall peaked p wave
Lead II
LA hypertrophy
P mitrale
Bifid P wave split into 2 and
prolonged
Best in V1
LVH

Deep S in V1/2
Tall R in V5/6
LAD

Case 6
ST elevation in V1-6
Anterior MI

Case 7
Pathologic J waves
Hypothermia

Case 8
Tall R waves
ST depression
Inverted T waves in leads V1-3
Posterior MI

ANTIHYPERTENSIVES
Case 1
58M
GP ! BP 220/100
Smoker
Smoothly enlarged prostate,
PSA normal

Nina Cooper

Doxazosin
Enlarged prostate
Give alpha 1 blocker ! for BPH

Case 2
Obese 47F
170/90
Blood glucose 11.3
Cholesterol 6.9
Ramipril

Case 3
32F
24 weeks gestation
Lethargy and malaise
BP 160/80
Protein in urine
Methyldopa
Alpha 2 agonist
Only antihypertensive safe in
pregnancy

Case 4
62 smoker
Episodic chest pain
Relieved by rest
Normal glucose
BP 160/90
Atenolol
Got angina
Therefore give beta blocker to
treat HTN and angina

Case 5
86 male
6 weeks post MI
Dry cough
Aspirin, metoprolol, lisinopril
and GTN spray
GP stops lisinopril and
exchanges it for another
Losartan

EMQ Lecture Notes

Nina Cooper

EMQ Lecture Notes

Treatment of Status Epilepticus


ABC: Basic Resus

100% O2

IV Phenytoin
General
Anaesthesisa
! Paralysis
! Ventilation

18mg/kg at rate of
<50mg/min

Or
IV Diazepam
- 100MG IN 500ML
OF 5% DEXTROSE

IV Cannula
Correction of hypotension, glucose etc.
Give thiamine if alcoholism suspected

BENZODIAZEPINE:
IV lorazepam, PO midazolam, IV/PR diazepam

Chromosomal Syndromes
Name
Downs Syndrome

Abnormality
Trisomy 21

Key Features
Epicanthal folds
Round face
Small eyes
Single palmar crease

Edwards
Syndrome

Trisomy 18

Small head
Mostly female
Cleft lip/palate
Multiple heart defects
Widely spaced eyes
Ptosis
Rocker bottom feet
nd rd
Webbed 2 /3 toe

Patau Syndrome

Trisomy 13

Polydactyly
Cyclopia
Low set ears
Abnormal genitalia
Dextrocardia

Turners Syndrome

45X

Webbed neck
Amenorrhoea
Short stature
Wideley spaced nipples
Visual problems
Hearing loss

Klinefelters
Syndrome

47XXY

Hypogonadism
Small testicles
Low testosterone
Gynaecomastia

Image

DiGeorge
Syndrome

Microdeletion at
22q11

Congenital heart
disease
Palate defect
Learning disability
Recurrent infections
Hypertelorism
Characteristic facial
features

Williams
Syndrome

Microdeletion at
7q11

Elfin facial appearance


Low nasal bridge
Mental disability
Heart defect
Visuo-spacial disability

Cri-du-Chat

5p deletion

Cat-like cry
Unusual facial features
Behavioural problems
Hypotonia
Microcephaly

Triple X Syndrome

47 XXX

Not distinguishable from other females

Hyponatraemia
Anorexia, nausea, malaise, headache, irritability,
confusion, weakness, seizures, cardiac failure, oedema
Dehydrated
Not Dehydrated
Urine osmolality
Oedematous
>20mmol/L:
Nephrotic syndrome
Addisons
Cardiac failure
Renal failure
Cirrhosis
Diuretic excess
Renal failure
Osmolar
diuresis
Urine Osm >500
SIADH
Urine osmolality
<20mmol/L
Osm <500mmol/kg, no
Diarrhoea
oedema
Vomiting
Water overload
Fistulae
Severe
Burns
hypothyroidism
Trauma
Glucocorticoid
CF
insufficiency
Heat

Hypernatraemia
Lethargy, thirst, weakness,
irritability, confusion, coma, fits +
signs of dehydration
Water

loss in excess of Na loss


Diarrhoea, vomiting, burns
Excess Saline
Diabetes insipidus
Osmotic diuresis
Primary aldosteronism

Give water po or 5% dextrose IV

!
Hypokalaemia
K < 2.5mmol/L

K+ >6.5mmol

Muscle weakness, hypotonia, hyporeflexia,


cramps, tetany, palpitations, arrhythmia

Fast irregular pulse, chest pain, weakness,


palpitations, light-headedness

Small/inverted T waves, prominent U waves,


prolonged PR, depressed ST

Tall tented T waves, small P waves, broad


QRS, ventricular fibrillation

Causes

Causes

Diuretics
D&V
Cushings/steroids/ACTH
Conns syndrome
Alkalosis
Bulimia
Renal tubular failure

Treatment: oral K

Hyperkalaemia

Oliguric renal failure


K-sparing diuretics
Rhabdomyolysis
Metabolic acdisosis
Addisons
Burns
ACE-I

Treatment: underlying cause

!
Hypocalcaemia
SPASMODIC:
S: spasms
P: periorbital paraesthesiae
A: anxious, irritable
S: seizures
M: muscle tone increased in smooth
muscle (colic, wheeze, dysphagia)
O: orientation impaired, confusion
D: dermatitis
I: impetigo herpetiformis
C: chvosteks sign
Raised Phosphate

Hypercalcaemia
Bones, Stones, Groans, Moans
Abdominal pain
Vomiting
Constipation
Polyuria/polydipsia
Depression
Anorexia, Weight loss
Tiredness, Weakness
HTN
Confusion
Renal stones/failure
Ectopic calcification

CKD
Hypoparathyroidism
Pseudohypoparathyroidism
Acute rhabdomyolysis
Vitamin D deficiency
Hypomagnesaemia

Normal/low Phosphate
Osteomalacia
Acute pancreatitis
Over-hydration
Resp alkalosis

Causes

Malignancy
Primary hyperparathyroidism
Renal failure
Addisons
TB
Vitamin D overdose
Raynauds
Muscle tumour
Endocrine tumours

!
Urea
Urea: waste product

Creatinine
Indicative of renal function

Causes of hyperuricaemia:
Drugs: cytotoxics, thiazides, loop diuretics,
pyrazinamide
Increased Cell Turnover: lymphoma,
leukaemia, psoriasis, haemolsis,
rhabdomyolysis, tumour lysis syndrome
Reduced Excretion: primary gout, CKD,
hyperparathyroidism, pre-eclampsia
Other: htn, hyperlipidaemia

Rise = damage to nephrons therefore use


to monitor levels of kidney disease
GFR: more sensitive indication of degree of
renal impairment

Urate ! precipitation in renal tubules !


renal failure

!
ACIDOSIS
Metabolic
Low pH, low HCO3

Respiratory
Low pH, High CO2

Raised Anion Gap


Increased
production/decrease
d excretion of
fixed/organic acids
Lactic acid (shock.
Infection, tissue
ischaemia)
Urate (renal failure)
Ketones (DM,
alcohol)
Drugs/toxins
(salicylates,
biguanides,
methanol)

T2 Respiratory failure
Most commonly
COPD

Normal Anion Gap


Loss of
HCO3/ingestion of
H+
Renal tubular

ALKALOSIS
Metabolic
Respiratory
High pH, high HCO3
High pH, low CO2
Vomiting
Hypokalaemia
Burns
Ingestion of base

Hyperventilation

acidosis
Diarrhoea
Addisons
Acetazolamide

!
1. Look at pH: acidosis = <7.35, alkalosis >7.45
2. Is the CO2 abnormal? Normal = 4.7-6.0
If the CO2 is raised with an acidosis picture = respiratory
cause
If the CO2 is raised with an alkalotic picture = compensatory
change for a metabolic cause
3. Is the HCO3 normal? Normal = 22-28
HCO3 is alkaline ! raised in alkalosis
Therefore, high HCO3 and alkalosis picture = metabolic
High HCO3 and alkalosis picture = compensatory
The Anion Gap
Estimates everything else which is hard to measure directly
Gap = (Na + K) (Cl + HCO3)
Normal = 10-18

Vous aimerez peut-être aussi